2ちゃんねる ■掲示板に戻る■ 全部 1- 最新50    

■ このスレッドは過去ログ倉庫に格納されています

■ちょっとした物理の質問はここに書いてね180■

1 :ご冗談でしょう?名無しさん:2014/06/16(月) 21:27:39.13 ID:BcEjvbOD.net
前スレ
■ちょっとした物理の質問はここに書いてね179■
http://ai.2ch.net/test/read.cgi/sci/1400668015/

★荒らし厳禁、煽りは黙殺
★書き込む前に   >>2   の注意事項を読んでね
★数式の書き方(参考)はこちら   >>3-5   (予備リンク:   >>2-10    )
===質問者へ===
重要 【 丸 投 げ 禁 止 】

・質問する前に
1. 教科書や参考書をよく読む
2.
http://www.google.com/
  などの検索サイトを利用し、各自で調べる
3. 学生は自分の学年、物理科目の履修具合を書く
4. 宿題を聞くときは、どこまでやってみてどこが分からないのかを書く
5. 投稿する前に、ちゃんと質問が意味の通る日本語か推敲する、曖昧な質問文には曖昧な回答しか返せない
・「力」「エネルギー」「仕事」のような単語は物理では意味がはっきり定義された言葉です、むやみに使うと混乱の元
・質問に対する回答には返答してね、感謝だけでなく「分からん」とかダメOK
・質問するときはage&ID表示推奨
・高度すぎる質問には住人は回答できないかもしれないけれど、了承の上での質問なら大歓迎

===回答者へ===
・丸投げは専用スレに誘導
・不快な質問は無視、構った方が負け
・質問者の理解度に応じた適切な回答をよろしく
・単発質問スレを発見したらこのスレッドへの誘導をよろしくね
・逆に議論が深まりそうなら新スレ立てて移動するのもあり
・板違いの質問は適切な板に誘導を
・不適切な回答は適宜訂正、名回答は素直に賞賛

2 :ご冗談でしょう?名無しさん:2014/06/16(月) 21:28:24.71 ID:???.net
数式の書き方の例 ※適切にスペースを入れると読みやすくなります
●括弧: (), [], {}を適切に入れ子にして分かりやすく書く
●スカラー: a,b,...,z, A,...,Z, α,β,...,ω, Α,Β,...,Ω,...(「ぎりしゃ」「あるふぁ〜おめが」で変換)
●ベクトル: V=(v1,v2,...), |V>,V↑, (混乱しないならスカラーの記号でいい。通常は縦ベクトル)
●テンソル: T^[i,j,k...]_[p,q,r,...], T[i,j,k,...; p,q,r,...]  (上下付き1成分表示)
●行列: M[i,j], I[i,j]=δ_[i,j] M = [[M[1,1],M[2,1],...], [M[1,2],M[2,2],...],...], I = [[1,0,0,...],[0,1,0,...],...]
(右は全成分表示。行または列ごとに表示する。例:M=[[1,-1],[3,2]])
●対角行列: diag(a,b) = [[a,0],[0,b]]
●転置行列・随伴行列:M^T, M†("†"は「だがー」で変換可) ●行列式・トレース:|A|=det(A), tr(A)
●複号: a±b("±"は「きごう」で変換可)
●内積・外積: a・b, a×b
●関数・汎関数・数列: f(x), F[x(t)] {a_n}
●平方根: √(a+b) = (a+b)^(1/2) = sqrt(a+b) ("√"は「るーと」で変換可)
●指数関数・対数関数: exp(x+y)=e^(x+y) ln(x)=log_e(x) (底を省略して単にlogと書いたとき多くは自然対数)
 括弧を省略しても意味が容易に分かるときは省略可: sin(x) = sin x
●三角関数、逆三角関数、双曲線関数: sin(a), cos(x+y), tan(x/2), asin(x)=sin^[-1](x), cosh(x)=[e^x+e^(-x)]/2
●絶対値:|x| ●ノルム:||x|| ●共役複素数:z^* = conj(z)
●階乗:n!=n*(n-1)*(n-2)*...*2*1, n!!=n*(n-2)*(n-4)*...

3 :ご冗談でしょう?名無しさん:2014/06/16(月) 21:28:51.85 ID:???.net
●微分・偏微分: dy/dx=y', ∂y/∂x=y_x ("∂"は「きごう」で変換可)
●ベクトル微分: ∇f=grad f, ∇・A=div A,∇xA=rot A, (∇^2)f=Δf ("∇"は「きごう」,"Δ"は「でるた」で変換可.)
●積分: ∫[0,1] f(x)dx = F(x)|_[x=0,1], ∫[y=0,x] f(x,y)dy, ∬[D] f(x,y)dxdy, ∬[C] f(r)dl 
("∫"は「いんてぐらる」,"∬"は「きごう」で変換可)
●数列和・数列積: Σ[k=1,n] a(k), Π[k=1,n] a(k) ("Σ"は「しぐま」,"Π"は「ぱい」で変換可)
文脈によっては単に同じ添字が2回出てきただけで a_i b_i = Σ[i] a_i b_i と積の総和をとることも(Einsteinの縮約)
●極限: lim[x→∞] f(x) ("∞"は「むげんだい」で変換可)
●確率・期待値:P(x), <x>=E(x)
●論理・集合: "⇔⇒∀∃∧∨¬∈∋⊆⊇⊂⊃∪∩"は「きごう」で変換
●等号・不等号: "≠≒<>≦≧≪≫"は「きごう」で変換

読みやすい書き方の例:∫[-∞,∞] exp{ -Σ[i,j=1,n] A_[i,j](x_i)(x_j) } dx = √(π^n/det A)

読みにくい書き方の例:∫[-∞,∞]exp(-Σ[i,j=1,n]A_[i,j]x_ix_j)dx=√(π^n/det A)

4 :ご冗談でしょう?名無しさん:2014/06/16(月) 21:29:21.62 ID:???.net
質問・回答に標準的に用いられる変数の例

a:加速度、昇降演算子 A:振幅、ベクトルポテンシャル B:磁束密度 c:光速 C:定数、熱・電気容量
d:次元、深さ D:領域、電束密度 e:自然対数の底、素電荷 E:エネルギー、電場
f:周波数 f,F:力 F:Helmholtzエネルギー g:重力加速度、伝導度
G:万有引力定数、Gibbsエネルギー、重心 h:高さ、Planck定数 H:エンタルピー、Hamiltonian、磁場
i:虚数単位 i,j,k,l,m:整数のインデックス I:電流、慣性モーメント j:電流密度・流束密度
J:グランドポテンシャル、一般の角運動量 k:バネ定数、波数、Boltzmann定数 K:運動エネルギー
l,L:長さ L:Lagrangian、角運動量、インダクタンス m,M:質量 n:物質量 N:個数、トルク
M:磁化 O:原点 p:双極子モーメント p,P:運動量、圧力 P:分極、仕事率、確率 q:波数
q,Q:一般化座標、電荷 Q:熱 r:距離 R:抵抗、気体定数 s:スピン S:エントロピー、面積 t,T:時間 T:温度
U:ポテンシャル、内部エネルギー v:速度 V:体積、ポテンシャル、電位
W:仕事、状態数 x,y,z:変数、位置 z:複素変数 Z:分配関数

β:逆温度 γ:抵抗係数 Γ:ガンマ関数 δ:微小変化 Δ:変化 ε:微小量、誘電率 θ:角度 κ:熱伝導率
λ:波長、固有値 μ:換算質量、化学ポテンシャル、透磁率 ν:周波数 Ξ:大分配関数 π:円周率 ρ:(電荷)密度、抵抗率
σ:スピン τ:固有時 φ:角度、ポテンシャル、波動関数 ψ:波動関数 ω:角振動数 Ω:状態密度

5 :ご冗談でしょう?名無しさん:2014/06/17(火) 21:42:20.32 ID:xKf0bFJm.net
電流は単純な回路なら回路内のどこでも一定なのに
水流は落下すると大きくなりますよね?
なぜですか?

6 :ご冗談でしょう?名無しさん:2014/06/18(水) 00:08:32.26 ID:???.net
Gr(p)=1/(p^2+mr^2+O(p^4))をくりこみ条件と解釈することはできますか?

Grは(運動量空間上の)くりこまれたグリーン関数、mrはくりこまれた質量とします。
Oはオーダーゼロの意。

7 :ご冗談でしょう?名無しさん:2014/06/18(水) 06:47:55.19 ID:???.net
>>5
単純な回路とはなんでしょうか
落下する水路は単純な水路でしょうか

8 :ご冗談でしょう?名無しさん:2014/06/18(水) 08:03:15.26 ID:???.net
http://lyman.c.u-tokyo.ac.jp/~hachisu/lecture/astronomy/astronomy.pdf
ブランク分布の積分でp19でなぜ上半面の立体角で積分するのですか?

9 :ご冗談でしょう?名無しさん:2014/06/18(水) 08:16:22.17 ID:???.net
>>8
図18.4 p81に書いてあると書いてある

10 :ご冗談でしょう?名無しさん:2014/06/18(水) 08:27:01.26 ID:???.net
>>5
電気回路と水循環路は違うと言いたいのだろうが、電気回路でも真空中で電子を加速できる。
速度が速くなると密度が小さくなるから一定。電流と水流は巨視的に連続で同じになる。

11 :ご冗談でしょう?名無しさん:2014/06/18(水) 08:51:23.42 ID:???.net
意味不明

12 :ご冗談でしょう?名無しさん:2014/06/18(水) 09:07:19.84 ID:C9/znp3a.net
水流に例えなきゃいいんだよ

13 :ご冗談でしょう?名無しさん:2014/06/18(水) 09:07:39.33 ID:???.net
頭の固すぎる人には分からないだけ

14 :ご冗談でしょう?名無しさん:2014/06/18(水) 09:13:58.26 ID:???.net
脳の柔軟性試験、「コイルと質量は同じ」と言ってみて反応を見る。

15 :ご冗談でしょう?名無しさん:2014/06/18(水) 09:53:28.72 ID:???.net
ブラックホール内部からこちらに向かってくる光の速度はCですか?

16 :ご冗談でしょう?名無しさん:2014/06/18(水) 11:57:21.71 ID:???.net
コイルは水流中のはずみ車
コンデンサはゴム膜やばね仕掛けの付いた隔壁

17 :ご冗談でしょう?名無しさん:2014/06/18(水) 13:33:17.68 ID:???.net
>>16
マックスウェルはコンデンサの力学的張力から変位電流を推論したのかもしれない。

18 :ご冗談でしょう?名無しさん:2014/06/18(水) 14:41:45.83 ID:???.net
ダイオードは逆止弁だし、トランジスタもゴム膜その他で実現できるもんね
これで水流論理回路ができるはずだから組み合わせてCPU作った人とかいないのかな

19 :ご冗談でしょう?名無しさん:2014/06/18(水) 15:13:26.72 ID:???.net
>>18
デジタルのCPUは複雑すぎて難しいが、アナログコンピュータは20世紀初頭に
発明され実際使われていた。回転運動メカや電子回路による力学シミュレータで
電子回路の素子が加減・微積分演算できるオペアンプ。

20 :ご冗談でしょう?名無しさん:2014/06/18(水) 16:46:39.27 ID:???.net
>>6
できる。が、
>Oはオーダーゼロの意。
記号の定義は分かってるの?

21 :ご冗談でしょう?名無しさん:2014/06/18(水) 17:14:58.08 ID:???.net
横だけど、よくわかんねー

22 :ご冗談でしょう?名無しさん:2014/06/18(水) 18:07:18.79 ID:???.net
>>18
流体論理素子というのはあるよ。流体素子でぐぐってみて。
機械的な可動部はない

23 :ご冗談でしょう?名無しさん:2014/06/18(水) 19:23:56.34 ID:???.net
>>20
ご明察!
そこはよく分かりません。
Gr(p)の極の位置が問題なんでしょうか。
また、この条件を課すことは物理的には何を要請することになるんでしょうか。

24 :ご冗談でしょう?名無しさん:2014/06/18(水) 19:26:45.03 ID:???.net
エスパーごっこかよ

25 :ご冗談でしょう?名無しさん:2014/06/18(水) 19:47:54.40 ID:???.net
>>23
微積分をやり直し

26 :ご冗談でしょう?名無しさん:2014/06/18(水) 20:52:44.35 ID:???.net
>>9
「輻射は上半面に放出されるので」
と書かれていますがこの理由もよくわかりません

27 :ご冗談でしょう?名無しさん:2014/06/18(水) 20:58:45.78 ID:???.net
>>25
>記号の定義は分かってるの?
とは、「O(p^4)という表記の意味を理解しているか」という問いですか?
てっきり「O(p^4)を要請する理由を理解しているか」という問いだと勘違いしました。
というか、「定義」を「意義」と読み違えました。

O(p^4)という表記の意味するところは理解しているつもりです。
オーダーゼロという言い方は悪かったと思います。失礼しました。

しかし、>>6の式をくりこみ条件と呼ぶ意味は分かりません。

28 :ご冗談でしょう?名無しさん:2014/06/18(水) 21:20:03.24 ID:???.net
>>26
輻射と観測者の関係を考えてるんじゃないの
エネルギー密度との関係も18章でやってる

29 :ご冗談でしょう?名無しさん:2014/06/18(水) 22:05:08.77 ID:???.net
>>26
こちらに向いてない半球の分は考えないというだけの話

30 :ご冗談でしょう?名無しさん:2014/06/18(水) 22:10:45.44 ID:???.net
>>27
横だが、何を聞きたいのかさっぱりわからん
ラグランジアン?
繰り込み条件?

31 :ご冗談でしょう?名無しさん:2014/06/18(水) 22:48:52.41 ID:???.net
>>29
輻射であるのに放射状(球全体)について考えなくてもよいのですか?
勉強不足で何度も質問してしまって申し訳ないです

32 :ご冗談でしょう?名無しさん:2014/06/18(水) 23:13:25.32 ID:???.net
高1で、三角比についてです

33 :ご冗談でしょう?名無しさん:2014/06/18(水) 23:14:47.81 ID:???.net
sinθ=1/3の時、cosθ、tanθを求めよ

34 :ご冗談でしょう?名無しさん:2014/06/18(水) 23:15:54.59 ID:???.net
cosθ=√8/3
tanθ=1/√8
ではないんでしょうか?

35 :ご冗談でしょう?名無しさん:2014/06/18(水) 23:38:33.09 ID:???.net
>>30
>>6の式がくりこみ条件として機能する理由を知りたいです。

36 :ご冗談でしょう?名無しさん:2014/06/19(木) 00:04:38.93 ID:???.net
>>31
星の表面から内側に向かう分は入れたらかえっておかしいだろう。

37 :ご冗談でしょう?名無しさん:2014/06/19(木) 00:06:14.88 ID:???.net
>>35
君が言う「くりこみ条件として機能する」とはどういうことか詳しく

38 :ご冗談でしょう?名無しさん:2014/06/19(木) 05:27:04.14 ID:???.net
宇宙は何個あるのでしょうか?無限個でしょうか?

39 :ご冗談でしょう?名無しさん:2014/06/19(木) 07:25:17.49 ID:???.net
>>38
聞いていい?離婚したって本当?
(元?)嫁さんカレーという架空の存在で何をしたいの?

40 :ご冗談でしょう?名無しさん:2014/06/19(木) 07:49:58.39 ID:???.net
>>38
非加算無限

41 :ご冗談でしょう?名無しさん:2014/06/19(木) 08:23:45.62 ID:???.net
>>6
問:こういう問題が分かりません
答:問題設定が分からない

問:でも知りたいのです
答:・・・

42 :ご冗談でしょう?名無しさん:2014/06/19(木) 09:33:03.79 ID:???.net
>>40
どういうこと?

43 :ご冗談でしょう?名無しさん:2014/06/19(木) 12:56:02.62 ID:???.net
ひきこもり30代後半で童貞で低収入イラストレーターやってるバカ発見。
足立区に住んでいるそうだ
http://inumenken.blog.jp/archives/6580395.html

44 :ご冗談でしょう?名無しさん:2014/06/19(木) 14:06:05.45 ID:???.net
>>40
加算でなく可算(uncountable)だろ

45 :ご冗談でしょう?名無しさん:2014/06/19(木) 18:07:51.41 ID:MdfbXdV5.net
>>18
水はしらんけど油圧回路・空圧回路がそれにあたるんじゃね

46 :ご冗談でしょう?名無しさん:2014/06/19(木) 18:14:09.24 ID:???.net
アホな質問なのだが、磁力を使ってプラズマを作る事ってできるのかな?
調べてたらよく分からなくなった

47 :46:2014/06/19(木) 18:22:30.70 ID:???.net
解決しました。

48 :46:2014/06/19(木) 18:45:49.72 ID:???.net
>>47
いや、してないですよ!?

49 :ジェイ・ディ:2014/06/19(木) 22:09:59.07 ID:CQfB647l.net
>46
不可能
磁力は、集める力であり、プラズマは放つ力
属性が正反対だ

50 :ご冗談でしょう?名無しさん:2014/06/19(木) 22:20:19.49 ID:???.net
>>37
「これを定めておけば量子補正として現れる発散をくりこみにより取り除くことができる」という条件だと思っています。

51 :ご冗談でしょう?名無しさん:2014/06/19(木) 23:46:27.49 ID:???.net
>>50
>>6 の条件で波動関数と質量の繰り込み定数が決まるから、それで紫外発散が吸収できなかったらその理論が繰り込み不可能てことでは?

52 :ご冗談でしょう?名無しさん:2014/06/20(金) 00:22:30.65 ID:???.net
>>51
>>6の条件において紫外発散が吸収できない理論はくりこみ不可能な理論である」

>>6がくりこみ条件である」
は同値だと思います。

私が知りたいのは「なぜ>>6がくりこみ条件と呼べるか」です。

53 :ご冗談でしょう?名無しさん:2014/06/20(金) 00:24:11.23 ID:???.net
>>52
繰り込み条件って規格化条件みたいなもんで「繰り込み可能条件」とは別物だぞ。

54 :ご冗談でしょう?名無しさん:2014/06/20(金) 00:41:42.06 ID:???.net
>>52
その条件で繰り込めるかどうか確かめたら。そもそものラグランジアンはどんな?

55 :ご冗談でしょう?名無しさん:2014/06/20(金) 00:54:25.86 ID:???.net
>>53
なるほど。
いろいろと間違っていたようです。
では、>>6の式は一体どういう条件なのでしょうか?

56 :ご冗談でしょう?名無しさん:2014/06/20(金) 00:59:17.30 ID:???.net
>>55
>O(p^4)という表記の意味するところ
を書いてみて

57 :ご冗談でしょう?名無しさん:2014/06/20(金) 01:17:39.70 ID:???.net
>>49
おおなるほど。やっぱり無理かあ・・・ありがとうございます!

58 :ご冗談でしょう?名無しさん:2014/06/20(金) 01:17:41.40 ID:???.net
高3,力学と波の基礎的部分のみ把握している状態です。

長さ10.0m,質量0.0490gの弦を1.5mの長さに切って80gのおもりをつり下げ水平に張る。
この弦を振動させると腹が3つの定常波ができた。重力加速度=9.8m/s^2

1.この弦の振動数はいくらか。

2.弦の長さを2/1にして腹の数を4個にし、1と同じ振動数の定常波を発生させるには
おもりの質量をいくらにすればいいか。

この問題を解くためにv=fλとv=√S/ρを用いるまでは分かったのですが、そこからの求め方がよく分からないです。

59 :ご冗談でしょう?名無しさん:2014/06/20(金) 01:29:15.48 ID:???.net
>>58
その式に使われている量と問題文に与えられている量の関係をつければいい

60 :ご冗談でしょう?名無しさん:2014/06/20(金) 01:39:42.57 ID:???.net
>>56
セルフエナジーがゼロってことです。

61 :ご冗談でしょう?名無しさん:2014/06/20(金) 01:41:17.59 ID:???.net
>>56
p^4が無視できるほど小さいということです。

62 :ご冗談でしょう?名無しさん:2014/06/20(金) 01:43:10.12 ID:???.net
>>60
ランダウ記号のO(x)を知らないってこと?

63 :ご冗談でしょう?名無しさん:2014/06/20(金) 01:58:07.60 ID:???.net
>>59

頑張ってるけどなかなか分からない…。
S=0.08kg*9.8m/s=0.784
ρ=0.0000490kg*10=0.0000049kg/m
S/ρ=16000 √外して400
ここまでは合ってますか?

64 :6:2014/06/20(金) 02:54:20.04 ID:???.net
>>62
>>61>>6です。

65 :ご冗談でしょう?名無しさん:2014/06/20(金) 03:15:08.99 ID:???.net
無視できるっていうか、
Gr(p)^(-1)-p^2-mr^2が精々p^4の定数倍だってことじゃね
まぁ「精々p^4の定数倍だから無視する」ってことかもしれんが

66 :ご冗談でしょう?名無しさん:2014/06/20(金) 06:08:29.86 ID:???.net
解決したんだ、よかったね
高次の微分が相互作用に入ってると・・・かと思ったが

67 :ご冗談でしょう?名無しさん:2014/06/20(金) 07:07:40.27 ID:???.net
いや、解決はしてなくね
俺65だけど6じゃないし、6への回答は分からん

68 :ご冗談でしょう?名無しさん:2014/06/20(金) 08:15:27.87 ID:???.net
>>55
だから規格化条件みたいなもんだって。
大雑把に言うと繰り込みは∞引く∞を有限にする訳だけど「有限になるべし」というだけでは「引く∞」の有限部分は決まんないからなんか条件つけとく。

69 :ご冗談でしょう?名無しさん:2014/06/20(金) 08:17:55.96 ID:???.net
もうほっとけよ、レポート丸投げなんだろ

70 :ご冗談でしょう?名無しさん:2014/06/20(金) 09:16:44.28 ID:???.net
>>49
これはまた香ばしいな。

>>57
静磁場はエネルギーを運ばないからプラズマは作れないが、
電磁波はプラズマ作れるよ。当然元になる物質は他に必要だけど。

71 :ご冗談でしょう?名無しさん:2014/06/20(金) 09:49:45.07 ID:???.net
電磁波のエネルギー E=hν と
電磁場のエネルギー密度 u=1/2(εE^2 + 1/μ B^2)
の関係はどう考えればいいんですか?

72 :ご冗談でしょう?名無しさん:2014/06/20(金) 11:04:00.84 ID:???.net
>>71
> 電磁波のエネルギー E=hν と
> 電磁場のエネルギー密度 u=1/2(εE^2 + 1/μ B^2)
> の関係はどう考えればいいんですか?
一つ目は光子一個のエネルギー。電磁波は複数の光子が励起したものなので特に矛盾はない。

73 :ご冗談でしょう?名無しさん:2014/06/20(金) 11:04:18.72 ID:???.net
>>71
電磁場を量子化した光子のエネルギーがhνで対応する波動関数はベクトルポテンシャルA。
A^2が確率密度になる。 電場・磁場はAの微分形。

74 :ご冗談でしょう?名無しさん:2014/06/20(金) 11:17:52.90 ID:???.net
>>68
「くりこんだらこういう有限の値になるべし」という規格化条件の中でくりこまれた質量mrを定義するのはかなり恣意的な行為に思えます。
くりこみという操作自体恣意的な性格を(一部に)持っているのかもしれませんが、>>6のような条件を与える必然性はあるのでしょうか。
「必然性」ほど強い言葉じゃなくとも、そうするのが自然だと思える理由はあるのでしょうか。

75 :ご冗談でしょう?名無しさん:2014/06/20(金) 13:40:10.29 ID:???.net
>>74
繰り込み条件は場合に応じて便利なように選ぶ。

76 :ご冗談でしょう?名無しさん:2014/06/20(金) 13:47:41.50 ID:???.net
>>74はゆとりか

77 :ご冗談でしょう?名無しさん:2014/06/20(金) 13:57:53.65 ID:???.net
>>72
>>73
ありがとうございました

78 :ご冗談でしょう?名無しさん:2014/06/20(金) 15:26:13.03 ID:???.net
>>58
> 2.弦の長さを2/1にして
特に意味も無く分母が1だと、2分の1を 2/1 と書いてしまったのではと心配...とりあえず2倍として、
弦が同じ長さで腹が 4つなら波長は 3/4 倍、そのまま長さを2倍すれば、
λ'/λ = (3/4) * 2 = 3/2
となってる事がわかる。

> この問題を解くためにv=fλとv=√S/ρを用いる

f は固定なので v'/v = λ'/λ = 3/2
また張力は重りの質量に比例するので、
m'/m = S'/S = (v'/v)^2 = (3/2)^2 = 9/4
m' = 9/4 m = 180 g

79 :ご冗談でしょう?名無しさん:2014/06/20(金) 16:01:39.24 ID:???.net
横だけど>>6関連の問答がよく分からん
>場合に応じて便利なように選ぶ
のは良いとして、どういう場合に>>6を採用するのかって話じゃないのか?

80 :ご冗談でしょう?名無しさん:2014/06/20(金) 16:04:16.01 ID:???.net
>>6が何を聞いてるのかわからにので終了

81 :ご冗談でしょう?名無しさん:2014/06/20(金) 16:32:10.93 ID:???.net
>>79
>>6の条件はzero-momentum subtraction schemeだろ。例えば
http://hitoshi.berkeley.edu/230A/Yukawa.pdf
"is easier for the purpose of seeing that all physical amplitudes are finite,"
つまり>>50 を確かめやすい条件。だから>>6はぐだぐだ言ってないで手を動かせって話。

82 :6:2014/06/20(金) 16:38:26.48 ID:???.net
皆様、丁寧なお導きどうもありがとうございました。

83 :ご冗談でしょう?名無しさん:2014/06/20(金) 16:45:14.34 ID:???.net
と質問には答えず去って行った

84 :ご冗談でしょう?名無しさん:2014/06/20(金) 22:50:17.20 ID:???.net
>>70
なるほど、電磁波を使えば可能なのですね。
亀ですみませんがありがとうございます!

85 :ご冗談でしょう?名無しさん:2014/06/20(金) 23:44:40.61 ID:WcSBu03n.net
あのね、どっかの物理のセンセ(確か高校の〜)がね、ニンジンのかけら
を2個つき合わせて電子レンジでチン!しててね、プラズマ作ってたおw
探偵ナイトスクープでやってたお。でもとってもアブナイからやっちゃだめだおw

86 :ご冗談でしょう?名無しさん:2014/06/21(土) 12:27:13.99 ID:hYfloRQQk
ビッグバン直後の光子はいまどうなってるのでしょうか?

87 :ご冗談でしょう?名無しさん:2014/06/21(土) 13:32:59.41 ID:keh9b70R.net
電池をつなげたとき
内部抵抗は考えない(電流=0)として
電圧は厳密に2倍になるんですか?
ちょっと少ないなんてことはないんですか?

88 :ご冗談でしょう?名無しさん:2014/06/21(土) 13:33:36.94 ID:keh9b70R.net
電池は理想的な全く同じ電池です

89 :ご冗談でしょう?名無しさん:2014/06/21(土) 14:34:29.72 ID:???.net
フォン・ブラウンとフォン・ノイマンはどっちの方が天才ですか?

90 :ご冗談でしょう?名無しさん:2014/06/21(土) 14:36:01.47 ID:???.net
>>87
理想的電池なんだから厳密に二倍になるさ。
厳密に二倍にならないのなら電圧以外の何らかのパラメーターがあることになる。
それはもう理想的電池じゃない。

91 :ご冗談でしょう?名無しさん:2014/06/21(土) 14:46:59.50 ID:???.net
∞^0=1は自明でしょうか?

92 :ご冗談でしょう?名無しさん:2014/06/21(土) 15:37:14.79 ID:???.net
釣られないぞ

93 :ご冗談でしょう?名無しさん:2014/06/21(土) 18:47:24.17 ID:keh9b70R.net
>>90
そういう意味じゃなくて
理想的な同じ電池にしたのは例えの都合で
内部抵抗を考えなくて良いような状況ではどんな電池を繋いでも線形性が厳密に成り立つのかって話です

94 :ご冗談でしょう?名無しさん:2014/06/21(土) 19:09:34.74 ID:???.net
電池は電位差を与える機構。電位差は加算できる。

95 :ご冗談でしょう?名無しさん:2014/06/21(土) 19:18:13.47 ID:hYBhZ1ry.net
宇宙のモデルには「曲率が正の閉じた宇宙」「曲率0の平坦な宇宙」「曲率が負の開いた宇宙」
があると知りました。
宇宙のはじまりを考えるとき、
閉じた宇宙は有限なので、1点からスタートして膨張したということは納得なのですが、
残りの2つのモデルの場合、宇宙は無限の広さを持つとのことです。
無限の宇宙の場合、どのような状態からスタートしたのでしょう?
感覚的に想像し難い感じです。

96 :ご冗談でしょう?名無しさん:2014/06/21(土) 19:29:01.20 ID:keh9b70R.net
>>94
電位差が加算できるのはわかるけど
電池をどんなに繋いでも一つの電池が作る電位差が一定という理由にはならないと思うのですが

97 :ご冗談でしょう?名無しさん:2014/06/21(土) 19:57:45.10 ID:???.net
>>96
何を心配してんだか、さっぱりわかんない。

98 :ご冗談でしょう?名無しさん:2014/06/21(土) 20:11:01.20 ID:???.net
たぶん質問者の頭の中に何かの物理モデルがあって、それに関する質問なんだろうな
それを本人が言わないのだからこれ以上何も答は出てこないだろ

99 :ご冗談でしょう?名無しさん:2014/06/21(土) 20:32:39.25 ID:???.net
>>95
よくわかりませんが始まりがあるって誰が決めたんでしょうか

100 :94:2014/06/21(土) 20:47:04.19 ID:hYBhZ1ry.net
>>99
遠くの天体を観測した結果、宇宙は過去も今も膨張を続けているらしいです。
過去は今よりも天体同士の距離が近かった、つまり膨張宇宙を前提とすると、
過去に密度が極大になる時点が存在したに違いない。
そこから宇宙が始まったと考えられているみたいですよ。

101 :ご冗談でしょう?名無しさん:2014/06/21(土) 21:14:09.75 ID:7ROyTpe+.net
宇宙が太ってんじゃねえ?

102 :ご冗談でしょう?名無しさん:2014/06/21(土) 21:21:07.78 ID:???.net
>>96
電池の外部に電池を繋いでも内部状態が変わるわけではない。
電流を流さない限り内部状態は変わらず、起電力は変わらない。

103 :ご冗談でしょう?名無しさん:2014/06/21(土) 21:27:22.09 ID:???.net
等価回路のようなものを考えてるのだろうけど、それが本人の知識を越えてるのだろう

104 :ご冗談でしょう?名無しさん:2014/06/21(土) 21:29:03.59 ID:7ROyTpe+.net
量子論 粒子論

105 :ご冗談でしょう?名無しさん:2014/06/21(土) 21:58:26.33 ID:???.net
>>98
これ
以上。

106 :ご冗談でしょう?名無しさん:2014/06/21(土) 23:17:41.70 ID:jy4Liu5m.net
物理の理解のために微分方程式のグリーン関数を利用した解法を知りたいのですが
@dv/dt+kv=αの場合 同次形の解v=Ce^-xに特殊解を加えればいいとのことですが
私の浅知恵で@のグリーン関数を1/kと推測すると右辺が1になってそれっぽいなと。
その1/kにαをかけてα/k @に代入するとこれは特解。
同次形の解を加えればv=Ce^-x+α/k が一般解。
方向性はいいでしょうか? 本なんか見ると非同次項とグリーン関数との部分積分が
特解になっているような感じですがデルタ関数もあいまいな理解であって。
どなたかグリーン関数を使った解法を実例で教えてもらえないでしょうか。
よろしくお願いしますm()m

107 :ご冗談でしょう?名無しさん:2014/06/21(土) 23:22:32.13 ID:jy4Liu5m.net
自己レス e^-xでなくe^-tです

108 :ご冗談でしょう?名無しさん:2014/06/22(日) 00:05:34.36 ID:/D+JtEpg.net
再自己レス e^-t→e^-kt

109 :ご冗談でしょう?名無しさん:2014/06/22(日) 06:43:33.73 ID:???.net
>>106
@の解はv=(d/dt+k)^(-1)α、(d/dt+k)^(-1)を積分核で表示するのがグリーン関数の方法
後はグリーン関数の本を読んでください

110 :ご冗談でしょう?名無しさん:2014/06/22(日) 10:02:39.40 ID:???.net
宇宙に値段を付けるとしたら幾らぐらいなんでしょうか?

111 :ご冗談でしょう?名無しさん:2014/06/22(日) 11:07:42.27 ID:???.net
葡萄板を荒らして10年

112 :ご冗談でしょう?名無しさん:2014/06/22(日) 12:32:50.83 ID:OAMeVQFvC
全世界の全ての言語に対応した動画共有サイトを作りたいのですが、
例えば、インドネシア語版のサイトだと、当然、インドネシア語での問い合わせがあるわけじゃないですか。
同様に、アイスランド語版のサイトでは、アイスランド語での問い合わせがあるということですよね?
てことは、今の例で考えると、インドネシア人やアイスランド人を雇わないといけないということなんでしょうか?
それとも、何か別の方法があるのでしょうか?
そもそも、地球上には全てでいくつの言語があるのでしょうか?

教えてください。

113 :ご冗談でしょう?名無しさん:2014/06/22(日) 14:56:13.13 ID:???.net
宇宙飛行士とYouTubeの創業者はどっちの方がエリートですか?

114 :ご冗談でしょう?名無しさん:2014/06/22(日) 15:53:57.26 ID:???.net
全世界の全ての言語に対応した動画共有サイトを作りたいのですが、
例えば、インドネシア語版のサイトだと、当然、インドネシア語での問い合わせがあるわけじゃないですか。
同様に、アイスランド語版のサイトでは、アイスランド語での問い合わせがあるということですよね?
てことは、今の例で考えると、インドネシア人やアイスランド人を雇わないといけないということなんでしょうか?
それとも、何か別の方法があるのでしょうか?
そもそも、地球上には全てでいくつの言語があるのでしょうか?

教えてください。

115 :ご冗談でしょう?名無しさん:2014/06/22(日) 16:08:05.70 ID:???.net
物理関係ねえ〜

116 :ご冗談でしょう?名無しさん:2014/06/22(日) 16:11:16.07 ID:???.net
>>115
触るなよ

117 :ご冗談でしょう?名無しさん:2014/06/22(日) 16:13:09.81 ID:???.net
誰か真面目に>>114の質問に答えてください。お願いします。本当に作りたいと思っていますので。

118 :ご冗談でしょう?名無しさん:2014/06/22(日) 16:18:16.90 ID:???.net
>>116
二つもレスがつくから図に乗ったぞどうしてくれる

119 :ご冗談でしょう?名無しさん:2014/06/22(日) 16:31:00.27 ID:???.net
統計力学や解析力学やってないのに量子力学に背伸びするのは無理があると思う?

120 :ご冗談でしょう?名無しさん:2014/06/22(日) 16:37:46.68 ID:???.net
>>119
別にいいんじゃない?
わからないところがあったらそこを勉強すればいいんだから。

121 :ご冗談でしょう?名無しさん:2014/06/22(日) 16:40:30.82 ID:???.net
>>114
無と無限大
ttp://katahiromz.bbs.fc2.com/

122 :ご冗談でしょう?名無しさん:2014/06/22(日) 16:44:47.00 ID:???.net
>>121
真面目に教えてください。お願いします。

123 :ご冗談でしょう?名無しさん:2014/06/22(日) 16:52:15.89 ID:???.net
>>122
巣で聞きましょう、キャハ
http://maguro.2ch.net/test/read.cgi/budou/1329235888/

124 :ご冗談でしょう?名無しさん:2014/06/22(日) 16:57:24.31 ID:???.net
>>123
俺はキャハの文章をコピペしまくっただけであって、キャハではない。まったくの別人。勘違いしないように。

125 :ご冗談でしょう?名無しさん:2014/06/22(日) 17:02:59.81 ID:???.net
>>124
巣で聞きましょう
http://peace.2ch.net/test/read.cgi/gender/1393296640/

126 :ご冗談でしょう?名無しさん:2014/06/23(月) 00:04:15.18 ID:???.net
ヘリシティの値が0の光子って存在するんですか?

127 :ご冗談でしょう?名無しさん:2014/06/23(月) 03:58:15.78 ID:m0AL9Y8p.net
        ||.ギ━∧━∧━ ン!!!! 
        ||  ,r((゚)Å(゚))  
 ____||  ツィー=ニ彡つc□

128 :ご冗談でしょう?名無しさん:2014/06/23(月) 04:03:14.37 ID:m0AL9Y8p.net
        ||.ギ━∧━∧━ ン!!!! 
        ||  ,r((゚)Å(゚))  
 ____||  ツィー=ニ彡つc□

129 :ご冗談でしょう?名無しさん:2014/06/23(月) 18:59:31.67 ID:U1GR1yIj.net
>>102
じゃあコンデンサーにつながれているときは?

130 :ご冗談でしょう?名無しさん:2014/06/23(月) 20:57:56.46 ID:???.net
>>129
当然、コンデンサを充電するだけの電荷が流れるから、
それによる内部状態の変化がある。
充電し終わった時の起電力がずっと保たれる。

131 :ご冗談でしょう?名無しさん:2014/06/23(月) 21:31:53.23 ID:Y2c2y5Ky.net
東京電機大学中学校評判万引少年ザキシマ君
http://i.imgur.com/bpuheFv.jpg
稲城サッカースポーツ少年団評判稲城SSS評判TDU万引ザキシマ
http://i.imgur.com/bpuheFv.jpg
稲城市立向陽台小学校評判Y子がJSフェラ千円TDUザキシマ妹
http://i.imgur.com/bpuheFv.jpg

132 :ご冗談でしょう?名無しさん:2014/06/23(月) 22:32:19.01 ID:???.net
>>130みて、じゃあじゃねーよと思ったのはおれだけ?

133 :ご冗談でしょう?名無しさん:2014/06/23(月) 22:33:03.35 ID:???.net
ミスった。>>129

134 :ご冗談でしょう?名無しさん:2014/06/23(月) 22:38:10.85 ID:???.net
というか回路理論の話をしてるのか熱力学の話をしてるのかはっきりして欲しい。

135 :ご冗談でしょう?名無しさん:2014/06/23(月) 22:38:59.66 ID:???.net
物性の話してね

136 :ご冗談でしょう?名無しさん:2014/06/23(月) 22:39:54.58 ID:???.net
化学の話とも言える

137 :ご冗談でしょう?名無しさん:2014/06/23(月) 22:40:51.00 ID:???.net
電磁気学の話では

138 :ご冗談でしょう?名無しさん:2014/06/23(月) 22:44:08.66 ID:???.net
せめて電気化学と言ってくれ

139 :ご冗談でしょう?名無しさん:2014/06/23(月) 23:00:42.20 ID:???.net
デンカ

140 :ご冗談でしょう?名無しさん:2014/06/24(火) 09:09:10.44 ID:???.net
他のスレに書き込んだら物理板で聞いてみたらと言われたのでここで聞いてみます。

シリコンの真性キャリア密度を以下の値から計算して求めたいのです。

k=1.38*10^(-23)
T=300
Eg=1.12 * 1.60*10^(-19)
Nc=2.8*10^19
Nv=1.02*10^19

ni=√(NcNv) * exp(-Eg/2kT)

niの答えが1.5*10^10になるみたいなんですが、何度やってもni=6739932263≒6.7*10^9になります
与えられた数値がおかしいのかなと思ったのですが、どのテキストにもこれらの値と式が書かれています。
何がおかしいのでしょうか?

141 :ご冗談でしょう?名無しさん:2014/06/24(火) 09:30:39.99 ID:???.net
>>140
ググれば知恵遅れが教えてくれるよ

142 :ご冗談でしょう?名無しさん:2014/06/24(火) 09:34:28.57 ID:???.net
どうでもいいけど単位書け

143 :ご冗談でしょう?名無しさん:2014/06/24(火) 09:48:03.54 ID:???.net
>>140
5年前の質問だな
ttp://okwave.jp/qa/q5223297.html

144 :ご冗談でしょう?名無しさん:2014/06/24(火) 09:57:46.49 ID:???.net
>>140
そのniの式が厳密に成り立つのは、伝導帯と価電子帯が幅を持たないときだけ。
本当は幅があって、温度が上がるほど使える状態は増えていく。
だから、有効状態密度という温度依存の値を考えてそのniに代入しなければ、
有限温度のキャリア濃度を正しく求めることはできない。

145 :ご冗談でしょう?名無しさん:2014/06/24(火) 10:09:38.47 ID:???.net
>>142

k=1.38*10^(-23) [J/K]
T=300 [K]
Eg=1.12 * 1.60*10^(-19) [J]
Nc=2.8*10^19 [cm^(-3)]
Nv=1.02*10^19 [cm^(-3)]

ni=√(NcNv) * exp(-Eg/2kT) [cm^(-3)]

です。

>>143
この回答者のとおりに計算すると、 1.5*10^10程度の値になると言っていますが、ni=1.12*10^10となります。
これは真性キャリア密度が求まったとは言いづらい気がします。
質問者のお礼も、すっきり解決したという感じがしません

146 :ご冗談でしょう?名無しさん:2014/06/24(火) 10:19:56.02 ID:???.net
>>144
Nc,Nvは室温(300K)における有効状態密度みたいです。
書き方が悪くて申し訳ないです。
だとしたら真性キャリア密度は正しく求まるはずですよね...

147 :ご冗談でしょう?名無しさん:2014/06/24(火) 15:02:55.24 ID:???.net
>>145
五年間も悩んでたんですか?

148 :ご冗談でしょう?名無しさん:2014/06/24(火) 16:42:26.01 ID:???.net
コンピュータに詳しくなりたかったら、高度情報処理技術者の資格を獲るべきなんでしょうか?

149 :ご冗談でしょう?名無しさん:2014/06/24(火) 18:16:30.94 ID:???.net
至高の拳法は少林寺

150 :記憶喪失した男:2014/06/24(火) 18:45:28.00 ID:HLcECneM.net
おれは考えたよ。

「真面目について」のつづきなんだけど。
宇宙というのは死の土地だ。
地球に生物が存在するのはとても不自然な異常現象であり、
それゆえ、人は放っておくと死にたくなってしまう。

また、学問というものは宇宙全体について解析したものであるため、
総合的に考えれば、必ず生きることとはかけ離れた答えにたどりついてしまう。
つまり、真面目にやると、死ぬべき世界について研究することになり、
ワルがモテるというのは、不真面目に都合のよい異常現象についてだけ取り出して考えるからではないだろうか?

物理現象を計算機で総当たりに調べて出てくる物理方程式は、死の世界について記述した答えではないだろうか?

151 :記憶喪失した男:2014/06/24(火) 19:30:49.47 ID:HLcECneM.net
究極の物理学というものがあったら、それは「真面目」なものになるか、「不真面目」なものになるか、
あるいは、「命を肯定する」ものになるか、「死を肯定する」ものになるか、教えてほしいのですよ。

152 :ご冗談でしょう?名無しさん:2014/06/24(火) 21:03:20.90 ID:???.net
究極の野菜は野沢菜

153 :ご冗談でしょう?名無しさん:2014/06/24(火) 21:05:50.62 ID:???.net
>>151
あんたのいうようなことを考えた人は大勢いるんだよ
だから一回図書館でよく調べてみな

154 :記憶喪失した男:2014/06/24(火) 21:09:22.80 ID:HLcECneM.net
>>153
ええ? 初めて知りました。
ぜひ、書名などを教えいただけないでしょうか?

ちなみに、わたしは仏教の一切皆苦から発展してここに至りました。

155 :ご冗談でしょう?名無しさん:2014/06/24(火) 21:10:45.08 ID:???.net
記憶また物理板荒らしにきたのかよ

統合失調症の薬飲んで寝ろ

156 :ご冗談でしょう?名無しさん:2014/06/24(火) 21:13:11.47 ID:???.net
>>154
その場合まず仏典に関して徹底して勉強すべき
そうすれば自然といろいろな研究が出てくる
スレチなのでこれだけな

157 :ご冗談でしょう?名無しさん:2014/06/24(火) 21:16:22.70 ID:???.net
記憶喪失した男の妄言録
http://anago.2ch.net/test/read.cgi/tubo/1373374628/

158 :ご冗談でしょう?名無しさん:2014/06/25(水) 01:07:00.15 ID:WqguR1sqn
ブラックホールは「ホーキング輻射」によって長い時間をかけて蒸発するらしいですが、
中性子星やクォーク星は永久に安定なんでしょうか?

159 :ご冗談でしょう?名無しさん:2014/06/25(水) 12:59:23.03 ID:???.net
あのあの

考えていた事が 一瞬後に失われる事ありますよね

何しようとしていたのか忘れてしまう事が 一瞬前まで頭の中にあったのに

あれって不確定性原理によって 電気パルスの経路がかわったととか

そういう不確定性原理的な事が関係していますか?

160 :ご冗談でしょう?名無しさん:2014/06/25(水) 13:01:22.88 ID:???.net
脳科学者にでも聞け

161 :ご冗談でしょう?名無しさん:2014/06/25(水) 13:16:32.35 ID:???.net
どこの板ですか

162 :ご冗談でしょう?名無しさん:2014/06/25(水) 13:29:30.37 ID:???.net
@kenichiromogi

163 :ご冗談でしょう?名無しさん:2014/06/25(水) 13:32:19.76 ID:???.net
わからなくても考えることが 脳にいいんです

164 :ご冗談でしょう?名無しさん:2014/06/25(水) 15:18:21.79 ID:FhCWRBY5.net
少し前にdv/dt+kv=αのグリーン関数を利用した解法をたずねたのですが
まだ理解できないのでどなたか助力してくださいm(_)m
作用素d/dt+kの逆作用素とは実際にどんな数式ですか?
その逆作用素とαを部分積分すれば解がでるのですか?
そしてそれは特解で同次方程式の解との和が一般解になるということで
いいでしょうか?一般解をご提示ください。どうぞよろしくお願いいたしますm(_)m

165 :ご冗談でしょう?名無しさん:2014/06/25(水) 15:32:58.40 ID:???.net
第二次世界大戦の時

ガンダムが何機あれば

日本はアメリカに勝てましたか?

166 :ご冗談でしょう?名無しさん:2014/06/25(水) 15:34:15.16 ID:FhCWRBY5.net
私が逆作用素と書いたのはd/dt+kのインバース(-1)=グリーン関数のことで
これだと用語使い方がおかしいのかな。これと積分すると逆作用素ってことかな。
なにぶん物理や数学を履修したものではない初学者なのでどうもすいません。

167 :ご冗談でしょう?名無しさん:2014/06/25(水) 15:52:28.92 ID:???.net
>>164
http://www.amazon.co.jp/dp/4004160057

168 :ご冗談でしょう?名無しさん:2014/06/25(水) 16:02:00.96 ID:???.net
http://imgur.com/IQwxzw1.jpg
この回路の変圧器T1,T2,T3の動作の仕方がわかりません
例えばT1の二次側の左上から右に向けて電流が流れる時、
T2とT3ではどの方向に電流が流れるのでしょうか

169 :ご冗談でしょう?名無しさん:2014/06/25(水) 16:21:40.25 ID:???.net
>>166
逆作用素の積分核K(s,t)による表現
{(d/dt+k)^(-1)f}(t)=∫K(t,s)f(s)ds

物理だとグリーン関数K(t,s)の定義は
(d/dt+k)K(s,t)= δ(s-t)

後は勉強してね

170 :ご冗談でしょう?名無しさん:2014/06/25(水) 16:56:30.83 ID:???.net
コンピュータに詳しくなりたいのですが、まずは何から勉強すれば良いのでしょうか?

171 :ご冗談でしょう?名無しさん:2014/06/25(水) 17:35:31.30 ID:???.net
統計力学を習い始めたんですが
同種粒子を区別しない場合の状態の数が
Ω(E) = (相空間の体積V) / h^3N × N!

とあります。N!は同種粒子を入れ替える回数みたいなんですが、なぜこれはN!なのでしょうか?

許されるエネルギー状態の数をM、粒子数をNとすれば、単純に重複組合せの考え方より
Ω(E) = N+M-1_C_M-1 = (N+M-1)! / (M-1)!
になると思うんですがどうなのでしょうか?

172 :ご冗談でしょう?名無しさん:2014/06/25(水) 18:48:32.18 ID:FhCWRBY5.net
どなたかd/dt+kの逆作用素の積分核=グリーン関数?
というものを具体的に文字式で書いて
その積分核と非同次項との積分結果を書いていただけないでしょうか。
どうぞよろしくお願いいたします。
微分方程式を解くということは関数の形で独立変数に数値を入れれば任意の従属変数の値が
わかるということですよね、グラフも書けると。何冊か本を買ったんですが
よくわからないんです。

173 :ご冗談でしょう?名無しさん:2014/06/25(水) 19:25:39.45 ID:???.net
>>172
アドバイス:お前には無理

174 :ご冗談でしょう?名無しさん:2014/06/25(水) 19:38:18.99 ID:???.net
グリーン関数は正直面倒なだけのテンソル計算と違って物理学徒をガチで殺しにくるからな

175 :ご冗談でしょう?名無しさん:2014/06/25(水) 21:52:00.50 ID:???.net
>>168
AC回路で電流の向き聞いちゃう?

176 :ご冗談でしょう?名無しさん:2014/06/25(水) 23:49:32.25 ID:???.net
>>173
わからないやつからアドバイスされてもな...

177 :ご冗談でしょう?名無しさん:2014/06/26(木) 00:26:27.17 ID:???.net
>>171
古典的には区別可能だった粒子に対して 1/N! の因子を掛けることで不可分別性を取り入れる
重複を許してそれらの組み合わせを考えるのとは違う
どうでもいいが {}_{N+M-1}C_{M-1} = (N+M-1)!/((M-1)!N!) であって = (N+M-1)!/(M-1)! ではない

178 :ご冗談でしょう?名無しさん:2014/06/26(木) 01:37:20.71 ID:???.net
>>177
その因子が1/N! であることはどうやって導出されるんでしょうか?

あと少し調べたらこの因子は高温の場合にしか成り立たないので、ボースアインシュタイン統計が正確みたいなことが分かりました

179 :ご冗談でしょう?名無しさん:2014/06/26(木) 01:45:31.16 ID:???.net
古典統計なんだからボルツマン分布になるのは当然のこと。
あと二項係数は \binom#1#2 の方が見やすいと思う。

180 :ご冗談でしょう?名無しさん:2014/06/26(木) 02:05:22.96 ID:???.net
>>179
勉強不足で何を仰ってるのか分からないです。もう少し自分で考えてみます
ともあれレスありがとうございました。

181 :ご冗談でしょう?名無しさん:2014/06/26(木) 02:50:32.49 ID:???.net
>>178
高校の場合の数からやりなおせ

182 :ご冗談でしょう?名無しさん:2014/06/26(木) 03:13:34.26 ID:Kl437J4Z.net
20〜40キロの加速につかう燃料は
0〜20キロの加速に使う燃料の3倍

座標の原点を20キロにとれば
燃料の消費は同じな気がします

摩擦や抵抗が無いロケットのようなものだと矛盾が生じませんか?

183 :ご冗談でしょう?名無しさん:2014/06/26(木) 06:57:58.48 ID:???.net
>>182
いや、使う燃料はそういう式じゃない。
実際に成り立つ式は
△V =w logδ
(wは噴射速度、δは質量比)
だからな。
△V(得られる速度)を倍にするにはδを自乗する。
状況により、4倍より大きいことも小さいこともある。

184 :ご冗談でしょう?名無しさん:2014/06/26(木) 07:02:44.58 ID:???.net
>>176
君には無理な理由

1.勉強してない
2.勉強するする気がない
3.人の言うことを聞かない
4.問題の丸投げ

185 :ご冗談でしょう?名無しさん:2014/06/26(木) 07:27:27.33 ID:???.net
煽っても教えない

186 :ご冗談でしょう?名無しさん:2014/06/26(木) 08:50:15.24 ID:oFkR/hY3.net
>181
私は質問主169(仮にA)ですけど私に無理と言っている170(B)ではないし
(B)を非難している173(C)でもありません。
勉強していないように感じられるのは私の能力不足なのでしょうが
もう2年位前からグリーン関数でつまっていてグリーン関数の本は2冊買ってますし
加えて微分方程式本は5冊は買ってます。私は物理を履修したことのない中年で
どうにかこれを理解したいんです。どなたか教えていただけないでしょうか。

187 :ご冗談でしょう?名無しさん:2014/06/26(木) 08:57:37.94 ID:???.net
>>186
本の名前は何?

188 :ご冗談でしょう?名無しさん:2014/06/26(木) 09:09:13.34 ID:oFkR/hY3.net
>184
「物理/工学のためのグリーン関数入門」と「現代工学のための偏微分方程式とグリーン関数」です。
Amazonで検索して出てきたので買いました。

数時間、返信できないと思います。

189 :ご冗談でしょう?名無しさん:2014/06/26(木) 09:21:32.72 ID:???.net
>>188
「物理/工学のためのグリーン関数入門」は読んでも意味なさそう。
基本となる微積分、フーリエ変換、関数論は勉強したの?

190 :ご冗談でしょう?名無しさん:2014/06/26(木) 09:31:07.70 ID:9IsQ5BOv.net
◎2chスレッド勢いランキングサイトリスト◎

★+ニュース板
・ 2NN (推奨サイト)
・ 2chTimes
★+ニュース板新着
・ 2NN新着
・ Headline BBY
・ unker Headline
★+ニュース板その他
・ Desktop2ch
・ 記者別一覧
★全板
・ 全板縦断勢いランキング (推奨サイト)
・ スレッドランキング総合ランキング
・ ログ速
★全板実況込み
・ 2勢 (推奨サイト)
・ READ2CH
・ i-ikioi

※ 要タイトル検索
※ 2chブラウザ併用推奨

191 :ご冗談でしょう?名無しさん:2014/06/26(木) 13:00:07.22 ID:oFkR/hY3.net
>186
微分方程式とフーリエ変換の本は何冊か買いましたし
練習問題も多少はやりました。
で、私の知りたいのは非同次微分方程式における
グリーン関数を使った解法と積分記号を使った一般的な概念式ではなく具体的な
関数で書かれた(sinだとかx^2だとか)結果の式なので、
そちらの方をよろしくご教授ください。

192 :ご冗談でしょう?名無しさん:2014/06/26(木) 13:12:33.58 ID:???.net
>>191
全然分かってないように見えるし、何回質問してもレスに進歩がない。
そんなに答えが知りたければ金で解決しなさい。

193 :ご冗談でしょう?名無しさん:2014/06/26(木) 13:56:32.85 ID:???.net
>>191
>>167
>d/dt+kの逆作用
がどういうものか詳しく明示されてるから読んだら

194 :ご冗談でしょう?名無しさん:2014/06/26(木) 14:30:00.72 ID:???.net
思ったのですが、多言語サイトを作る場合、コンテンツも、その国で浸透しているジャンルのものを作らないといけないということですよね?
例えば、インドネシア語版のサイトだと、当然、インドネシアで浸透しているもののコンテンツを作らないといけないわけですよね?
そうなってくると、その言語が話されている国の文化や歴史などを全て把握しなければならないと思うのですが、
世界約200カ国の国々の全ての文化や歴史などを知ることは不可能だと思います。
そういう場合、どうしたらいいのでしょうか?
例えば、巨大掲示板を作ったとします。で、そのサイトをそのまま多言語展開しても、
例えば、スポーツだと、野球なんかはかなりマイナーなスポーツなので、
日本語版のサイトなら、野球板があっても普通に人が入ってくると思いますが、
インドネシア語版のサイトに、野球板を設けても無意味だと思うのです。
そんな感じで、様々な言語版のサイトに、どんな板を設ければ良いのかを知るためには、
その言語が話されている国の文化や歴史などを全て知らないと駄目だと思うのですが、
それは不可能だと思うのです。
こういう場合、どうすれば良いのでしょうか?
個人的に考えている方法としては、少々大雑把になってしまうのですが、
国際的に見ても通用する板を設けるという方法です。
例えば、スポーツ関係なら、サッカー、野球、バスケット・・・みたいに分けないで、
スポーツ板という形でまとめてしまうのです。
同様に、食べ物関係なら、ラーメン、ハンバーガー、ポテト・・・みたいに分けないで、
食べ物板という形でまとめてしまう感じです。
そうすれば、少々大雑把にはなってしまうものの、世界的に見ても通用すると思うのです。

やっぱり、多言語展開する場合、この方法しか良さそうな方法はないでしょうか?
他に良さそうな方法があったら教えてください。

195 :ご冗談でしょう?名無しさん:2014/06/26(木) 14:34:00.09 ID:???.net
少林寺拳法は至高です

196 :ご冗談でしょう?名無しさん:2014/06/26(木) 15:24:04.22 ID:???.net
メコスジ道は恥垢です

197 :ご冗談でしょう?名無しさん:2014/06/26(木) 20:56:52.23 ID:oZ9Wps6FI
質問なのですが、
ブラックホールからある程度離れた場所に観測者がいるとして、
ブラックホールの真後ろに天体が無い場合@と、ある場合A、
観測者の感じる重力は違いますか?

こんなイメージ↓
@
[自分]   [BH]

A
[自分]   [BH] [星]

198 :ご冗談でしょう?名無しさん:2014/06/27(金) 00:34:13.03 ID:OeZX6p5g.net
これっていつ、なぜ止まるんですか?
http://www.youtube.com/watch?v=RkLfpXpO5sQ
それとも完璧な永久機関なんじゃないですか?

199 :ご冗談でしょう?名無しさん:2014/06/27(金) 00:40:25.68 ID:???.net
そのうち摩擦で止まる。
そもそも、あの機構で勝手に回り出すことはない。

200 :ご冗談でしょう?名無しさん:2014/06/27(金) 00:43:30.96 ID:OeZX6p5g.net
永久機関ってたいてい摩擦で止まるとは言え
かなり長く回ってるものが多い気がするけど
実用性はないんですか?

201 :ご冗談でしょう?名無しさん:2014/06/27(金) 00:45:06.23 ID:???.net
>>200
永遠に回ってるわっかを見ててなんか楽しいの?
実用的なの?

202 :ご冗談でしょう?名無しさん:2014/06/27(金) 00:46:01.40 ID:???.net
摩擦を減らすことに努力を重ねた方が意味があるからでしょうね
あとは、コスト対効果
結局その手の自称永久機関は慣性の法則通りに近づける工夫
というだけでしょうから…。

203 :ご冗談でしょう?名無しさん:2014/06/27(金) 00:46:49.97 ID:???.net
第一種永久機関と第二種永久機関の定義調べてこいよ

204 :ご冗談でしょう?名無しさん:2014/06/27(金) 00:47:51.45 ID:???.net
仕事が取り出せない永久機関っておいおい

205 :ご冗談でしょう?名無しさん:2014/06/27(金) 00:50:00.07 ID:???.net
>>203
誰に言ってんだよカス(死神くんの監視官風)

206 :ご冗談でしょう?名無しさん:2014/06/27(金) 00:51:59.89 ID:???.net
>>204
ほんとそう
wikipediaにも偉そうに第二種永久機関とか書いてあるけど
実際外部に仕事を出来ない永久機関なんて実用性ゼロだよね
周囲に影響を与えない=周りに損も得も与えない=なくてもいい=存在価値なし

207 :ご冗談でしょう?名無しさん:2014/06/27(金) 00:54:12.88 ID:OeZX6p5g.net
永久機関を応用した軸受けとか作れないの?

208 :ご冗談でしょう?名無しさん:2014/06/27(金) 00:55:21.68 ID:OeZX6p5g.net
別にエネルギーを取り出すだけが工学じゃないと思うんだけど
頭の固い物理スレで言うのもなんだけどさ

209 :ご冗談でしょう?名無しさん:2014/06/27(金) 00:58:14.41 ID:???.net
>>207
何度も言うけど費用対効果
ボールベアリングや流体軸受けと同程度のコストで
かつ損失が低いシンプルな機関が作れればまだ
「一般的な」実用性はあるかもね

あるいは、超ニッチで趣味的要素も含むアカデミックな分野で
いくらおカネかかってもいいからという分野であれば、もしかしたら
実用性を発揮できるものが存在する可能性はかろうじてあるかもしれない

210 :ご冗談でしょう?名無しさん:2014/06/27(金) 00:59:27.06 ID:???.net
>>208
工学とは、人の役に立ってこそ意味があるのです
それは、量産性という側面も重要になってきます

物理学などの理学は、そういった面は気にしなくてもよいです

211 :ご冗談でしょう?名無しさん:2014/06/27(金) 01:01:43.03 ID:???.net
>>205
バカに言ってんだよ?

212 :ご冗談でしょう?名無しさん:2014/06/27(金) 01:03:49.52 ID:???.net
>>211
物理板なんだから、どの第三者から見ても明らかなように安価ふれよカス

213 :ご冗談でしょう?名無しさん:2014/06/27(金) 01:04:50.40 ID:???.net
>>207
逆だよ。真面目に軸受け作ってる人が作った軸受けを
永久機関と称するオモチャを作る輩が利用してるの。
そもそも永久機関を称してるのに
摩擦を押さえたことをウリにしてたら噴飯ものだからね。

214 :ご冗談でしょう?名無しさん:2014/06/27(金) 01:05:26.51 ID:???.net
>>212
え?自分がバカな自覚があるの?

215 :ご冗談でしょう?名無しさん:2014/06/27(金) 01:08:07.89 ID:???.net
>>213
全くだ
エネルギー保存則により、エネルギーを生み出す第一種永久機関が
存在し得ない以上、現存する流体軸受けやボールベアリングを使って
第二種永久機関をめざすその様は、実に笑える
流体軸受け以上によくなることは見込めないことがやる前から自明

216 :ご冗談でしょう?名無しさん:2014/06/27(金) 01:09:13.38 ID:???.net
>>214
いいから>>203はどのレス宛のレスだったのかを書けっつってんだよカス

217 :ご冗談でしょう?名無しさん:2014/06/27(金) 01:12:16.50 ID:???.net
>>216
196宛だったけど定義を知らないで議論しようとする
バカ全員が調べるべきだと思うよ?

218 :ご冗談でしょう?名無しさん:2014/06/27(金) 01:13:27.56 ID:???.net
>>217
了解した

219 :ご冗談でしょう?名無しさん:2014/06/27(金) 01:13:43.88 ID:???.net
馬鹿共が必死に永久機関を作ろうとして悉く失敗してること自体エネルギー保存則を裏付ける結果となってる

220 :ご冗談でしょう?名無しさん:2014/06/27(金) 01:15:27.35 ID:???.net
>>219
そういう意味では、エネルギー保存則の反証可能性に基づいた反証を
積み重ねて確からしさを高めているという意味において物理学に貢献
してるからまあいいのかなw

221 :ご冗談でしょう?名無しさん:2014/06/27(金) 01:16:48.70 ID:???.net
ちょっと語弊があったか
反証可能性があるエネルギー保存則において反証仮説をたくさん作って
ことごとく失敗することによってエネルギー保存則の確からしさを高めている

222 :ご冗談でしょう?名無しさん:2014/06/27(金) 02:16:38.31 ID:???.net
永久機関を作ろうと同じ失敗を繰り返す彼らの姿はまるで永久機関のようだ

223 :ご冗談でしょう?名無しさん:2014/06/27(金) 07:34:50.12 ID:???.net
>>222
誰がうまいこと言えと

224 :ご冗談でしょう?名無しさん:2014/06/27(金) 10:34:19.64 ID:???.net
場の量子論によれば真空のエネルギーは無限大となる。
つまり真空ポテンシャルの差を作り出せばエネルギーが取り出せる。
カシミール効果等による巨大装置を作れば真空エネルギー機関が可能。

225 :ご冗談でしょう?名無しさん:2014/06/27(金) 10:36:17.99 ID:???.net
メルヘンですのん

226 :ご冗談でしょう?名無しさん:2014/06/27(金) 12:35:48.83 ID:???.net
質問します。
無重力の空間で、無限に近い棒を、頭の上にてグルッと振り回した場合
棒は、渦を巻くようにぐるぐる形になるんでしょうか?

227 :ご冗談でしょう?名無しさん:2014/06/27(金) 12:36:41.54 ID:???.net
>>226訂正
× 無限に近い棒
○ 無限に近い長さの棒を

228 :ご冗談でしょう?名無しさん:2014/06/27(金) 13:05:02.41 ID:???.net
>>226
まあなるでしょうね

229 :ご冗談でしょう?名無しさん:2014/06/27(金) 13:38:22.33 ID:sni4C3+m.net
>>167
>>193
近所の図書館にあるようなので借りてきます。
どうもありがとうございます。

230 :ご冗談でしょう?名無しさん:2014/06/27(金) 16:17:55.64 ID:???.net
言語は全部でいくつあるの?

231 :ご冗談でしょう?名無しさん:2014/06/27(金) 16:25:33.58 ID:???.net
野沢菜は至高の漬物です

232 :ご冗談でしょう?名無しさん:2014/06/27(金) 18:46:47.39 ID:OeZX6p5g.net
重力によって空間が曲がっているということは
絶対に曲がらない物差しを水平線に伸ばしていくと曲がるということですか?

233 :ご冗談でしょう?名無しさん:2014/06/27(金) 18:52:02.92 ID:???.net
「絶対に曲がらない」と定義したんなら曲がらんだろ。

234 :ご冗談でしょう?名無しさん:2014/06/27(金) 20:02:48.10 ID:???.net
>>232
強い重力圏の近くにその物差しを置くと、「U」みたいな状況になるかもね。

235 :ご冗談でしょう?名無しさん:2014/06/27(金) 20:53:34.25 ID:???.net
デイトレーダーとして食っていける割合ってどのくらいなんでしょうか?

236 :ご冗談でしょう?名無しさん:2014/06/27(金) 20:59:05.47 ID:???.net
偏差値40では無理です 野沢菜うめー

237 :ご冗談でしょう?名無しさん:2014/06/27(金) 21:31:31.59 ID:???.net
>>222
仕事してないけどな

238 :ご冗談でしょう?名無しさん:2014/06/27(金) 21:43:13.03 ID:???.net
>>232
時空が曲がるというのは、測地線が曲がるということ。
最短距離を走るためには回り道をする必要があるということ。

239 :ご冗談でしょう?名無しさん:2014/06/27(金) 22:56:55.47 ID:TaZA1wtn.net
>>226
剛体なら回せないけどね

240 :ご冗談でしょう?名無しさん:2014/06/28(土) 07:45:37.57 ID:???.net
>>239
>剛体の場合には回せない

剛体の棒の長さが5m、その棒をぐるっと回したときには手元の速度より、棒先の速度の方が早いですよね?
しかも、5m程度の長さならぐるっと回せると思います。
では、剛体の棒の長さがどれくらいになると、ぐるっと回すことができなくなりますか?

無重力の空間であり、質量が0にきわめて近い、長さ無限の剛体の棒があるとして

241 :ご冗談でしょう?名無しさん:2014/06/28(土) 09:30:30.08 ID:???.net
どうでもいいけどポエムが好きだね

242 :ご冗談でしょう?名無しさん:2014/06/28(土) 10:46:33.29 ID:???.net
儲かりやすい銘柄は何ですか?

243 :ご冗談でしょう?名無しさん:2014/06/28(土) 11:04:32.95 ID:???.net
長野県の野沢菜の漬物です。

244 :ご冗談でしょう?名無しさん:2014/06/28(土) 11:59:57.46 ID:???.net
株で数千億円稼ぐのはどれくらい難しいですか?

245 :ご冗談でしょう?名無しさん:2014/06/28(土) 12:42:37.08 ID:???.net
>>240
そりゃあ先端が光速に達するから回せなくなるんでしょうよ

246 :ご冗談でしょう?名無しさん:2014/06/28(土) 12:45:06.18 ID:???.net
回せないとか回せるとか、剛体が存在することを前提にするのがまず間違いだよね。
どの長さから剛体近似が成り立たなくなるかを考えるべきでしょ

247 :ご冗談でしょう?名無しさん:2014/06/28(土) 13:09:11.13 ID:???.net
観測点によっても違いますよね

始点となる回す人を観測点としたケース
回転面から垂直方面に十分離れて観測点を設けるケース
棒先を観測点としたケース

248 :ご冗談でしょう?名無しさん:2014/06/28(土) 13:52:23.18 ID:???.net
「完全に固い」ものは作れないんじゃないかなあ
慣性系に棒があるとして、ある地点に力を加えて位置が変わると、
(分子がどういう構造で結合しているとしても)
分子間では電子が介在して近隣の分子の位置も変化していくと思う。
つまり電磁気力によって位置の変化が有限のスピードで伝わっていくイメージ?

249 :ご冗談でしょう?名無しさん:2014/06/28(土) 15:03:58.81 ID:???.net
そうすると、素粒子以外の物質は、光速に近づくにつれ、全て形状が変化するってことですかね?
確率的に存在する、という考え方は、光速や他の力の影響で、存在確率は変化するってことですか?

250 :ご冗談でしょう?名無しさん:2014/06/28(土) 15:38:44.22 ID:???.net
244ですが>>249は自分に対する疑問?
ごめんいまいち文章がわからないかも
「光速に近づくにつれ、全て形状が変化」はどこから出てきた話?
慣性系なら光速に近くても当然形状は変化しないと思うけど、非慣性系のこと言ってる?
2行目の「確率的に存在する」は(主語がわからないけど)どういう関連が?

251 :ご冗談でしょう?名無しさん:2014/06/28(土) 15:48:36.65 ID:???.net
デイトレーダーはニートですか?

252 :ご冗談でしょう?名無しさん:2014/06/28(土) 16:07:29.50 ID:ROrImigP.net
ほっとこうぜそんなアホ
何も分かってないじゃないか

253 :ご冗談でしょう?名無しさん:2014/06/28(土) 17:02:38.87 ID:???.net
光速を超えた情報伝達を禁じると、剛体の存在も禁じられる。
ただそれだけのことなのに何でまたよくわからんイメージ論を持ち出すのか

254 :ご冗談でしょう?名無しさん:2014/06/28(土) 17:19:11.81 ID:???.net
偏差値40のおっさんに荒らされて物理板の住人はなにもしないのでしょうか?

255 :ご冗談でしょう?名無しさん:2014/06/28(土) 17:22:44.89 ID:edgu1V58.net
剛体の存在が禁じられる理由を定性的に述べただけだろ

256 :ご冗談でしょう?名無しさん:2014/06/28(土) 18:31:49.68 ID:???.net
定性的にもクソも剛体の定義が光速を超える情報伝達を前提とするしなあ

257 :ご冗談でしょう?名無しさん:2014/06/28(土) 19:10:52.41 ID:???.net
31万kmちょい先にあるスイッチを長さ31万kmの棒で押すとか?

258 :ご冗談でしょう?名無しさん:2014/06/28(土) 19:17:29.76 ID:???.net
剛体であっても、外から見たら曲がって見えて、棒先からみたら真っ直ぐなんじゃない?

259 :ご冗談でしょう?名無しさん:2014/06/28(土) 19:57:36.04 ID:???.net
なら相対論では剛体は扱えない?

260 :ご冗談でしょう?名無しさん:2014/06/28(土) 20:16:27.19 ID:???.net
ローレンツ収縮しない理想的剛体を考える意味はあるのかね?

261 :ご冗談でしょう?名無しさん:2014/06/28(土) 20:16:45.12 ID:???.net
>>257
棒の長さは幾らでもいいんだよ。1cmだっておなじ。反対側に動きが伝わるのが光速を越えないということ。

262 :ご冗談でしょう?名無しさん:2014/06/28(土) 20:25:00.10 ID:???.net
曲がったり縮むんじゃない
そう見えるってだけ
実際に曲がるわけじゃない

263 :ご冗談でしょう?名無しさん:2014/06/28(土) 20:33:58.44 ID:???.net
絶対ゼロ度ってあるじゃないですか
あれは圧力が0以下になることがないからって理論ですけど
宇宙が加速度的に膨張しているという理論からいえば
圧力が0以下になり、絶対ゼロ度以下の低温も存在することになりますか?
それを数学的に証明し、実証できたらノベルショ、受賞できますかいな?

264 :ご冗談でしょう?名無しさん:2014/06/28(土) 20:36:08.85 ID:???.net
絶対0度以下の低温が存在するとしたら
その時、物体の振る舞いはどうなるんですかね?

265 :ご冗談でしょう?名無しさん:2014/06/28(土) 21:27:43.09 ID:???.net
さー

266 :ご冗談でしょう?名無しさん:2014/06/28(土) 21:29:38.59 ID:???.net
>>264
絶対零度以下は「低温」ではなく「∞を越える程の高温」だ。

267 :ご冗談でしょう?名無しさん:2014/06/28(土) 21:40:56.53 ID:???.net
>>264
google : 反転分布

268 :ご冗談でしょう?名無しさん:2014/06/28(土) 21:59:52.50 ID:???.net
負温度、読みました。なるほど、∞高温なのですねー

宇宙が加速度的に膨張してるという話を聞いたので
そういう力があるのなら、圧力0以下で絶対零度以下の温度も存在するのかなぁと
絶対零度以下の温度を実証できたら圧力0以下の力も証明できて
宇宙の加速度的な膨張を証明することもできるかなぁと
すみません、素人の質問に答えてくださってありがとうでした

269 :ご冗談でしょう?名無しさん:2014/06/29(日) 00:23:02.43 ID:???.net
さも熱平衡状態の温度と地続きであるかのように統計力学的負温度を持ち出すのは
不親切というかいい加減というか

270 :ご冗談でしょう?名無しさん:2014/06/29(日) 01:06:09.41 ID:???.net
>>269
他にちゃんと地続きな負温度があるんならともかく、そうじゃないんだから統計力学を出したら不親切と言われても。

271 :ご冗談でしょう?名無しさん:2014/06/29(日) 01:08:18.23 ID:???.net
「絶対零度以下の低温」なるものを聞いているのに負温度を持ち出した結果
>>268を見る限り明らかに変な誤解をしたようだが

272 :ご冗談でしょう?名無しさん:2014/06/29(日) 01:11:23.54 ID:???.net
負温度って「温度」とは付いてるけど温度とは全く別物じゃね

273 :ご冗談でしょう?名無しさん:2014/06/29(日) 02:40:12.79 ID:???.net
圧力ゼロの力ってそもそも力としてなりたってないじゃないか

274 :ご冗談でしょう?名無しさん:2014/06/29(日) 02:41:25.48 ID:2aOCSc0r.net
>190
ありがとうございます。借りてきて読んでます。207-210ページのところ
d/dt+kの逆作用が「e^-kt∫e^kt()dt」ということで()内にgを入れると
Ce^-kt+g/kでdv/dt+kv=gの一般解が得られると。
とするとグリーン関数というのはどう書かれるのでしょうか?「」内?
本なんか見ると一階のがないので二階ですがd^2u/dt^2=f G(t、ξ)=t(ξ-l)/l ξ(t-l)/l
とか書かれてたりしますが...

275 :ご冗談でしょう?名無しさん:2014/06/29(日) 02:41:53.26 ID:???.net
しかも宇宙の膨張は理論的に証明はされてないが観測結果から膨張していることはわかってるぞ。

276 :ご冗談でしょう?名無しさん:2014/06/29(日) 02:59:24.35 ID:2aOCSc0r.net
270に追加ですが二次元ラプラス関数の主要解G =-1/2πlogrとか
それもわかっていないんですが雲をつかむと言いますか、今回ので雲じゃなくて
水をつかむぐらいのなったかどうか。

277 :ご冗談でしょう?名無しさん:2014/06/29(日) 03:17:45.42 ID:2aOCSc0r.net
>272
×-1/2πlogr
○-1/2π・logr

278 :ご冗談でしょう?名無しさん:2014/06/29(日) 06:37:31.32 ID:???.net
>>268
>>269-267が補足しているけれど、熱力学的には「「負温度は存在しない」。
統計力学的にも、統計分布のパラメーターを温度だと解釈すれば、負温度に当てはまる分布は考えることができる。
この場合の負温度の分布というのは非平衡系における分布なので、平衡系で普通に定義される温度とは全く性質が異なる。
注意しなくちゃいけないのは、もともとの温度とか圧力とか(エントロピーとか)というのは、
熱力学的な意味で平衡に達した系を特徴付ける量として定義されていて、非平衡系に対しては定義できない。

宇宙の膨張に関しては一般相対論が絡むけど、相対論的熱力学は勉強してないのでパス。
教科書は知らないけどいくつか論文があったはずなので arXiv とかで探してみよう。

279 :ご冗談でしょう?名無しさん:2014/06/29(日) 09:38:57.59 ID:???.net
>>276
こんにちはグリーン関数君、あいかわらず分かってないということはよく伝わってくる

280 :ご冗談でしょう?名無しさん:2014/06/29(日) 10:40:43.79 ID:???.net
>>278 >負温度の分布
レーザー発振の原理として実際使われてる。

281 :ご冗談でしょう?名無しさん:2014/06/29(日) 11:08:33.08 ID:???.net
要するに、あれですね、?273.15 ℃を絶対零度とすると
-300℃とか-500℃とかは存在しえない温度ってことなんですね?

282 :ご冗談でしょう?名無しさん:2014/06/29(日) 11:12:44.56 ID:???.net
で、圧力がマイナスになることもないってことですね

283 :ご冗談でしょう?名無しさん:2014/06/29(日) 11:36:25.23 ID:???.net
相対的にはあるんじゃね?
負圧ってあるし。

284 :ご冗談でしょう?名無しさん:2014/06/29(日) 12:05:36.77 ID:GE8DKevZ.net
  
【国内】セルジオ越後氏の甥、山口農産研究所でイチゴ新品種を開発し命名「セルジオいちご」
http://maguro.2ch.net/test/read.cgi/yasai/1332334159/
 

285 :ご冗談でしょう?名無しさん:2014/06/29(日) 13:48:01.25 ID:GE8DKevZ.net
  
【国内】セルジオ越後氏の甥、山口農産研究所でイチゴ新品種を開発し命名「セルジオいちご」
http://maguro.2ch.net/test/read.cgi/yasai/1332334159/
 

286 :ご冗談でしょう?名無しさん:2014/06/29(日) 15:40:52.52 ID:???.net
トリチェリの定理を示す問題で
大気に接している点でベルヌーイの定理のp/ρの項をp=大気圧としているんですが
下から受ける圧力は考えなくていいんですか?

287 :ご冗談でしょう?名無しさん:2014/06/29(日) 23:00:12.10 ID:???.net
空飛ぶ円盤実現へ
ttp://sankei.jp.msn.com/world/news/140629/amr14062916350004-n1.htm
ttp://www.nasa.gov/mission_pages/tdm/ldsd/
ttp://www.youtube.com/watch?v=9h1NtQJ59kM
ttp://www.youtube.com/watch?v=G0asOKx5Xp0

288 :ご冗談でしょう?名無しさん:2014/06/29(日) 23:01:11.07 ID:???.net
ニュートン力学の話です。
よく「A君は最初の5分間時速4kmで歩き、次の5分間時速6kmで歩きました」
みたいな算数問題の表現ありますが、これを厳密に考えると、
加速度が無限大になる瞬間があるから厳密にはあり得ない話ですよね?

では不連続なのが速度ではなく加速度なら、つまり
「B君は最初の10秒間は2m/s^2で加速し、次の10秒間は1m/s^2で減速しました」
のように瞬間的に加速度が不連続であってもそれはあり得るのでしょうか?

289 :ご冗談でしょう?名無しさん:2014/06/29(日) 23:24:07.27 ID:EMBk2HTw.net
力が変化するには有限の時間が必要だから現実にはありえない

290 :ご冗談でしょう?名無しさん:2014/06/29(日) 23:28:49.22 ID:???.net
>>288
>加速度が無限大になる瞬間があるから厳密にはあり得ない話ですよね?
無限大の力を無限小の時間だけ受けることで有限の大きさの速度変化が瞬間的に起こる
それは数学的にデルタ関数を用いて定式化できる

291 :ご冗談でしょう?名無しさん:2014/06/29(日) 23:56:43.19 ID:oFTd/UjV.net
>>288
それを現実的でないというのなら物理で取り扱う微分方程式はほぼ壊滅状態
加速度が無限大でも移動距離とか何秒後にどの位置にいるかということなら問題なく解ける
微分方程式で調べたいことが何かということを考えれば無限大がでてきても問題ないケースが多い

292 :ご冗談でしょう?名無しさん:2014/06/30(月) 03:38:45.57 ID:???.net
巨視的には無限の加速度でも特に差し支えないし
微視的には不確定性原理からそもそも考える意味がない

293 :ご冗談でしょう?名無しさん:2014/06/30(月) 03:55:00.42 ID:2uLuI/Hd.net
>>288は加速度習いたての高校生とかだろ?
じゃあ“そうだね!有り得ないね!”くらいのこと言ってあげれば十分じゃないかな

294 :ご冗談でしょう?名無しさん:2014/06/30(月) 11:21:45.88 ID:???.net
>>288
力学や信号処理によく出る不連続問題は >>290 の数学処理が可能。
現実には不可能でもエネルギー、運動量変化が無限大でなければ物理的に問題ない。

295 :ご冗談でしょう?名無しさん:2014/06/30(月) 15:16:58.07 ID:???.net
閻魔王宮と地球はどっちの方が広いのでしょうか?

296 :ご冗談でしょう?名無しさん:2014/06/30(月) 16:21:08.86 ID:???.net
この宇宙は実は閻魔王宮な可能性はありますか?

297 :ご冗談でしょう?名無しさん:2014/06/30(月) 17:13:47.84 ID:???.net
カノニカル分布では系がエネルギー固有状態E_iをとる確率は
p_i = exp(-β E_i) / Z
で与えられますが、これはエネルギーが低い状態の方が出現確率が高いってことですよね?

だとすれば系は絶対零度に近い状態ほど出現確率が高いということになりますが、
直感的には熱浴の温度Tと近い状態ほど出現確率が高くなる気がします
これはどう考えたらよいでしょうか?

298 :ご冗談でしょう?名無しさん:2014/06/30(月) 17:20:44.81 ID:???.net
温度は自由にとれんの?

299 :ご冗談でしょう?名無しさん:2014/06/30(月) 18:12:19.22 ID:???.net
>>297
>これはエネルギーが低い状態の方が出現確率が高いってことですよね?
ここまではいいが

「系は絶対零度に近い状態ほど出現確率が高い」というのは意味を成さない
「エネルギーが低い」を「絶対零度に近い」などと意味不明な置き換えをするから変なことになる

300 :ご冗談でしょう?名無しさん:2014/06/30(月) 18:19:35.63 ID:???.net
>>299
すみません、条件が曖昧でした

粒子の運動エネルギー以外のエネルギーはほとんどない系においてはエネルギーと温度は比例するかと思うんですがどうでしょうか

301 :ご冗談でしょう?名無しさん:2014/06/30(月) 18:32:40.13 ID:???.net
いや
「これはエネルギーが低い状態の方が出現確率が高い」って正しくないだろ
混合状態だってこと理解してないんじゃねーの

302 :ご冗談でしょう?名無しさん:2014/06/30(月) 19:01:36.82 ID:???.net
>>301
でも
p_i = exp(-β E_i) / Z
に従えばE_iが低いほど確率は高いですよね?違うんですか?

303 :ご冗談でしょう?名無しさん:2014/06/30(月) 19:47:40.85 ID:???.net
確率じゃなくて重み

304 :ご冗談でしょう?名無しさん:2014/06/30(月) 20:21:53.78 ID:???.net
>>303
どういうことでしょうか…?

305 :ご冗談でしょう?名無しさん:2014/06/30(月) 20:38:37.40 ID:???.net
>>297
カノニカル分布って熱浴にひたされて温度ー定になってる系のお話だってことは理解してる??
理解してりゃ「絶対零度が一番よく出現」なんておかしな結論考えないはずだが。

306 :ご冗談でしょう?名無しさん:2014/06/30(月) 20:43:42.55 ID:???.net
>>305
分かってます、だからこそ
p_i = exp(-β E_i) / Z
をどう解釈したら良いか分からないんです

307 :ご冗談でしょう?名無しさん:2014/06/30(月) 20:48:26.49 ID:???.net
>>306
だったら、定数である温度が下っちゃダメじゃん。
exp(ーE/kT)という分布になったところで熱浴と考えてる系を合成した系の状態数が最大になる。それよりエネルギー上げても下げても状態数が減る(エントロピーが減る)。

308 :ご冗談でしょう?名無しさん:2014/06/30(月) 21:01:34.37 ID:???.net
すべての i での和が1にならないと確率とは呼ばない

309 :ご冗談でしょう?名無しさん:2014/06/30(月) 21:02:07.91 ID:???.net
あ、1になってるな
じゃあ確率でいいよ

310 :ご冗談でしょう?名無しさん:2014/06/30(月) 21:03:08.38 ID:???.net
>>307
補足すると、回りに熱浴があって温度一定という条件置いた時点でエネルギーは外から流れ込んだり外へ流れ出したりするもんだと考えてる。それがカノ二カル分布ってもの。
その状況でエネルギーが絶対零度状態に下がるなんてあり得ねえ。

311 :ご冗談でしょう?名無しさん:2014/06/30(月) 21:10:56.60 ID:???.net
分かりそうな気がしてきました
ありがとうございました!

312 :ご冗談でしょう?名無しさん:2014/06/30(月) 22:38:12.53 ID:???.net
>>306
別に温度 kT = E_i となるような状態 i だけからなる分布を考えてもいいけど
それは熱平衡状態では実現しないというのが統計力学の教えるところでしょ
何でもかんでも直観で分かるなら苦労しないよ

313 :ご冗談でしょう?名無しさん:2014/06/30(月) 22:41:48.04 ID:???.net
熱平衡状態では、というか
熱浴とエネルギーをやり取りするときの熱平衡状態では、だけど

314 :ご冗談でしょう?名無しさん:2014/06/30(月) 23:13:44.35 ID:???.net
たびたびすいません

@熱平衡状態のため系の温度Tは一定
Aよって系のエネルギーEも一定
BだとするとエネルギーE_iが低い方が出現確率が高い(p_i = exp(-β E_i) / Z )とは言えない
CなぜならE_iは一定でばらつきがないから

たぶんどれも変なことを言ってるんでしょうが、どうも理解できません
何が間違ってるんでしょうか?よろしくお願いします…

315 :ご冗談でしょう?名無しさん:2014/06/30(月) 23:18:40.42 ID:???.net
エネルギーの期待値計算してみーや

316 :ご冗談でしょう?名無しさん:2014/07/01(火) 00:37:31.27 ID:???.net
具体的に書いてみます

系のエネルギーが粒子の運動エネルギーの和で表される理想気体では

Z = (2^N V^N / N! )(2π m k T / h^2)^3N/2

となりますが、系のどの粒子もほとんど運動エネルギーをもってないような状態(E_0とします)の出現確率は
p_i = exp(-β E_0) / Z
となり、これが最も出現しやすいということになるのでしょうか?
これは明らかに熱平衡状態ではありませんがどこが間違ってるのか分かりません

317 :ご冗談でしょう?名無しさん:2014/07/01(火) 00:40:45.48 ID:???.net
「明らかに熱平衡状態ではありませんが」という点

318 :ご冗談でしょう?名無しさん:2014/07/01(火) 00:40:58.22 ID:???.net
適当なエネルギー揺らぎ E±δE の範囲でとれる状態の数 W(E) は一般にエネルギーが高くなるほど増える
この疎視化により確率密度: P(E) ∝ W(E)*exp(-βE) = exp(-β(E-TS)) を得る。
(エントロピー:S(E) = k log(W) の関係を使った)
なので熱浴下では E ではなく E-TS が最低の状態が平衡状態となる
自由エネルギー: F = E - TS

319 :ご冗談でしょう?名無しさん:2014/07/01(火) 00:49:30.00 ID:???.net
>>317
熱平衡ってことですか?
熱浴の温度がTでも系の出現確率はE_0が一番高いのは納得できませんが…

>>318
すぐには理解できないのでしばらく考えてみます
解説ありがとうございます

320 :ご冗談でしょう?名無しさん:2014/07/01(火) 00:51:02.36 ID:???.net
E_i = kT なる微視的状態しかないのはミクロカノニカルだろ
温度はカノニカルアンサンブルにしか定義されないとか思ってんのか?

321 :ご冗談でしょう?名無しさん:2014/07/01(火) 01:51:35.45 ID:???.net
「温度Tで平衡状態になってる系はマクロな視点では一つの状態に定まってるが、微視的状態は膨大な数がある
つまり系のエネルギーはEに定まってるように見えてもE〜E±δEぐらいの間で微妙に揺らいでいる
微視的状態の出現確率はその E〜E±δE の範囲においてエネルギーが低いほど高い
だから E〜E±δE の範囲外の絶対零度になるエネルギーはありえない
どの程度揺らいでいるかは、U = 〈E〉= Σp_i E_i で求められる」

こういうことでしょうか?
分かった気がします

322 :ご冗談でしょう?名無しさん:2014/07/01(火) 01:55:25.50 ID:???.net
>>321
それミクロカノニカルだろ
もう今すぐに理解するのは諦めたら?

323 :ご冗談でしょう?名無しさん:2014/07/01(火) 01:57:48.26 ID:???.net
Eが内部エネルギーなら2行目までは間違っていないけど
それ以降は全然進歩がない

324 :ご冗談でしょう?名無しさん:2014/07/01(火) 02:04:51.56 ID:???.net
みなさんありがとうございました
基本から勉強しなおします

325 :ご冗談でしょう?名無しさん:2014/07/01(火) 04:39:35.78 ID:???.net
解決しました
p(E) = Ω(E,V,N) exp(-βE) / Z

p_i = exp(-βE_i) / Z
の区別がついてないだけでした

326 :ご冗談でしょう?名無しさん:2014/07/01(火) 07:55:28.67 ID:???.net
ブラックホールは「ホーキング輻射」によって長い時間をかけて蒸発するらしいですが、
では中性子星やクォーク星は永久に安定なんでしょうか?

327 :ご冗談でしょう?名無しさん:2014/07/01(火) 14:09:20.88 ID:???.net
式とかわかんねーよ
日本語でお願いします

328 :ご冗談でしょう?名無しさん:2014/07/01(火) 15:47:35.04 ID:???.net
>>327
ばかも〜ん!
物理屋なら
「日本語なんてわかんねえよ、数式でお願いします」
と言わんかあ!

329 :ご冗談でしょう?名無しさん:2014/07/01(火) 16:26:02.04 ID:???.net
YouTubeの創業者とノイマンはどっちの方が賢いのでしょうか?

330 :ご冗談でしょう?名無しさん:2014/07/01(火) 16:40:01.58 ID:???.net
キャハ

331 :ご冗談でしょう?名無しさん:2014/07/01(火) 16:42:25.60 ID:???.net
ミクロカノニカルなんて使う場面あんのか?

332 :ご冗談でしょう?名無しさん:2014/07/01(火) 17:37:47.38 ID:???.net
ミクロカノニカルの設定の量子系を真面目に考えるのには使うんじゃないか?
まあ、あとは(グランド)カノニカル分布への踏み台だよね。

333 :ご冗談でしょう?名無しさん:2014/07/01(火) 19:40:19.43 ID:???.net
分布定数回路において、反射係数が負の時は電流は正になるんですか

334 :ご冗談でしょう?名無しさん:2014/07/01(火) 20:03:16.64 ID:???.net
数学は暗記で済ませても良いですか?
頭が致命的に悪いので、基本、暗記しかできないのです。

335 :ご冗談でしょう?名無しさん:2014/07/01(火) 20:07:24.02 ID:???.net
いいよ

はい次の質問者さんどうぞ

336 :ご冗談でしょう?名無しさん:2014/07/02(水) 00:11:28.18 ID:???.net
お大事に

337 :ご冗談でしょう?名無しさん:2014/07/02(水) 00:35:21.84 ID:???.net
頭の悪い奴に数学はできない
算数やっとけ

338 :ご冗談でしょう?名無しさん:2014/07/02(水) 16:55:14.20 ID:???.net
ドップラーシフト
fd=2v/λを証明したいのですがどのように求めればいいですか?
答えでも構わないのですがヒントや考え方を教えて下さるとうれしいです。

339 :ご冗談でしょう?名無しさん:2014/07/02(水) 17:43:03.18 ID:???.net
>>338
何でそれ2がついてんの?
一定の速度で通りすぎた救急車の
サイレンの周波数のちがいとか?

340 :ご冗談でしょう?名無しさん:2014/07/02(水) 18:01:43.29 ID:H/q1T/jj.net
反射して、正から負、それから正に変わることがある。べクトル関数で、虚無解まではいかなくても、負の解で反射して、正になることはある。数学では、ベクトル引いて高校レベルで考えてもわかるが、ベクトル関数だと、大学レベルの問題だ

341 :ご冗談でしょう?名無しさん:2014/07/02(水) 19:40:42.44 ID:???.net
最近光学100倍ズームのカメラとか売られていますが、理論的には光学って何倍
まで可能なんですか?というか光って直進するから物理的には無限に遠い物体からの
光も正確に届くんですよね?何故光学1兆倍のカメラとかがないんでしょうか?

342 :ご冗談でしょう?名無しさん:2014/07/02(水) 19:49:09.07 ID:0hBu1vBv.net
それは…子牛の数が足りんよーになるからとちゃうん?

343 :ご冗談でしょう?名無しさん:2014/07/02(水) 19:58:56.16 ID:???.net
>>342
詳しく
子牛って何ですか?

344 :ご冗談でしょう?名無しさん:2014/07/02(水) 20:01:16.82 ID:???.net
横だけど、孔子

345 :ご冗談でしょう?名無しさん:2014/07/02(水) 20:06:42.98 ID:???.net
光は波の性質があるから倍率を上げようとして焦点距離を伸ばすと像がボケるんですよ
倍率(焦点距離)を保ったままボケないようにするにはレンズ口径を広げればいいです
それでも手持ち出来るサイズでの限界は数百倍程度でしょう。(参考: 天体望遠鏡)
重量と工作精度の問題かと思います。

346 :ご冗談でしょう?名無しさん:2014/07/02(水) 20:06:48.60 ID:???.net
子曰く、

347 :ご冗談でしょう?名無しさん:2014/07/02(水) 20:19:25.27 ID:0hBu1vBv.net
光あれ!

348 :ご冗談でしょう?名無しさん:2014/07/02(水) 20:21:56.84 ID:???.net
それは主だろ

349 :ご冗談でしょう?名無しさん:2014/07/02(水) 20:22:09.47 ID:???.net
>>339
実験で光のドップラー効果というものをやりました。
きっと光の戻り光だと思うのですが、実験ノートを見ると
レーザー光f
戻り光f'
f'=f+-fd さらに
fd=2v/λ v=Vcosθ

と、記載されてました

350 :ご冗談でしょう?名無しさん:2014/07/02(水) 20:25:55.65 ID:???.net
だから、式とかわかんねーっていってるだろ!日本語つかえよ、日本人だろ!

351 :ご冗談でしょう?名無しさん:2014/07/02(水) 21:28:59.98 ID:???.net
物理屋なら日本語で言うなよ!式で言えよ!だろ?

352 :ご冗談でしょう?名無しさん:2014/07/02(水) 21:29:08.92 ID:???.net
東大法学部卒の最高裁長官と東大理学部卒の東大数学科教授はどっちの方がエリートですか?

353 :ご冗談でしょう?名無しさん:2014/07/02(水) 21:33:05.93 ID:???.net
どっちもエリートです

354 :ご冗談でしょう?名無しさん:2014/07/02(水) 21:38:53.79 ID:???.net
>>349
回転鏡?

355 :ご冗談でしょう?名無しさん:2014/07/02(水) 21:40:14.96 ID:???.net
>>353
アホに触るなら最高裁長官に突っ込むぐらいしてやれよボケてんだから

356 :ご冗談でしょう?名無しさん:2014/07/02(水) 21:42:47.17 ID:???.net
>>353
どっちかだったらどっちの方がエリートですか?

357 :ご冗談でしょう?名無しさん:2014/07/02(水) 21:43:32.21 ID:???.net
>>349
戻り光はコヒーレントなのでしょうか?

358 :ご冗談でしょう?名無しさん:2014/07/02(水) 21:47:03.27 ID:???.net
>>356
少林寺拳法と野沢菜はどっちが強いのでしょうか?

359 :ご冗談でしょう?名無しさん:2014/07/02(水) 21:47:53.85 ID:???.net
>>356
日本の総理大臣とアメリカの大統領はどちらがエリートですか?

360 :ご冗談でしょう?名無しさん:2014/07/02(水) 21:55:20.60 ID:???.net
>>352
偏差値40のおっさんが質問してもしょうがねーだろ

361 :ご冗談でしょう?名無しさん:2014/07/02(水) 21:57:59.09 ID:???.net
銀河にある天体の回転速度について。
目に見えている物質だけじゃ説明がつかないので、
目に見えている物質の何倍ものダークマターが銀河中に満ちている、との予測があります。

でもおかしくないですか?
太陽系は目に見えている天体の質量だけで何の矛盾も無くケプラーの法則を満たした運動をしていると思います。
なんで太陽系の惑星にはダークマターの影響がないのでしょうか?

362 :ご冗談でしょう?名無しさん:2014/07/02(水) 22:01:56.10 ID:???.net
おかしくないです、たぶん。

363 :ご冗談でしょう?名無しさん:2014/07/02(水) 22:03:29.36 ID:???.net
>>354
回転鏡ですね、一定速度で回転してました

>>357
使用した光がレーザー光でしたので戻り光もコヒーレントだと思います

364 :ご冗談でしょう?名無しさん:2014/07/02(水) 22:43:53.00 ID:???.net
>>357
4元ベクトルをローレンツブーストして鏡の静止系で考えれば簡単
vec(E', p'c) = γ*Matrix({1, +β},{+β, 1}) *vec(E, pc)
そこでは光をそのまま反射するだけなので、観測者の座標系に戻れば
vec(E'', +p''c) = γ*Matrix({1, -β},{-β, 1}) *vec(E', -p'c)
vec(E'', -p''c) = γ*Matrix({1, +β},{+β, 1}) *vec(E', +p'c)
        = γ^2 * Matrix({1+β^2, +2β},{+2β, 1+β^2}) *vec(E, pc)
E'' = ( (1+β^2)E + 2βpc )/(1-β^2) = ((1+β)/(1-β)) E
f'' ≒ (1 + 2β) f
δf = f''-f = 2βf = 2v/λ

365 :ご冗談でしょう?名無しさん:2014/07/02(水) 22:45:39.05 ID:???.net
>>361
http://scienceblogs.com/startswithabang/2013/07/03/does-dark-matter-affect-the-motion-of-the-solar-system/
http://physics.stackexchange.com/questions/107764/why-doesnt-dark-matter-affect-planetary-motion

太陽系が収まる球内のダークマターの総質量は月1個分くらいだそうな。

366 :ご冗談でしょう?名無しさん:2014/07/02(水) 22:47:48.54 ID:???.net
偉いねー、オーダーが違うだろうといわないなんて

367 :ご冗談でしょう?名無しさん:2014/07/02(水) 23:24:26.25 ID:jmVTZxWM.net
反射して、正から負、それから正に変わることがある。べクトル関数で、虚無解まではいかなくても、負の解で反射して、正になることはある。数学では、ベクトル引いて高校レベルで考えてもわかるが、ベクトル関数だと、大学レベルの問題だ
登記

368 :ご冗談でしょう?名無しさん:2014/07/02(水) 23:35:50.87 ID:???.net
>>363
鏡に光が当たった瞬間とその点から一波長分進んだ瞬間の
鏡と素元波を描いてみれば?波長が決まれば光速は一定だから
周波数は求まる。

369 :ご冗談でしょう?名無しさん:2014/07/02(水) 23:39:11.56 ID:???.net
>>363
ああいや、一波長分進んだってのは曖昧すぎるか。
入射光の極大点が鏡に当たった瞬間と、
その次の極大点が鏡に当たった瞬間の図を
重ねて描いてみるといい

370 :ご冗談でしょう?名無しさん:2014/07/03(木) 00:08:04.94 ID:C2X77Ydj.net
点は描画できない
線も描画できない
空間上に面も描画できない
描画できないのにそんなものあるんでしょうか?

371 :ご冗談でしょう?名無しさん:2014/07/03(木) 00:12:02.49 ID:???.net
誤差ほにゃららの範囲で点と見なせるてことでいいんじゃないか
点とは何かは数学の人に聞いて

372 :ご冗談でしょう?名無しさん:2014/07/03(木) 00:25:18.50 ID:???.net
正規グロー放電は何故一定になるんですか。

373 :ご冗談でしょう?名無しさん:2014/07/03(木) 00:50:22.27 ID:???.net
量子力学の摂動論で、縮退しない場合の1次や2次の「摂動エネルギーが0になる」とき、それが実際にどういう意味を持つのかが理解できません
V^の値によってある次数の摂動エネルギーが0になることがありますが(例えば2次の摂動エネルギーが0になるとき)、それはあくまで計算結果であって意味は持たないのでしょうか

374 :ご冗談でしょう?名無しさん:2014/07/03(木) 01:27:00.88 ID:???.net
全体のエネルギーに摂動のそれぞれの次数のエネルギーの寄与を表してるんじゃないの
例えばテーラー展開の各項もそうでしょ

375 :ご冗談でしょう?名無しさん:2014/07/03(木) 01:29:21.62 ID:???.net
摂動エネルギーがゼロってことは多少の摂動ポテンシャルを加えても状態が変わらないってことだよ。
何を訊いてるかしらないけど。

376 :ご冗談でしょう?名無しさん:2014/07/03(木) 06:36:27.45 ID:???.net
そのエネルギーがマイナスになることはありえないことですか?

377 :ご冗談でしょう?名無しさん:2014/07/03(木) 08:13:04.97 ID:???.net
普通にあるわ

378 :ご冗談でしょう?名無しさん:2014/07/03(木) 16:30:41.32 ID:???.net
>>365
>太陽系が収まる球内のダークマターの総質量は月1個分くらいだそうな。

そうなんだ、ダークマターのイメージが出来た。
太陽系が収まる球内の光のエネルギー量も質量に換算すると同じくらいあるのだろうか?

379 :357:2014/07/03(木) 19:17:05.41 ID:???.net
>>365
ありがとうです
下のリンクが自分と同じタイプの疑問のページだったのでわかりやすかった!

リンク先を参考に自分で計算してみました。
太陽系(海王星軌道くらいの球体)内のダークマターは、
月1個分よりもだいぶ少なくて月の100分の1くらいのオーダーの質量になりました

> moon (7 x 10^22 kg) has a larger impact
だと「月はダークマターよりも大きな影響を与える」って意味なのでそんなものかと

>>378
光って背景放射?たいしてエネルギーにならなそうだけど

380 :ご冗談でしょう?名無しさん:2014/07/03(木) 19:21:09.32 ID:???.net
>>378
ない。

太陽定数1366W/m^2に4π(1AU)^2をかければ、
太陽放射の総エネルギー量が3.8e23kWと分かる。
太陽系の大きさをヘリオポーズまで160AUと大きめに見積ると、
光がここまで到達するのに22hourかかる。
よってこの球体の中に存在する光のエネルギーは3.1e28kJ、
質量に換算すれば3.4e11トンとなる。
月の質量は8.1e19トンだから、比べると5ppbぐらいしかない。

381 :ご冗談でしょう?名無しさん:2014/07/03(木) 21:41:15.67 ID:???.net
宇宙飛行士と内閣総理大臣はどっちの方がエリートですか?

382 :ご冗談でしょう?名無しさん:2014/07/03(木) 21:48:41.83 ID:???.net
キャハとヒマラヤはどっちがいい?

383 :ご冗談でしょう?名無しさん:2014/07/03(木) 22:25:34.79 ID:???.net
     ∧_∧
 ピュー (  ^^ ) <これからも山崎を応援して下さいね(^^)。
  =〔~∪ ̄ ̄〕
  = ◎――◎                      山崎渉

384 :ご冗談でしょう?名無しさん:2014/07/03(木) 23:38:21.11 ID:???.net
片持ち梁を宇宙空間で振動させても実際永遠に振動せんと、減衰してやん?
この場合全部熱エネルギになると思う?

385 :ご冗談でしょう?名無しさん:2014/07/04(金) 00:01:49.34 ID:???.net
それどこのことば?

386 :ご冗談でしょう?名無しさん:2014/07/04(金) 00:17:16.74 ID:???.net
>>384
重力波分は熱に比べると無視できる出来るベルだと思うよ。

387 :ご冗談でしょう?名無しさん:2014/07/04(金) 01:10:32.64 ID:???.net
>>380
ありがとうございます。
ダークマタ−なみを期待していたんですが。

>光がここまで到達するのに22hourかかる
こういうところは頭いいと感じた

388 :ご冗談でしょう?名無しさん:2014/07/04(金) 01:11:34.88 ID:???.net
単数形なあたりへの皮肉か

389 :ご冗談でしょう?名無しさん:2014/07/04(金) 01:18:19.60 ID:???.net
関数f(x)のMaclaurin展開を考えると、xは無次元であることが要求されるという話がありますが
ここでy=a(1-e^-t) (但しaは定数、tは時間)である物理量yが与えられる場合について考察する時、指数関数をMaclaurin展開すると両辺の次元が一致しない矛盾はどう解決するんですか?

390 :ご冗談でしょう?名無しさん:2014/07/04(金) 01:43:54.86 ID:???.net
e^-t とした時点で -t は無次元量でないといけない。t を時間として使いたいなら t の係数は時間の逆数の次元を持つ。
あと Maclaurin 展開だと分かりづらいので原点以外で Taylor 展開した方がいい。
今回の場合、級数の収束とか考えなくていいのでどこで展開しても問題なし。

391 :ご冗談でしょう?名無しさん:2014/07/04(金) 01:48:06.35 ID:???.net
指数の肩に次元のある物理量は乗ってもいいと何回言えばわかるんだ…

392 :ご冗談でしょう?名無しさん:2014/07/04(金) 01:50:01.91 ID:???.net
指数関数の引数が無次元出ない場合を考察する場合がそもそも無い

393 :ご冗談でしょう?名無しさん:2014/07/04(金) 01:52:25.95 ID:???.net
>>391
対数と勘違いしてないか?
指数だと定義からして、(e^x)' = e^x なんだから x は無次元じゃないと e^x の次元が決まらない。

394 :ご冗談でしょう?名無しさん:2014/07/04(金) 02:00:10.56 ID:???.net
>>388
あー、GNU units使ってたもんでついやっちゃったわ。

395 :ご冗談でしょう?名無しさん:2014/07/04(金) 02:40:19.59 ID:dLmEhKHa.net
ブラックホール・流体対応という理論の存在を最近知りました。
かなり興味あるんですが、詳しい人いますか?

396 :ご冗談でしょう?名無しさん:2014/07/04(金) 02:57:55.60 ID:???.net
>>395
AdS/CFT 対応とかホログラフィ原理とかで調べれば出てくる
最近は日本語の書籍・解説記事も増えてきた

397 :ご冗談でしょう?名無しさん:2014/07/04(金) 04:00:17.19 ID:dLmEhKHa.net
ありがとうございます

流体力学は深く勉強していて、相対論もそれなりに勉強しました

場の量子論はほんの少ししか勉強してないという現状ですが、この分野を将来研究して見たいと思うのですが、なにを勉強すれば良いでしょうか
このまま場の量子論でしょうか

398 :ご冗談でしょう?名無しさん:2014/07/04(金) 07:29:05.77 ID:???.net
途中計算で次元が不定になったからって矛盾とは限らないんですよね
もちろん両辺の次元が異なったらマズいですが
y=e^-t (yは次元が不定)
z=y^a (aは時間の逆数次元, zは無次元)

399 :ご冗談でしょう?名無しさん:2014/07/04(金) 09:09:28.61 ID:???.net
>>398
「不定」じゃなくて両辺で異なるわけだが
少なくとも指数関数は定義されない

400 :ご冗談でしょう?名無しさん:2014/07/04(金) 10:17:17.38 ID:???.net
>>397
横だけど、ちょとぐぐればすぐ分かるだろう、超弦理論と重力理論

401 :ご冗談でしょう?名無しさん:2014/07/04(金) 17:00:06.35 ID:fbOn1TFE.net
ある物理現象を時間反転してもエントロピーが増大するってどういうことですか?
いわゆる逆再生みたいな現象が物理法則を反しない、とのことですが。。

402 :ご冗談でしょう?名無しさん:2014/07/04(金) 17:09:16.62 ID:???.net
>>393
対数だって引数は最終的には無次元にまとまらないとおかしいわけだが

403 :ご冗談でしょう?名無しさん:2014/07/04(金) 17:11:57.22 ID:???.net
東京大学医学部の学生とtokyotubeの創業者はどっちの方が凄いのでしょうか?

404 :ご冗談でしょう?名無しさん:2014/07/04(金) 17:15:16.65 ID:???.net
tokyotubeってなんでしょうか

405 :ご冗談でしょう?名無しさん:2014/07/04(金) 17:37:57.34 ID:???.net
>>384
真空中の梁の振動特性の動画あるけど見る?
見たい奴いればVHSカセットから変換してうpするけどあんまり興味なさそうだな

406 :ご冗談でしょう?名無しさん:2014/07/04(金) 17:40:11.67 ID:???.net
>>403
通報されたぞ、早く逃げろよ
http://maguro.2ch.net/test/read.cgi/budou/1380498037/485,486

407 :ご冗談でしょう?名無しさん:2014/07/04(金) 17:55:08.82 ID:???.net
>>386
なるほど. 重力場は抜けてました
今日材力の教授に聞いてきたのですが若干材力の範疇を超えるのでちょっと難しいなと言ってました
かといって物理学科は他キャンパスなので聞けず(笑)

>>405
ありがとうございます
興味深いですがお手間を取らせるのは申し訳ないので結構です

408 :ご冗談でしょう?名無しさん:2014/07/04(金) 18:02:56.61 ID:???.net
>>385
三重県です
混乱させて申し訳ありませんでした

409 :ご冗談でしょう?名無しさん:2014/07/04(金) 18:25:03.33 ID:???.net
医師と物理学者はどっちの方が多くの知識を必要としますか?

410 :ご冗談でしょう?名無しさん:2014/07/04(金) 18:52:27.14 ID:???.net
>>404
http://www.tokyo-tube.com/

411 :ご冗談でしょう?名無しさん:2014/07/04(金) 18:52:59.36 ID:???.net
暗記量では医師でしょうね
お互い分野がかなり違いますね
物理学者に薬の種類はそれほど分からない
医師にはリアクションホイールの仕組みなんてまず説明出来ない

ですが医学でも動脈硬化、血石のつまり改善の為に流体力学の理論が用いられたりします. どちらが上ということはなくお互い支えあっているのです.

412 :ご冗談でしょう?名無しさん:2014/07/04(金) 19:08:19.08 ID:???.net
>>411
東京大学医学部の学生とtokyotubeの創業者だと、どっちの方が凄いのでしょうか?

413 :ご冗談でしょう?名無しさん:2014/07/04(金) 19:10:42.76 ID:???.net
>>412
キャハと山本はどっちが強いのでしょうか?

414 :ご冗談でしょう?名無しさん:2014/07/04(金) 20:16:58.06 ID:???.net
ヒマラヤとそれを相手する奴はどっちがバカですか?

415 :ご冗談でしょう?名無しさん:2014/07/04(金) 21:59:06.37 ID:nllcJS0P.net
eの複素数乗って苦し紛れにテイラー展開に当てはめて定義されてますけど
たとえばそのような定義じゃなくてももし指数などの代数法則が成り立っていて便利であれば別の定義も考えられるんですか?
eの複素数乗なんてしょせん計算が便利になるだけの鉄砲玉なんでしょうか?

416 :ご冗談でしょう?名無しさん:2014/07/04(金) 22:13:17.00 ID:???.net
>>415
高木の解析概論でもジックリ読んでみろ。

417 :ご冗談でしょう?名無しさん:2014/07/04(金) 22:15:27.89 ID:???.net
>>415
lim_{N→∞}(1+ix/N)^N
の方なら苦し紛れでもないんでは。

418 :ご冗談でしょう?名無しさん:2014/07/04(金) 22:16:22.23 ID:???.net
>>415
物理でテーラー展開を否定してどうすんねん

419 :ご冗談でしょう?名無しさん:2014/07/04(金) 22:20:38.83 ID:???.net
その鉄砲玉のおかげで工学がいかに進展してきたことか...

420 :ご冗談でしょう?名無しさん:2014/07/04(金) 22:48:01.58 ID:nllcJS0P.net
話かわりますけど
光が屈折率の大小関係で固定端反射や自由端反射になるのはなぜですか

421 :ご冗談でしょう?名無しさん:2014/07/04(金) 22:50:53.00 ID:???.net
整数も質点もみんな鉄砲玉って事でいいよ

422 :ご冗談でしょう?名無しさん:2014/07/04(金) 22:57:58.02 ID:???.net
>>421
質点も含む点概念はあだ花になりそうだけどね。

423 :ご冗談でしょう?名無しさん:2014/07/04(金) 23:11:13.93 ID:???.net
神と仏はどっちが上ですか?

424 :ご冗談でしょう?名無しさん:2014/07/04(金) 23:56:13.27 ID:BvHC6v6b.net
量子力学の基本原理ってなんですか?

425 :ご冗談でしょう?名無しさん:2014/07/05(土) 00:04:07.49 ID:Aqfwu5s7.net
静電力のエネルギーは二体間の距離をrとしてkQq/rで表せる
Qが作る静電場に試験電荷qを置く
静止状態の試験電荷は加速運動し、速度がΔv、位置がΔrだけ変位した
ここで力学的エネルギー保存の関係からkQq/r-kQq/(r+Δr)=mΔv^2/2とすることのどこが間違いになるか
説明してください

426 :ご冗談でしょう?名無しさん:2014/07/05(土) 00:15:28.73 ID:???.net
>>425
(v+Δv)^2-v^2≠Δv^2

427 :ご冗談でしょう?名無しさん:2014/07/05(土) 00:34:34.18 ID:Aqfwu5s7.net
静止状態ですよ

428 :ご冗談でしょう?名無しさん:2014/07/05(土) 00:45:22.17 ID:???.net
変化を表す場合は
変化後から変化前を引くもんだよ

429 :ご冗談でしょう?名無しさん:2014/07/05(土) 02:16:56.02 ID:???.net
物理を分かると人生楽しくなりますか?

430 :ご冗談でしょう?名無しさん:2014/07/05(土) 02:43:57.45 ID:???.net
どうでしょう
実際人生には物の理(ことわり)外の事のも多いですからねぇ
論理的でない行動を起こすのも人間だったりします

431 :ご冗談でしょう?名無しさん:2014/07/05(土) 06:06:03.28 ID:???.net
自然の基礎(物理)法則の論理構成を理解するのは楽しい。丸暗記ではわからない。

432 :ご冗談でしょう?名無しさん:2014/07/05(土) 07:01:24.25 ID:???.net
危険物乙4の資格の勉強をしているのですが、
小中高と、全くと言って良いほど理科の勉強をしてこなかった為、
基礎物理学、基礎科学の項が、さっぱり解りません。

どこからやり直せば、乙4の基礎物理化学を理解する事が
出来るようになりますか?

スレ違いかも知れないのですが宜しくお願いします。

433 :ご冗談でしょう?名無しさん:2014/07/05(土) 07:13:35.84 ID:???.net
>>432
資格試験は4択問題60%以上だろうから受験問題集の丸暗記で合格できる。

434 :428:2014/07/05(土) 09:29:47.61 ID:???.net
>>433
レスありがとうございます。

でも、ある程度、理解をしたいんです。

435 :ご冗談でしょう?名無しさん:2014/07/05(土) 09:34:20.94 ID:???.net
>>432
ここで聞けよ
http://hello.2ch.net/test/read.cgi/lic/1404049721/

436 :ご冗談でしょう?名無しさん:2014/07/05(土) 09:41:42.58 ID:???.net
>>424
重ね合わせの原理

437 :ご冗談でしょう?名無しさん:2014/07/05(土) 11:08:12.46 ID:???.net
>>436
簡単に言うと何を重ねるの?

438 :ご冗談でしょう?名無しさん:2014/07/05(土) 14:01:42.66 ID:???.net
>>437


439 :ご冗談でしょう?名無しさん:2014/07/05(土) 15:56:01.02 ID:???.net
神と仏はどっちの方が上ですか?

440 :ご冗談でしょう?名無しさん:2014/07/05(土) 18:09:50.47 ID:???.net
大学受験で物理を使わずに大学に入学して入学後に
大学物理をならってついていくことはできますか?

現在、高校で物理を選択しておらず入試でも物理を当然使いません。
ですが、大学では物理関係を専攻しようと思っているのですが無謀でしょうか?

441 :ご冗談でしょう?名無しさん:2014/07/05(土) 18:52:01.38 ID:???.net
>>440
ぜんぜん無謀ではない。高校の物理の内容など高が知れてるし、変な癖もあるし。
ただしうまく大学で勉強する物理に入って行く道をみつけないと。

442 :ご冗談でしょう?名無しさん:2014/07/05(土) 19:10:10.28 ID:???.net
無茶かな。
高校物理がわかっているものとして進められるだろうから。
用語の問題も生じるかも知れない。
知ってて当たり前のことを知らないというのはどこで躓くかわからんよ。

443 :ご冗談でしょう?名無しさん:2014/07/05(土) 19:21:14.15 ID:???.net
凡人には無理だろ

444 :ご冗談でしょう?名無しさん:2014/07/05(土) 19:22:19.66 ID:???.net
神と仏はどっちの方が上ですか?

445 :ご冗談でしょう?名無しさん:2014/07/05(土) 19:38:27.26 ID:???.net
大学の物理に向けて頑張るなら
今は取りあえず微積分や空間ベクトルについて勉強しておくと後々楽ができると思います
高校物理は無視して構いません

446 :ご冗談でしょう?名無しさん:2014/07/05(土) 20:45:59.70 ID:???.net
東京大学理学部物理学科卒の物理学者と、fc2の創業者はどっちの方が凄いのでしょうか?

447 :ご冗談でしょう?名無しさん:2014/07/05(土) 21:39:42.41 ID:???.net
fc2の創設者かな

448 :ご冗談でしょう?名無しさん:2014/07/05(土) 21:42:43.34 ID:???.net
理由は?

449 :ご冗談でしょう?名無しさん:2014/07/05(土) 21:44:34.08 ID:???.net
キャハと野沢菜はどっちが凄いのでしょうか?

450 :ご冗談でしょう?名無しさん:2014/07/05(土) 22:15:46.14 ID:/JdOnEZC.net
すみません、飛行機っていったいどおゆうワケで飛んでいるんですか?
あ、もちろん人や荷物が短時間に移動できるから、なんて回答は望んでません。
要するに、

「ベルヌーイの定理」で飛んでいる
「ベルヌーイの定理」で説明してあるけどそれは間違い

いったいどっちなん?

451 :ご冗談でしょう?名無しさん:2014/07/05(土) 22:19:13.49 ID:???.net
ttp://www.youtube.com/watch?v=bv3m57u6ViE

452 :ご冗談でしょう?名無しさん:2014/07/05(土) 22:30:15.49 ID:/JdOnEZC.net
ええ、それがベルヌーイの定理による説明ですね。でも、ブルーバックス
読んでたら、それが間違いだったと海外のさほどエラくない(たぶん)が
証明した、という解説を見かけたものですから。そして、先日書店でこんな
帯がついてる新刊本を見かけて…いったいどっちなん?物理的には決着ついてんの?
って。
ttp://ecx.images-amazon.com/images/I/51fSkJqnCbL._SS400_.jpg

453 :ご冗談でしょう?名無しさん:2014/07/05(土) 22:32:19.72 ID:???.net
「定理を使って飛ぶ」とかそういう頭悪い表現やめろ

454 :ご冗談でしょう?名無しさん:2014/07/05(土) 22:37:20.17 ID:/JdOnEZC.net
「使って」なんてどこにも書いてありませんがあなたには見えるの?

455 :ご冗談でしょう?名無しさん:2014/07/05(土) 22:39:35.95 ID:/JdOnEZC.net
いいや、仕方ないから流体力学スレッドで聞こうっと………ないやないかいっ!

456 :ご冗談でしょう?名無しさん:2014/07/05(土) 22:39:54.07 ID:???.net
「そういう」と書いたがお前には見えないのか?

457 :ご冗談でしょう?名無しさん:2014/07/05(土) 22:41:42.53 ID:/JdOnEZC.net
文句が言いたいなら直接著者さん(つか、帯の煽り考えた編集さんかw)に
言ってください。

458 :ご冗談でしょう?名無しさん:2014/07/05(土) 22:44:47.08 ID:???.net
おまえが聞けよ

459 :ご冗談でしょう?名無しさん:2014/07/05(土) 22:46:09.08 ID:???.net
そもそもベルヌーイの定理を用いて
揚力を計算できるというだけの話を
なぜこの理由で飛ぶという話になるのか

460 :ご冗談でしょう?名無しさん:2014/07/05(土) 22:47:33.35 ID:???.net
馬鹿雌に物理は無理だから諦めてさっさと首吊って自殺しろカス。
馬鹿雌は馬鹿で雑魚な上に粘着質で負けず嫌いだから不愉快だし目障りなんだよ。

461 :ご冗談でしょう?名無しさん:2014/07/05(土) 22:48:34.64 ID:/JdOnEZC.net
よし来た、無いなら仕方ない、それじゃあボクが立てちゃうからネ!
そこでどんなたいとるがええか募集します。採用された方にはみんな
からひゅーひゅー!の罵声を浴びる権利を進呈します。

462 :ご冗談でしょう?名無しさん:2014/07/05(土) 22:49:05.92 ID:???.net
飛行機の飛ぶ原理については、とりあえず竹内糞とアンダーソンの言う事は無視してください

463 :ご冗談でしょう?名無しさん:2014/07/05(土) 22:50:45.10 ID:???.net
鶴丸航空のHPにはジェットエンジンによる推進力も、と書いてあるが

464 :ご冗談でしょう?名無しさん:2014/07/05(土) 22:54:03.36 ID:Aqfwu5s7.net
いずれにしても空気の流れの向きを変えることによる反作用で浮いている

バネが伸びるから浮き上がる
バネに与える作用の反作用により浮き上がる
言い方や見方が違うだけで同じこと

465 :ご冗談でしょう?名無しさん:2014/07/05(土) 22:56:56.23 ID:Aqfwu5s7.net
いちおうナビエ-ストークス方程式を解くことで説明できるはず
CFDは詳しくないから知らんけど

466 :ご冗談でしょう?名無しさん:2014/07/05(土) 23:08:01.57 ID:/JdOnEZC.net
ハハハ、ここは頭の固い人たちが多いインターネットですね。
それではひとつ、あのNHKが誇る大人気番組【サイエンスZERO】のMC
竹内 薫先生のご著書でも熟読して柔軟な思考をするトレーニングと行きましょう。
ttp://ecx.images-amazon.com/images/I/51YZAecH1IL.jpg
【飛行機はなぜ飛ぶのか?科学では説明できない!】
ttp://ecx.images-amazon.com/images/I/51%2B-L-dmgmL.jpg
その目次を見ると、プロローグにはこうあります。
    ↓↓↓↓↓↓↓↓↓↓↓↓↓↓↓↓
    飛行機はなぜ飛ぶのか?実はよくわかっていない

467 :446:2014/07/05(土) 23:10:47.84 ID:/JdOnEZC.net
………うわ……wこんなネタ…wサラっと流されて終わりだと思ったら…wけっこー盛り上がってて…w
うけるーーー…wつかぶっちゃけこのオレがとまどっっちゃてるうわこれっ!www

468 :ご冗談でしょう?名無しさん:2014/07/05(土) 23:12:23.99 ID:???.net
なんだ馬鹿だったか、疲れた

469 :446:2014/07/05(土) 23:13:32.70 ID:/JdOnEZC.net
>>464
ハイ、それ…いわゆるアンダーソン先生派の【抗力(*)】説ですよネ。

* 流体力学and航空力学なんかで言う『抗力』とは別物。

470 :446:2014/07/05(土) 23:16:11.51 ID:/JdOnEZC.net
ところで、>>466読むと登場する「老物理学者」っていったい誰なんすか?
そして、「老物理学者」からアンダーソンet al.の和訳本出したいから
出版社紹介してくれないか、と竹内が頼まれたそうなんですが、その後
その出版計画はどうなったんでしょう?教えて!>裏事情に詳しい方

471 :ご冗談でしょう?名無しさん:2014/07/05(土) 23:17:56.94 ID:???.net
馬鹿ビッパーはもう寝ろよ

472 :446:2014/07/05(土) 23:19:10.39 ID:/JdOnEZC.net
>>465
解くためにはまずその方程式を立てなきゃならんわけで、
そのためにはまずモデルを構築せねばならないってことになるんですが、
そのモデルをデッチ上げる段階から問題になってるように愚考してるんですが…。

473 :ご冗談でしょう?名無しさん:2014/07/05(土) 23:43:05.03 ID:???.net
物理現象には理由など存在しないし、
かつ理論的に等価な複数の説明が存在しうる。
それだけの話だよ

474 :ご冗談でしょう?名無しさん:2014/07/05(土) 23:56:41.09 ID:???.net
ベルヌーイの定理を用いて予言することができないのは、
ある翼を設計したときにどのような流れが生じるか。
どのような流れが生じるかは実験かナビエストークス方程式を解くしかない。
生じる流れがわかっていればベルヌーイの定理から揚力を計算できる。

475 :ご冗談でしょう?名無しさん:2014/07/06(日) 00:02:12.66 ID:???.net
>>472
するってーとナビエストークス方程式は間違ってて、
それに基づいて作られた流体シミュレーションを用いて設計された
ステルス爆撃機が飛んでるのは偶然なのか。

476 :ご冗談でしょう?名無しさん:2014/07/06(日) 00:02:54.86 ID:???.net
もうベルヌーイの定理でもなんでも使って空飛んでこいよ
きっと気持ちよかろう

477 :446:2014/07/06(日) 00:12:58.96 ID:e45laDXT.net
だってタケウチカオルだってゆってましたお!

   それなら何故 背面飛行 なんてのが可能なんだ?!

478 :446:2014/07/06(日) 00:14:28.83 ID:e45laDXT.net
そしてボクは不思議に思いました。

  …んーーーっと、背面飛行って難病くらい持続可能なん?
  ずっと続けてられるの?

479 :ご冗談でしょう?名無しさん:2014/07/06(日) 00:25:58.66 ID:???.net
>>477
迎え角とかの諸条件をちゃんと考えれば

480 :446:2014/07/06(日) 00:27:50.42 ID:e45laDXT.net
>>473
>理論的に等価な複数の説明が可能
ところが現状はそうなっていません。

 ベルヌーイ派とアンダーソン一派は、お互いに相手の解釈を
間違いだとして退けています。

481 :446:2014/07/06(日) 00:29:26.75 ID:e45laDXT.net
>>479
背面飛行だと抑え角は逆になっちゃう=抑えてないお角になっちゃうんじゃないっすか?
すいません、現実の飛行機がどーなってるかよく知らないんです。
ごめんね無知でおバカでw

482 :ご冗談でしょう?名無しさん:2014/07/06(日) 00:56:58.13 ID:apEenNzk.net
>>469
なんらかの作用の反作用で浮いているという当たり前のことを言ってるだけだぞ
で、作用が考えられるものは空気以外にないわけだから空気の運動量を可変させることで
飛んでいるというロジックに穴は無い

483 :ご冗談でしょう?名無しさん:2014/07/06(日) 01:05:15.89 ID:???.net
豆腐がこっちでウザがられてる・・・

484 :ご冗談でしょう?名無しさん:2014/07/06(日) 01:06:06.28 ID:???.net
>>481
目が悪いの?迎え角だよ

485 :ご冗談でしょう?名無しさん:2014/07/06(日) 01:07:20.99 ID:???.net
>>482
その上ベルヌーイの定理とも矛盾しないしね

486 :ご冗談でしょう?名無しさん:2014/07/06(日) 01:07:38.91 ID:apEenNzk.net
>>477
MSFSやってたから飛行機は割と詳しいけど
相応のフォワードプレッシャーを加えなければ+1G(重力+揚力-慣性力=揚力)で勢いよく落ちてくよ

487 :ご冗談でしょう?名無しさん:2014/07/06(日) 01:09:49.29 ID:???.net
なにそのアタマ悪そうな式

488 :ご冗談でしょう?名無しさん:2014/07/06(日) 01:10:38.20 ID:???.net
大体迎え角を適切に取れば凧も飛ぶのにな

489 :ご冗談でしょう?名無しさん:2014/07/06(日) 01:19:25.37 ID:apEenNzk.net
わかりにくかったかな?
重力-慣性力=0の系を考えてるのね
その状況で残るは揚力
揚力は背面飛行でも最初のうちは変わらないだろうから+1G(機体が基準で)
実際は重力が仕事するから速度が上昇してどんどん揚力も増える
だから+1Gよりもどんどん大きくなるけどね
結局、背面飛行でなにもパイロットが操作しなかったらみんな逆立ちできるような状況になる
揚力の方向は常に翼の上面方向だからヨークを傾けるだけで素人でもグラスのワインをこぼさずにロールすることができる
旅客機じゃ厳しいだろうけど

490 :ご冗談でしょう?名無しさん:2014/07/06(日) 01:21:25.50 ID:???.net
>>489
話が全然違う方向にいってるぞ

491 :ご冗談でしょう?名無しさん:2014/07/06(日) 01:22:53.50 ID:???.net
背面飛行してる場合だろうと
高度一定なら慣性力が現れるのは地上の静止系でも飛行機の静止系でもなく変な系だけだけど
どんな系なの?

492 :ご冗談でしょう?名無しさん:2014/07/06(日) 01:24:48.95 ID:apEenNzk.net
高度一定の条件は考えてないよ
高度一定ならそりゃみんな天井にまっさかさまだよ

493 :ご冗談でしょう?名無しさん:2014/07/06(日) 01:32:30.95 ID:???.net
本の解説には翼が流れと平行の場合しか書いてないが、実際はプラスにもマイナスにも角度を付けられる。
実際、飛行機が離陸するときは大きな迎え角を取ってる。
背面飛行でもマイナスの迎え角を取ることで揚力を生じさせることは可能だよ。失速しない範囲が狭いけど。

494 :ご冗談でしょう?名無しさん:2014/07/06(日) 01:34:30.01 ID:apEenNzk.net
実際ANAが夜中に背面飛行してまったとき誰も気づかなかったらしい  脱線してすまん

495 :ご冗談でしょう?名無しさん:2014/07/06(日) 01:38:16.68 ID:???.net
科学はWhy?ではなくHowを教えるものである

496 :ご冗談でしょう?名無しさん:2014/07/06(日) 01:38:28.69 ID:???.net
>>480
そいつらが間違ってるだけ。
因みに昔の一般向けの説明は間違ってた。

497 :ご冗談でしょう?名無しさん:2014/07/06(日) 01:38:54.97 ID:???.net
いいからブルーバックスなんて捨てちまえよ

498 :ご冗談でしょう?名無しさん:2014/07/06(日) 01:40:14.89 ID:???.net
そういや辞典に載ってたサイフォンの説明が間違ってた例もあったな

499 :ご冗談でしょう?名無しさん:2014/07/06(日) 01:43:53.19 ID:???.net
互いに隣り合った小部分は上下に分かれて、翼のうしろで再び出会うという同時到達説は誰が考えたのかね。
英語のサイトでも見かけるから日本人ではないようだが。

500 :ご冗談でしょう?名無しさん:2014/07/06(日) 01:51:32.95 ID:apEenNzk.net
ブルーバックス馬鹿にすんなよ
良書もあるだろ

501 :ご冗談でしょう?名無しさん:2014/07/06(日) 01:59:10.63 ID:???.net
ttp://blog.livedoor.jp/rbkyn844/archives/5633113.html
これ見て思ったんだが毛細管現象とか色々含めて管の太さはどれくらいあれば1の言い分通りになる?
内径φ5mmの耐圧チューブで想像したけど実感がわかない
内径φ13mmの塩ビパイプだとなんとなく納得できるからこの間に答えがありそうな気がするな

502 :ご冗談でしょう?名無しさん:2014/07/06(日) 04:23:59.05 ID:X5vJQbCQ.net
量子テレポーテーションをわかりやすく教えての巻

503 :ご冗談でしょう?名無しさん:2014/07/06(日) 05:33:23.66 ID:???.net
A「xxxxxだった」
B「oooooだった」
A「知ってた」

504 :ご冗談でしょう?名無しさん:2014/07/06(日) 12:05:17.71 ID:KZ8FOJzL.net
水素を金属に固定化するだけ、物質をつくることができる量産もできる、簡単に、登記

海水を燃料に転換する技術は、科学者たちが長年高い関心を持ってきた課題だが、米海軍は今月に入り、この研究がついに成果をみせたと発表した。
液体炭化水素燃料の開発は「大変革」をもたらすものと考えられてきた。敵に攻撃されやすい燃料補給ラインを画期的に短縮化できるからだ。
米海軍のフィリップ・クロム(Philip Cullom)中将は「われわれにとって極めて画期的な達成だ」と語った。
米専門家らは、海水から二酸化炭素と水素ガスを抽出する方法を特定。これらを触媒コンバーターを用いて液化させ、燃料に変えるという。
船舶だけではなく航空機への利用も期待されており、実現すればタンカーに依存することなく、洋上の船舶で燃料を生産できるようになる。

505 :ご冗談でしょう?名無しさん:2014/07/06(日) 12:16:03.85 ID:???.net
>>502
量子テレポーテーションでは、系Aの量子状態を、系Bの量子状態として再現できる。
この際、Aの量子状態は変化するので原本が残らない転送といえる。
よくある誤解だが、量子的な遷移が関わるからといって、どんなに離れていても速度無限大の瞬時にテレポートされるわけではない
手続きの途中で従来の通信手段が必要なので光速を越える事は無い。
またこの間にA(あるいはB)の量子状態を「知る」事はできない。
量子力学上、素性の知れない一点物の量子状態を「知る」事は本質的に出来ない。
観測行為の瞬間に他の重ね合わせ成分が消し飛ぶので、どんな係数比率だったのかは知りようがない。

506 :ご冗談でしょう?名無しさん:2014/07/06(日) 14:10:58.16 ID:???.net
力学と解析力学って何が違うの?

507 :ご冗談でしょう?名無しさん:2014/07/06(日) 14:41:40.91 ID:???.net
同じだよ

508 :ご冗談でしょう?名無しさん:2014/07/06(日) 18:04:31.11 ID:???.net
ラグランジアンの無有

509 :ご冗談でしょう?名無しさん:2014/07/06(日) 18:41:32.19 ID:???.net
運動方程式をあまりよく考えないでアプリオリに出せるかどうか

510 :ご冗談でしょう?名無しさん:2014/07/06(日) 19:07:35.53 ID:???.net
この回路のスイッチSをA側に閉じた場合、どのような回路になるかという問題です。答えは8Ωと2Ωが並列に接続されており電源20vに繋がっています。 そこで、15Ωが消えるのはわかります。
ですが(5Ωの抵抗と10Ω×2の並列 以下x)が消えるのはなぜですか?xはそのまま2Ωと直列に接続されないんですか?

http://
i.imgur.com/
vSsckfW.jpg
BB2Cからに投稿で行が長すぎると言われました。ご了承ください。

511 :ご冗談でしょう?名無しさん:2014/07/06(日) 19:20:56.63 ID:???.net
死んだら無ですか?

512 :ご冗談でしょう?名無しさん:2014/07/06(日) 19:38:41.96 ID:???.net
物理学徒ならまず実験してみよ

513 :ご冗談でしょう?名無しさん:2014/07/06(日) 19:43:03.14 ID:???.net
>>510
たぶん 12Ω抵抗の誤植でしょう
大学の教科書だってそれくらいの間違いはよくある事です

514 :ご冗談でしょう?名無しさん:2014/07/06(日) 20:11:49.87 ID:8hYzMs0s.net
いや大学の教科書だからこそ間違いが多いんだよなぁ
小中高と違って人の目があまり入らないから

515 :ご冗談でしょう?名無しさん:2014/07/06(日) 21:42:15.23 ID:???.net
将来理論物理の道に進むには大学でどんな勉強をするんですか?

516 :ご冗談でしょう?名無しさん:2014/07/06(日) 21:43:58.24 ID:???.net
>>515
毎日楽しいですか?

517 :ご冗談でしょう?名無しさん:2014/07/06(日) 22:17:23.62 ID:???.net
>>505 せんせー! ありがとう!

518 :ご冗談でしょう?名無しさん:2014/07/06(日) 22:40:11.56 ID:???.net
神と仏はどっちの方が上ですか?

519 :ご冗談でしょう?名無しさん:2014/07/06(日) 22:43:55.40 ID:???.net
>>518
仏神習合でググれ

520 :ご冗談でしょう?名無しさん:2014/07/06(日) 22:50:34.68 ID:???.net
>>519
そんな単語初めて見ました。

521 :ご冗談でしょう?名無しさん:2014/07/06(日) 23:09:27.43 ID:???.net
それでも釣られる奴

522 :ご冗談でしょう?名無しさん:2014/07/06(日) 23:27:10.80 ID:clhbRb9p.net
アルミニウムの線膨張係数の実験をしたんですが、温度上昇時の結果と温度下降時の結果に差が生じてしまいました。
これは正しい実験ができていないということなんでしょうか?

523 :ご冗談でしょう?名無しさん:2014/07/06(日) 23:28:43.16 ID:???.net
熱処理の教科書を読んでみましょう

524 :ご冗談でしょう?名無しさん:2014/07/06(日) 23:31:07.01 ID:???.net
実験やりました。結果が予想したものではありませんでした。
評価:D

525 :ご冗談でしょう?名無しさん:2014/07/07(月) 00:11:15.79 ID:???.net
>>522
そこを考察するのが学生実験でしょうがよ

526 :ご冗談でしょう?名無しさん:2014/07/07(月) 00:20:08.27 ID:???.net
ブラックホールって質量だけじゃなくて電荷も情報として持ってるとのことですが、
事象の地平面の外までその電荷の情報は伝わるのでしょうか?

527 :ご冗談でしょう?名無しさん:2014/07/07(月) 00:29:25.44 ID:???.net
>>526
それだけの電荷が「入った」という情報は常に取得可能だし、
その電荷が事象の地平面を越えたとたんに
外部の電場が変動するのはおかしいだろ?

528 :522:2014/07/07(月) 01:03:03.29 ID:???.net
>>527
そうですね
でも、電場って光子が介在して相互作用するんですよね?
事象の地平面の内部から光子が外に出ていることになるんですか?

529 :ご冗談でしょう?名無しさん:2014/07/07(月) 01:06:08.47 ID:???.net
>>528
入る前からあった電場が入った瞬間消えるって考える方が非物理的だろ。

530 :ご冗談でしょう?名無しさん:2014/07/07(月) 02:39:54.86 ID:???.net
質点系の添え字にindexのiではなくovの筆記体?が使われてる表記を見たんですが何の略か分かる人いますか?
ヘルムホルツの19世紀の本なんですけど

531 :526:2014/07/07(月) 03:01:56.90 ID:???.net
すみません自己解決しました、ドイツ文字の筆記体のaでした

532 :ご冗談でしょう?名無しさん:2014/07/07(月) 03:09:33.80 ID:???.net
>>516
楽しいです

533 :ご冗談でしょう?名無しさん:2014/07/07(月) 03:16:50.92 ID:???.net
ドイツ文字の筆記体(フラクトュール、亀の子文字)は他にも
k, O, S, x, y, Y なんかがかなり凶悪なのでご注意を...
最近の物理本では減ったかもしれないが数学本ではバンバン使われてます

534 :ご冗談でしょう?名無しさん:2014/07/07(月) 06:15:32.49 ID:???.net
>>528
クーロン電場は消えないが、電磁波は消える。
仮想光子の相互作用は、「事象の境界面」越しに生じるが、
実光子のそれは、否。

535 :ご冗談でしょう?名無しさん:2014/07/07(月) 09:55:58.60 ID:???.net
>>528
光子は電磁波を量子化したものであって、
電場を量子化したものではない。

536 :ご冗談でしょう?名無しさん:2014/07/07(月) 10:03:24.67 ID:???.net
お好み焼き屋だが
熱した鉄板を気流中に置いたらどう考えても温度分布が変わってて焼き加減が変わってくるんだ?これは熱が移動したってこと?対策はある?職場暑くてさ

537 :ご冗談でしょう?名無しさん:2014/07/07(月) 10:05:45.01 ID:???.net
>>536
温度移流でググってみろ熱流体の分野だな

538 :ご冗談でしょう?名無しさん:2014/07/07(月) 10:07:25.32 ID:???.net
状況が不明だしお好み焼きの焼きかげんがわかるわけがない、温度測って試行錯誤しろよ

539 :ご冗談でしょう?名無しさん:2014/07/07(月) 10:20:02.64 ID:???.net
お好み焼き屋の頭の弱さが良く出てるw

540 :ご冗談でしょう?名無しさん:2014/07/07(月) 10:24:43.45 ID:???.net
>>536
何を聞きたいのかさっぱりわからないので
まずは物理より国語を勉強してきてくれ

541 :ご冗談でしょう?名無しさん:2014/07/07(月) 10:30:27.57 ID:???.net
お好み焼きを焼く鉄板の近くにいると暑いので扇風機なんかで風が来るようにしたら焼き加減が変わってしまった
焼き加減に影響を与えなで涼しくする方法ない?
という質問だろ

風の向きを工夫して自分にしか当たらないようにするしかない
しかし鉄板からは熱が放射されて来るのでそれを防ぐ手立てはないね

542 :ご冗談でしょう?名無しさん:2014/07/07(月) 11:07:12.66 ID:???.net
まず服を脱ぎます

543 :ご冗談でしょう?名無しさん:2014/07/07(月) 11:10:16.78 ID:???.net
お前が脱いでどうすんねんて話だ

544 :ご冗談でしょう?名無しさん:2014/07/07(月) 11:36:17.80 ID:???.net
もし現状で夏と冬で焼き加減が変わらないのなら、冬並みに室温を下げれはあるいはいけるのでは
もし下がらないのなら、鉄板を温めるのに使われなかった熱を逃がす。例えばバーナーの排気を室内ではなく屋外にファンで送るなど
鉄板からの輻射は変えられないし(焼き加減が変わるから)主に赤外線だから遮るよりない

まとめると、換気しつつエアコンの吹き出し口にホームセンターで買ってきた100Φのアルミ蛇腹を取り付けて自分に向けとく

545 :ご冗談でしょう?名無しさん:2014/07/07(月) 11:45:34.79 ID:g+MhdeEp.net
熱力学において一階の微分方程式の解法で全微分や完全微分なるものが出てきますが
二階の微分方程式でそのようなものがありますか?

546 :ご冗談でしょう?名無しさん:2014/07/07(月) 12:25:30.67 ID:???.net
>>545
微分形式でググれ

547 :ご冗談でしょう?名無しさん:2014/07/07(月) 12:28:01.22 ID:???.net
>>545
意味不明だが、拡散方程式の解き方を聞いてるの?

548 :ご冗談でしょう?名無しさん:2014/07/07(月) 13:54:05.74 ID:???.net
>>536
>>541の通りだとしたら、空調服買えば?
ttp://www.tokyotoraya-shop.jp/smp/list.php?type=class&mcat=89687

549 :ご冗談でしょう?名無しさん:2014/07/07(月) 20:01:31.67 ID:zyEub2zO2
交流電流が10kHzの時の1サイクルの時間を計算したいんですが、一秒は1000msで、さらにこれをマイクロ秒に直して10000Hzで割ればokですか?

550 :ご冗談でしょう?名無しさん:2014/07/07(月) 19:09:24.40 ID:thFC/J6n.net
解析概論って具体的に何がすごいの?

551 :ご冗談でしょう?名無しさん:2014/07/07(月) 19:29:38.24 ID:???.net
とにかくすごいんです

552 :ご冗談でしょう?名無しさん:2014/07/07(月) 20:13:06.38 ID:???.net
高校一般大学の数学教科書読んで楽しいと思うか

553 :ご冗談でしょう?名無しさん:2014/07/07(月) 20:22:59.63 ID:g+MhdeEp.net
>542
どうも。検索してみます。
>543
拡散方程式ではないです。というか変数分離だとかフーリエ変換だとか
微分方程式と解法について勉強中で、、熱力学で内部エネルギーだとかエントロピーだとか
いうところで全微分ってのが出てくるですよね。取り急ぎなので式は書きませんが。
物理で全微分が出てくるのは他にどの分野があるのかと思いまして。

554 :ご冗談でしょう?名無しさん:2014/07/07(月) 20:32:04.42 ID:???.net
多変数なら自然に出てくるだろ

555 :ご冗談でしょう?名無しさん:2014/07/07(月) 20:32:45.35 ID:???.net
偏微分が出てくれば全微分も出てくるでしょう

556 :ご冗談でしょう?名無しさん:2014/07/07(月) 20:32:50.75 ID:thFC/J6n.net
量子力学と流体力学どっちが難しいですか?

557 :ご冗談でしょう?名無しさん:2014/07/07(月) 20:34:10.23 ID:???.net
>>556
バカみたいな質問はやめて下さい

558 :ご冗談でしょう?名無しさん:2014/07/07(月) 20:47:55.92 ID:g+MhdeEp.net
>549 550
そうなんですか。物理を趣味的に本を流し読みしているので
きちんとした理解ができていないんです。
偏微分が出てきたら全微分も出てくるぐらいに考えて読み直そうと思います。どうも。

559 :ご冗談でしょう?名無しさん:2014/07/07(月) 20:49:55.92 ID:???.net
>>558
趣味でやるのいいですね
大学の微分積分と線形代数は一緒にやったほうがいいと思います
理解も早まりますし楽しいですよ

560 :ご冗談でしょう?名無しさん:2014/07/07(月) 20:54:08.54 ID:g+MhdeEp.net
>>559
大人になって物理の楽しさを知った者です。
励ましありがとうございます。

561 :542:2014/07/07(月) 22:17:47.50 ID:???.net
>>553
スレもあったわ。
http://wc2014.2ch.net/test/read.cgi/sci/1301844272/

562 :ご冗談でしょう?名無しさん:2014/07/08(火) 00:03:38.49 ID:WUs9Uisb.net
>556
物理の本はそこそこ持っててそのスレで話されている一般相対性理論の本も
持っているんですが微分形式とか多様体とかまだ私には背伸びの内容でして。
でも頑張って理解したいと思います。
ググったら熱力学も微分形式で書けるんですね。いろいろと勉強になります。どうも。
あ、そうそう偏微分方程式の右辺と左辺の積分は定数になるという本の記述は
全微分と今の話から来てるのかと納得しました(でいいのかな)。おやすみなさい。

563 :ご冗談でしょう?名無しさん:2014/07/08(火) 00:11:31.15 ID:???.net
ポエマー、相関、量関にならないように

564 :ご冗談でしょう?名無しさん:2014/07/08(火) 00:13:43.66 ID:???.net
相「関」って…

565 :ご冗談でしょう?名無しさん:2014/07/08(火) 00:19:42.50 ID:???.net
>>541
なるほど、すばらしい読解力

566 :ご冗談でしょう?名無しさん:2014/07/08(火) 01:01:26.32 ID:???.net
ニューヨークやドバイの超高級マンションに住むには、ビジネスで大成功を収めないと無理でしょうか?

567 :ご冗談でしょう?名無しさん:2014/07/08(火) 01:54:53.97 ID:???.net
>>566 治安が良いマンション買えばOK

568 :ご冗談でしょう?名無しさん:2014/07/08(火) 11:23:32.80 ID:tDyvZ4HY.net
東大の佐藤勝彦を呼んで、今年の学会の数式を書いてください。費用は払い済みだからです。

569 :ご冗談でしょう?名無しさん:2014/07/08(火) 13:18:29.72 ID:WUs9Uisb.net
>560
なんか違うっぽいですね。
夜になったら自分のやれる範囲で式の変形を紙に書いて考えて直してみます。

570 :ご冗談でしょう?名無しさん:2014/07/08(火) 14:23:16.31 ID:???.net
>>569
何が違うの?
560はあなたへのレスではないと思うけど。

571 :ご冗談でしょう?名無しさん:2014/07/08(火) 15:58:07.53 ID:???.net
お好み焼きを、ものすごい圧力で潰したら
どれくらいのエネルギーが放出されますか?

572 :ご冗談でしょう?名無しさん:2014/07/08(火) 16:00:05.39 ID:???.net
ものすごいエネルギーが出ます

573 :ご冗談でしょう?名無しさん:2014/07/08(火) 16:50:58.64 ID:WUs9Uisb.net
>>570
おお、そうですね。モバイルで見ているので見にくさから勘違いでした。
でもいい機会になったので紙に式を書いて本を見ながら
いろいろ格闘してみようと思います。ご指摘ありがとうございます。

574 :ご冗談でしょう?名無しさん:2014/07/08(火) 19:31:40.73 ID:???.net
銀河の質量は、その光度から求まりますか?

575 :ご冗談でしょう?名無しさん:2014/07/08(火) 19:41:48.05 ID:???.net
ノーベル物理学賞を受賞するのとGoogleみたいな会社を興すのはどっちの方が難しいですか?

576 :ご冗談でしょう?名無しさん:2014/07/08(火) 20:48:37.42 ID:N3/liwHr.net
今もっとも熱い物理学の分野はなんですか?
高エネルギー物理学以外で

577 :ご冗談でしょう?名無しさん:2014/07/08(火) 20:52:25.58 ID:???.net
坂道のパラドックスが分かりません
距離Xm離れた場所に行くのに、ルートAは坂道が1か所ある
Bは真っ直ぐ

AとBはどっちがしんどいかというとAですよね?
同じ距離なのに何故ですか?Aは坂道がある分、下り道でしんどさが
相殺されると思うのですが、何故Aのほうが結果としてエネルギーが
必要なのでしょうか?

578 :ご冗談でしょう?名無しさん:2014/07/08(火) 20:52:43.86 ID:N3/liwHr.net
金がかからない独自研究できそうな分野も教えてください

579 :ご冗談でしょう?名無しさん:2014/07/08(火) 21:05:11.26 ID:???.net
>>574
銀河の構成がどれも等しいという仮定の下に求まる。

580 :ご冗談でしょう?名無しさん:2014/07/08(火) 21:11:15.93 ID:???.net
>>577
そもそも何故始点と終点におけるエネルギー差が
しんどさに関係すると思うんだ?

摩擦(というかエネルギー散逸)を無視しても
坂がある方が終点に到達するために必要な
最低エネルギーは坂がある方が大きいぞ

581 :ご冗談でしょう?名無しさん:2014/07/08(火) 21:55:15.59 ID:???.net
>>576
秘密

582 :ご冗談でしょう?名無しさん:2014/07/08(火) 22:03:23.64 ID:???.net
ワイルズやペレルマンのような天才数学者と、Googleの創業者はどっちの方が凄いのでしょうか?

583 :ご冗談でしょう?名無しさん:2014/07/08(火) 22:06:02.87 ID:???.net
キャハと野沢菜はどちいがアホでしょうか?

584 :ご冗談でしょう?名無しさん:2014/07/08(火) 22:33:51.02 ID:???.net
>>580
レスありがとうございます
もう少し噛み砕くとどうなるでしょう?

585 :ご冗談でしょう?名無しさん:2014/07/08(火) 22:42:02.90 ID:???.net
孫正義とニュートンはどっちの方が凄いのでしょうか?

586 :ご冗談でしょう?名無しさん:2014/07/08(火) 22:51:26.99 ID:???.net
>>585
はやく答えろよ
>>583

587 :ご冗談でしょう?名無しさん:2014/07/08(火) 23:43:13.53 ID:???.net
台風8号、昨日の予報では数日後に中心気圧910hPaだったのが
今日の予報では予想進路が急角度で曲がり945hPaになってしまった。

渦って外圧で方向が変わるとエネルギーが下がるのでしょうか。

588 :ご冗談でしょう?名無しさん:2014/07/08(火) 23:48:48.12 ID:???.net
ローレンツ力の起源って現代の物理学では解明されてますか?
Maxwellの電磁気論は知ってますが、ローレンツ力が何故発生するのかわかりません。

589 :ご冗談でしょう?名無しさん:2014/07/08(火) 23:52:51.79 ID:???.net
>>588
相対論的効果

590 :ご冗談でしょう?名無しさん:2014/07/08(火) 23:54:55.73 ID:N3/liwHr.net
相対論的効果でなんで電荷が増えたり減ったりするの?

591 :ご冗談でしょう?名無しさん:2014/07/08(火) 23:57:48.02 ID:???.net
電荷は変わらねえよ

592 :ご冗談でしょう?名無しさん:2014/07/09(水) 00:00:30.98 ID:fOIndAH5.net
じゃあ電荷分布が変わるとして
なんで余分な静電力が生まれるの?

593 :ご冗談でしょう?名無しさん:2014/07/09(水) 00:02:27.94 ID:???.net
変わらねえって言ってんだろ
変わるのは電流分布だ

594 :ご冗談でしょう?名無しさん:2014/07/09(水) 00:07:00.84 ID:???.net
>>588
U(1)ゲージ不変性

595 :ご冗談でしょう?名無しさん:2014/07/09(水) 02:11:21.86 ID:???.net
>>577
坂があるなら距離は長くなるだろ

596 :ご冗談でしょう?名無しさん:2014/07/09(水) 02:28:19.12 ID:???.net
>>577
山道は下りの方が膝がしんどい

597 :sage:2014/07/09(水) 02:30:11.32 ID:ur9SG1sw.net
三角柱を倒す時。
底面の三辺の内一番長い辺側に倒れやすいと感覚的に思うのですが、
それを証明する方法を教えて下さい。

598 :ご冗談でしょう?名無しさん:2014/07/09(水) 02:40:17.75 ID:???.net
柱の重心がどこにくると倒れるのかを調べ
さらに三角形の重心のと各辺との距離を調べる

599 :ご冗談でしょう?名無しさん:2014/07/09(水) 02:44:36.10 ID:???.net
>>597
まず仮定している条件を詳しく列挙する

600 :ご冗談でしょう?名無しさん:2014/07/09(水) 03:10:08.71 ID:???.net
>>598 >>599
ありがとうございます。
駐車場の隣のバイクが倒れてきて車両を傷付けられました。
バイクはタイヤ二つとスタンドの三点で停めてあるため、
スタンドがない右側に倒れやすいと証言したのですが、
裁判所で車と違いバイクは公共の資料が少ないため、
バイクが右側に倒れやすい事をきちんと証明してほしいと言われました。
バイクの図面から大まかに重心を計算して各辺との距離の資料を提出してみます。

601 :ご冗談でしょう?名無しさん:2014/07/09(水) 03:39:18.00 ID:???.net
そういう意味ではバイクを三角柱と考えるのはモデルとして間違い

ていうか、右側のタイヤは丸いのに対して、スタンドの足は」みたいな形を
しているので、普通に考えて左側には倒れにくいだろ

602 :ご冗談でしょう?名無しさん:2014/07/09(水) 04:00:36.57 ID:???.net
裁判だと見た目でとか普通に考えてとかだと証拠にならないらしくて。
あんまりスレ独占しちゃうといけないんでスタンドとタイヤの形の違いも含めてもう少し具体的になってから質問しにきます。
マジでありがとうございました。

603 :ご冗談でしょう?名無しさん:2014/07/09(水) 05:41:19.38 ID:???.net
裁判で証明しろと言われたなら、理論的に示す事より実証する事の方が求められるんじゃないか?

つまりバイクとはかりを用意して、倒すときに必要な力の大きさを、いろんなケースを想定しながら実際に測れということだ。

604 :ご冗談でしょう?名無しさん:2014/07/09(水) 05:45:57.54 ID:???.net
ガリレオていうか、交通事故鑑定員たのめよ

605 :ご冗談でしょう?名無しさん:2014/07/09(水) 10:21:09.29 ID:???.net
経験的には停めてあるバイクはサイドスタンド側に倒れやすいな

スタンドがなければ左に倒れるように重心が外れていて、それに「つっかえ棒」をしてるような形だから
そのバイクが起き上がっていく方向に倒すにはバイクを起こすに足る外力が要る

もしバイクを人が動かそうとしていた時なら、やはり自分の方(左側)に傾けて体で重さを受けるから、そのつもりの時に向こう側(右側)に重心が外れると倒しやすい

状況が定かでないのではっきりとは言いにくいが、モデル化するならバイクの前後輪を繋ぐ線を軸にして左右方向のモーメントの釣り合いを考えたらどうだろうか

606 :ご冗談でしょう?名無しさん:2014/07/09(水) 16:57:40.25 ID:fOIndAH5.net
>>593
相対論的効果によって電荷が現れるって書いてあるんですが

607 :ご冗談でしょう?名無しさん:2014/07/09(水) 17:24:37.50 ID:???.net
>>606
ローレンツ変換によって電荷の分極が起きること。電荷の代数和は保存される(相対論では原理)。

608 :ご冗談でしょう?名無しさん:2014/07/09(水) 19:28:00.84 ID:???.net
下手の考え休むに似たり、と
ローレンツ変換勉強しましょうね
じゃないと文字通りお話にならない

609 :ご冗談でしょう?名無しさん:2014/07/09(水) 19:39:17.98 ID:???.net
レゴがくっつく物理的な理由は
プラスチックの弾性力って理解であってますか?

610 :ご冗談でしょう?名無しさん:2014/07/09(水) 19:53:27.47 ID:???.net
摩擦力という未解明の力のせいです。

611 :ご冗談でしょう?名無しさん:2014/07/09(水) 20:06:12.67 ID:???.net
剛体の摩擦力という解釈ではそもそも入らないのでは?

612 :ご冗談でしょう?名無しさん:2014/07/09(水) 20:10:20.22 ID:fOIndAH5.net
ローレンツ変換  分極でググってもぜんぜんそれらしいのがヒットしないんですが・・・
本当に理解してるなら丸投げしないで定性的な説明ができるはずですよね?

613 :ご冗談でしょう?名無しさん:2014/07/09(水) 20:31:44.40 ID:???.net
>>612
簡単に言えば座標変換で磁場が消えれば電場が現れるということ。
詳しくは>>608 の言うように電流・電磁場のローレンツ変換を知らなければ無理。

614 :ご冗談でしょう?名無しさん:2014/07/09(水) 21:18:23.89 ID:???.net
> Maxwellの電磁気論は知ってますが
...。

615 :ご冗談でしょう?名無しさん:2014/07/09(水) 21:56:38.51 ID:???.net
物理にショートカットは無い。

616 :ご冗談でしょう?名無しさん:2014/07/09(水) 22:14:34.50 ID:???.net
>>615
だが数学にまわりこまれた!

617 :ご冗談でしょう?名無しさん:2014/07/09(水) 22:29:07.01 ID:???.net
物理の数学はツールです。数式それ自体に意味はありません。

618 :ご冗談でしょう?名無しさん:2014/07/09(水) 22:48:29.22 ID:???.net
繰り込みは数式ばかりだが

619 :ご冗談でしょう?名無しさん:2014/07/09(水) 23:13:56.89 ID:???.net
ベクトル解析、ファインマンダイアグラム、アインシュタインの縮約あたりはショートカットと言えるんじゃないだろうか。

620 :ご冗談でしょう?名無しさん:2014/07/09(水) 23:45:04.58 ID:???.net
机上の物理なら応用数学とあまり変わらん。

621 :ご冗談でしょう?名無しさん:2014/07/10(木) 01:44:50.41 ID:???.net
NASAに科学者として入局するのと、Googleみたいな会社を興すのはどっちの方が難しいですか?

622 :ご冗談でしょう?名無しさん:2014/07/10(木) 02:05:08.21 ID:???.net
疑問に思ったのですが、宇宙科学の世界と相場の世界はどっちの方が深くて難しいのでしょうか?

623 :ご冗談でしょう?名無しさん:2014/07/10(木) 06:00:55.54 ID:???.net
私の使ってるfdtdのシミュレーションソフトだと単位セル内に空気と完全導体があるとそのセル内の完全導体が無かった事にされちゃうんですがどのソフトでもそうなんですか?

624 :ご冗談でしょう?名無しさん:2014/07/10(木) 07:53:27.88 ID:???.net
空中に目に見えないサイズの鉄粉が数個浮かんでいたからって
それをマクロな電場計算に考慮するのはナンセンスでしょう
とかそういう問題ではないの?

625 :ご冗談でしょう?名無しさん:2014/07/10(木) 09:16:02.40 ID:???.net
俺はショートカット好きだよ 人によるけど

626 :ご冗談でしょう?名無しさん:2014/07/10(木) 10:06:46.14 ID:???.net
物理学科の人は、情報科学などの、人間スケールのことには興味無いですか?

627 :ご冗談でしょう?名無しさん:2014/07/10(木) 10:43:05.81 ID:???.net
>>624
数十マイクロメートル導波管が広がってしまうので無視できるレベルではないです

628 :ご冗談でしょう?名無しさん:2014/07/10(木) 10:59:50.35 ID:???.net
単位セルのサイズを細かくして、かつての1セルサイズ導体を十分なセル数の塊りにしたら意味のある計算ができるようになる(無視されなくなる)のでは?
所詮は連続体を強引に離散化してるんだから1セルだけの導体なんて「無視できる鉄粉」扱いされてもしょうがないでしょう

629 :ご冗談でしょう?名無しさん:2014/07/10(木) 13:44:50.81 ID:???.net
>>625
人間いうなら人間の定義をしてみたら?

630 :ご冗談でしょう?名無しさん:2014/07/10(木) 15:40:42.45 ID:???.net
情報科学が人間スケールとか

631 :ご冗談でしょう?名無しさん:2014/07/10(木) 17:29:12.37 ID:???.net
>>619
ショートカットというよりは地ならしにあたるのでは

632 :ご冗談でしょう?名無しさん:2014/07/10(木) 17:43:50.10 ID:???.net
>>627
それってじゃあセルを細かく区切れよって話では?

633 :ご冗談でしょう?名無しさん:2014/07/10(木) 20:06:22.45 ID:???.net
>>623
ふつう物性値設定するとこで物質の種数とか設定できるんじゃないの?

というかソフトの名前明かしてくれないと質問としてはニッチ過ぎる気がするー。

634 :ご冗談でしょう?名無しさん:2014/07/11(金) 06:38:34.99 ID:74yc05uG.net
ベクトル関数で、虚数解から、虚数解同士で、+の解がでるなら、無から有はできるかな? 

635 :ご冗談でしょう?名無しさん:2014/07/11(金) 06:41:21.29 ID:74yc05uG.net
物理学ベクトル関数で、虚数界から、虚数同士で、+の解がでるなら、無から有はできるかな? 

636 :ご冗談でしょう?名無しさん:2014/07/11(金) 06:47:01.69 ID:74yc05uG.net
ベクトル関数で、虚数解から、虚数解同士で、+の解がでるなら、無から有はできるかな? 

637 :ご冗談でしょう?名無しさん:2014/07/11(金) 20:08:02.08 ID:+YropvQJ.net
ナビアストークス方程式をどんな境界条件でも10秒で解ける方法を発見してしまった

638 :ご冗談でしょう?名無しさん:2014/07/11(金) 20:12:05.86 ID:r0bC4KA6.net
なぁなぁ、流体力学のスレッドってないのん?

639 :ご冗談でしょう?名無しさん:2014/07/11(金) 21:04:51.64 ID:???.net
同種粒子は区別がつかないって話はどういった経緯、理由で原理とされてるか分かる人いますか?

640 :ご冗談でしょう?名無しさん:2014/07/11(金) 21:54:57.88 ID:???.net
ボーズに聞けば

641 :ご冗談でしょう?名無しさん:2014/07/11(金) 22:05:05.09 ID:???.net
ギブズのパラドックス

642 :ご冗談でしょう?名無しさん:2014/07/11(金) 22:16:38.06 ID:???.net
霊界の広さはどのくらいなんでしょうか?

643 :ご冗談でしょう?名無しさん:2014/07/11(金) 22:18:11.29 ID:???.net
霊界に広さなんてあるんですか?

644 :ご冗談でしょう?名無しさん:2014/07/11(金) 22:19:17.37 ID:???.net
無いの?

645 :ご冗談でしょう?名無しさん:2014/07/11(金) 22:20:44.81 ID:???.net
できれば神界の広さも教えてほしいです。

646 :ご冗談でしょう?名無しさん:2014/07/11(金) 23:11:10.65 ID:???.net
大川隆法に聞けよ

次の方どうぞ

647 :ご冗談でしょう?名無しさん:2014/07/11(金) 23:24:53.00 ID:???.net
マイクロソフト社と英国王室はどっちの方が価値が高いでしょうか?

648 :ご冗談でしょう?名無しさん:2014/07/11(金) 23:27:58.51 ID:???.net
>>647
通報されてるぞ、いいのか?

649 :ご冗談でしょう?名無しさん:2014/07/12(土) 00:33:26.28 ID:???.net
世界一カッコイイ都市はニューヨークですか?

650 :ご冗談でしょう?名無しさん:2014/07/12(土) 04:21:59.87 ID:???.net
http://i.imgur.com/OnfxbLp.jpg

鳥だと面倒だからホバー機として考える
底面S1と、ホバー機の下面(下向きベクトルと表面ベクトルの内積が正)とを含んだ上面S2とで挟まれた領域を考える
このとき定常状態だから流出量=流入量
こっから保存式とか云々考えると∫p*dS1=∫p*dS2で、後者はS2に掛かる大気圧とホバー機の下面に掛かる圧力の合計だからつまり大気圧+5g
よって目盛りも25g
----------------------------------------------------------

上の問題で上記の説明とともに箱の天井が開いていようが開いていまいが計りの重さは変わらないという主張があったのですが正しいですか?
自分は実際に天井を開いて計ると少し軽くなると思うってしまいあまり納得できません

651 :ご冗談でしょう?名無しさん:2014/07/12(土) 04:28:23.92 ID:???.net
342 番組の途中ですがアフィサイトへの転載は禁止です sage 2014/07/12(土) 03:16:55.93 ID:yQXIjo7N0
>>331
もうちょっとわかりやすく行ってやるわ
俺がさっき定義した領域では、もちろん空気は出ていくけどその分だけ空気は入ってくる。そうじゃないと真空になっちゃうから
そんである程度の圧力、空気の質量でとどまる。これが定常状態
この領域の空気は一部で出て行ったり入ってきたりしてるけど全体としたら動いていない=運動方程式から垂直水平の力が釣り合ってる
壁は垂直だから空気の粘性を無視するなら垂直の力は生まない
上面では大気圧+ヘリを支えるだけの圧力がかかって、下面でもこれと同じ圧力がかかる
それだけの話

さっきから言ってる気流の力とか風圧が逃げるとかはっきり言って意味不明。イメージで言いたいことは分かるけど全く数学的じゃないから的外れ

443 番組の途中ですがアフィサイトへの転載は禁止です sage 2014/07/12(土) 04:24:26.49 ID:yQXIjo7N0
ごめん>>300 >>346でちょい間違ってたから訂正
定常状態だから上下が釣り合っているってのは正しくなかった

正しくは、
運動量保存式を考えたとき、運動量の変化は上面での流入・流出空気によるものだが、流入量が等しく運動量ベクトルも同じなので合計はゼロ
なので上面下面での圧力積分は等しい
----------------------------------------------------------

後からこのような主張をしています
もし間違えがあるなら理由も付けて教えてもらえると嬉しいです

652 :ご冗談でしょう?名無しさん:2014/07/12(土) 04:32:45.72 ID:???.net
>>650
実際にやってみるしかないんじゃないの、
君に理論的に考える能力はないみたいだし。

653 :ご冗談でしょう?名無しさん:2014/07/12(土) 11:04:45.05 ID:???.net
>>650
変わらないのは第一近似だ
上に空いた箱の中で空気を乱せば、上に出る気流の反動で微小に重くなるはず

654 :ご冗談でしょう?名無しさん:2014/07/12(土) 11:14:55.88 ID:???.net
まったく、これだから困る
ちょっと考えれば25g以上重くなることはないだろと…。

箱を開けていけばどんどん重さを感じなくなる方向なのは
小難しい計算しなくても自明

箱の上のふたを開けた状態なら、手のひらの上で鳥に飛んでもらった時の
手のひらで受ける風よりは重く感じるけど、箱をしめた時よりは少し
軽く感じるかもなくらいだろ

655 :ご冗談でしょう?名無しさん:2014/07/12(土) 11:58:54.57 ID:???.net
回転している地球ゴマを動かそうとすると回転してない時より重く感じるのですが。
これは、回転しているときとしてない時では何の物理量が変わっているのでしょうか?
回転軸と垂直の方向へ動かした時の物理量、平行の方向の動かした時の物理量、回転軸と別の軸へ回転させる時の物理量、等々
これらのどこが変わるのでしょうか?

656 :ご冗談でしょう?名無しさん:2014/07/12(土) 12:13:12.29 ID:???.net
モーメントはもう習った?

657 :ご冗談でしょう?名無しさん:2014/07/12(土) 12:57:32.22 ID:???.net
お好み焼きの話でも思ったが
お前らってなんかこう高圧的だよな

658 :ご冗談でしょう?名無しさん:2014/07/12(土) 12:58:02.46 ID:???.net
本当は詳しく分かってないんじゃないの?

659 :ご冗談でしょう?名無しさん:2014/07/12(土) 12:58:40.67 ID:???.net
お好み焼きで圧かけないだろう

660 :ご冗談でしょう?名無しさん:2014/07/12(土) 12:59:45.32 ID:w9OZYg1U.net
お好み焼きで圧かけたらいかんらしいな
ふっくらしないとかだけど実際早く焼けるんだよな

661 :ご冗談でしょう?名無しさん:2014/07/12(土) 13:04:58.89 ID:???.net
ジャイロの原理をしっかりわかってる人って少なそう

662 :ご冗談でしょう?名無しさん:2014/07/12(土) 13:05:17.88 ID:???.net
まぁ相対性理論なんて1からきちんと説明すると1000レスじゃ足りないしな

↑(どうせ説明出来へん癖に...)

663 :ご冗談でしょう?名無しさん:2014/07/12(土) 13:08:45.59 ID:l+f2xiBc.net
ゴルフボールを打つ時やレシプロ機関のピストンで最大1万G以上って聞いたんだけど
人類が作り出すことのできる加速度の最大はどれくらい?

664 :ご冗談でしょう?名無しさん:2014/07/12(土) 13:11:45.26 ID:???.net
何理論的な話?現実的な話?

665 :ご冗談でしょう?名無しさん:2014/07/12(土) 13:20:59.80 ID:???.net
文章を普通に読むと後者だよね…

666 :ご冗談でしょう?名無しさん:2014/07/12(土) 14:14:33.84 ID:???.net
>>655
角運動量を変えるにはトルクが必要という話なんだが、
角運動量がわからなかったら、回転の向きを変えることで生じる
物体の各点での加速度を考えてみたらわかるんじゃないかな。
これは回転速度が大きければ大きいほど大きくなるから
ちょっと考えてみるといい。

667 :ご冗談でしょう?名無しさん:2014/07/12(土) 15:57:19.97 ID:???.net
F1のエンジンとかになると回転数多いから、ハンドル切りにくいってことか。
とまあネタはおいといて。

モーメントとかもう覚えていないんだよな。物理は高校ぐらいまでしかやってないから。
モーメントだってたしか回転を早くしたり遅くしたりする程度で、
回転軸とは違う方向に回転させるとかはやらなかったはずだし。
完全に知識外だわ。

あと、
>>650みたいな、密閉された物の中に回転するものがあった場合。
中で回転してるか、回転してないかでは物理量は変わってくるよね。てのがほんとの疑問でした。

668 :ご冗談でしょう?名無しさん:2014/07/12(土) 16:34:04.26 ID:???.net
>>667
リアクションホイールでググれ

669 :ご冗談でしょう?名無しさん:2014/07/12(土) 18:59:12.92 ID:???.net
1億pvのサイトって、どれくらいの収入になりますか?

670 :ご冗談でしょう?名無しさん:2014/07/12(土) 19:44:46.04 ID:???.net
>>669
jimさんに聞け

671 :ご冗談でしょう?名無しさん:2014/07/12(土) 22:08:07.19 ID:???.net
>>669
こちらでお待ちしております
ttp://katahiromz.bbs.fc2.com/

672 :ご冗談でしょう?名無しさん:2014/07/12(土) 23:13:08.02 ID:PLdp3XBW.net
せんせー! 質問です!

巨視的物体をテレポートさせる方法を開発
http://news.mynavi.jp/news/2014/07/01/043/

この記事の補足説明をお願いします!
主に、板チョコの様な画像の意味が全くわかりません。

673 :ご冗談でしょう?名無しさん:2014/07/12(土) 23:17:15.77 ID:???.net
はい、君、何がわからないのかな?

674 :ご冗談でしょう?名無しさん:2014/07/12(土) 23:32:21.10 ID:???.net
>>672
ひどい記事だな
論文読めよ

675 :ご冗談でしょう?名無しさん:2014/07/12(土) 23:33:44.24 ID:???.net
人間をBECさせるなんてねえ

676 :ご冗談でしょう?名無しさん:2014/07/12(土) 23:50:44.78 ID:???.net
あの「板チョコ」のような画像は相関を可視化したものなのかな?
元論文読んだ方いらっしゃいます?

677 :ご冗談でしょう?名無しさん:2014/07/12(土) 23:55:16.60 ID:PLdp3XBW.net
>>673 量子もつれから、わかんないっす

相関関係はあるけど、両側の量子の間で何が起こってるのか?
という因果関係は無いのですか?

伝送経路が無いのに、情報が伝わるのは不思議で
仕方ないんですが・・

678 :ご冗談でしょう?名無しさん:2014/07/13(日) 00:28:33.86 ID:???.net
>>677
イーピーアールパラドックスで調べてみて

679 :ご冗談でしょう?名無しさん:2014/07/13(日) 00:41:03.51 ID:???.net
http://homepage3.nifty.com/kuebiko/science/freestdy/buoyancy/img002.gif
横の圧力について下に行けばいくほど大きくなるのは何故ですか?
下の方が水に掛かる重力が大きいのでP_1<P_2は分かりますが横方向については重力は関係ありませんよね?

680 : ◆PHySica/AE :2014/07/13(日) 00:56:21.05 ID:???.net
>>679
水は逃げ場を求めて等方的に力を及ぼすからです

681 :ご冗談でしょう?名無しさん:2014/07/13(日) 01:16:04.90 ID:???.net
>>680
成程
ありがとうございます

682 :ご冗談でしょう?名無しさん:2014/07/13(日) 03:13:15.69 ID:icy1vLt+.net
>>678 あざーっす! 見てみまーっす!

683 :ご冗談でしょう?名無しさん:2014/07/13(日) 09:30:53.90 ID:???.net
テスト。

684 :ご冗談でしょう?名無しさん:2014/07/13(日) 12:14:01.68 ID:???.net
マイクロ波が吸収体に吸収される理由は何ですか。
吸収体の原理がよくわからないです

685 :ご冗談でしょう?名無しさん:2014/07/13(日) 12:16:22.45 ID:???.net
回転します

686 :ご冗談でしょう?名無しさん:2014/07/13(日) 12:24:24.79 ID:???.net
例えば水分子は酸素が負に、水素が正に分極している。
ここにマイクロ波を当てると、水分子に振動する力が加わる。
この結果として水分子は回転し、分極したものが回転するので
電磁波を発する。これがもとのマイクロ波を打ち消し、
マイクロ波を減衰させる。勿論減衰した分のエネルギーは
水分子が受けとるので、水の温度は上昇する。

687 :ご冗談でしょう?名無しさん:2014/07/13(日) 13:07:58.07 ID:???.net
>>677
因果律は無いし、それだけで情報伝達ができない。
簡単な理由は観測者が2状態の0と1を制御できないから。

688 :ご冗談でしょう?名無しさん:2014/07/13(日) 13:08:11.49 ID:???.net
なぜビッグバンが起こったのでしょうか?

689 :ご冗談でしょう?名無しさん:2014/07/13(日) 13:17:51.84 ID:???.net
>>688
先ず科学には繰返し実験・検証が出来ない現象もあると知るべきだ。

690 :ご冗談でしょう?名無しさん:2014/07/13(日) 13:40:05.54 ID:???.net
そもそも「何故」を扱うのは科学ではない。

691 :ご冗談でしょう?名無しさん:2014/07/13(日) 13:54:54.68 ID:???.net
うわぁ...


「何故」を扱うのが科学ではない(キリッ

692 :ご冗談でしょう?名無しさん:2014/07/13(日) 14:14:46.44 ID:???.net
ヒント:
「なぜ何もないのではなく、何かがあるのか?」wikipedia

693 :ご冗談でしょう?名無しさん:2014/07/13(日) 14:26:57.22 ID:???.net
宇宙を考察するのに科学は使えない

694 :ご冗談でしょう?名無しさん:2014/07/13(日) 15:10:33.14 ID:???.net
>>692
それが科学だとでも?

695 :ご冗談でしょう?名無しさん:2014/07/13(日) 15:26:44.48 ID:???.net
>>694
これは科学じゃないだろあほかw
ちゃんと読めよ

696 :ご冗談でしょう?名無しさん:2014/07/13(日) 16:20:53.83 ID:???.net
>>692
きみだったら、こういう奴がいったい何を主張せんとしてるかわかるのだろうか?
http://wc2014.2ch.net/test/read.cgi/galileo/1405233675/1

697 :ご冗談でしょう?名無しさん:2014/07/13(日) 18:14:34.20 ID:???.net
ジャンプして着地するまでの位置を1秒毎に求めたいんですが教えてください
風は吹いてなくて重力は地球位とお考えください

698 :ご冗談でしょう?名無しさん:2014/07/13(日) 19:07:41.99 ID:???.net
意味がわからん

699 :ご冗談でしょう?名無しさん:2014/07/13(日) 19:34:28.26 ID:???.net
結局、なぜビッグバンが起こったのか分からないのか。
残念。

700 :ご冗談でしょう?名無しさん:2014/07/13(日) 20:15:31.61 ID:???.net
ここの連中と来たら分からないことすぐそらそうとするからな

701 :ご冗談でしょう?名無しさん:2014/07/13(日) 20:20:11.43 ID:???.net
なんでも聞けばいいのは厨房まで

702 :ご冗談でしょう?名無しさん:2014/07/13(日) 20:50:18.66 ID:???.net
汝語り得ぬ事柄については口を閉ざせ

703 :ご冗談でしょう?名無しさん:2014/07/13(日) 20:53:34.12 ID:???.net
なんじとなんじはいまなんじ

704 :ご冗談でしょう?名無しさん:2014/07/13(日) 21:07:40.03 ID:???.net
第一原因は存在するのでしょうか?

705 :ご冗談でしょう?名無しさん:2014/07/13(日) 21:24:19.61 ID:IoGKYN1a.net
平均値の定理を使って重積分の変数変換
dS=|r_u×r_v|dudv
を証明できますか?

706 :ご冗談でしょう?名無しさん:2014/07/13(日) 21:25:51.21 ID:???.net
どうだろうね

707 :ご冗談でしょう?名無しさん:2014/07/13(日) 21:26:14.97 ID:???.net
またあんたか

708 :ご冗談でしょう?名無しさん:2014/07/13(日) 21:29:47.64 ID:???.net
人間が想像できることはすべてできるよ

709 :ご冗談でしょう?名無しさん:2014/07/13(日) 21:31:14.63 ID:IoGKYN1a.net
またあんたかってこのスレはじめてなんですが・・・

710 :ご冗談でしょう?名無しさん:2014/07/13(日) 21:35:51.83 ID:???.net
>>709
その上の基地外に対するレスだと思うよ

711 :ご冗談でしょう?名無しさん:2014/07/13(日) 21:43:01.84 ID:???.net
くやしいのう

712 :ご冗談でしょう?名無しさん:2014/07/13(日) 22:36:21.87 ID:???.net
ヒマヤラ

713 :ご冗談でしょう?名無しさん:2014/07/13(日) 22:37:28.52 ID:???.net
ヒマラヤ

714 :ご冗談でしょう?名無しさん:2014/07/13(日) 23:28:58.73 ID:???.net
あたーり

715 :ご冗談でしょう?名無しさん:2014/07/13(日) 23:34:39.73 ID:???.net
接続って物理学では何に対応しますか

716 :ご冗談でしょう?名無しさん:2014/07/13(日) 23:40:59.65 ID:???.net
ゲージ理論

717 :ご冗談でしょう?名無しさん:2014/07/13(日) 23:49:50.43 ID:???.net
もっと詳しくゲージ理論と接続の関係を論じていただけないでしょうか

718 :ご冗談でしょう?名無しさん:2014/07/13(日) 23:55:42.67 ID:???.net
まず君の理解を述べるのが先であろう。

719 :ご冗談でしょう?名無しさん:2014/07/13(日) 23:59:09.45 ID:???.net
>>717
23行では説明しきれない

720 :ご冗談でしょう?名無しさん:2014/07/14(月) 11:28:02.53 ID:???.net
ゲージ理論⇔ベクトル束
ゲージ場⇔切断
場のつながり⇔接続

721 :ご冗談でしょう?名無しさん:2014/07/14(月) 13:35:03.30 ID:???.net
横だけど観測できんの?

722 :ご冗談でしょう?名無しさん:2014/07/14(月) 18:56:26.25 ID:???.net
>>679
この説明だと、海底の物体には浮力が働かないってことにならないか?

723 :ご冗談でしょう?名無しさん:2014/07/14(月) 19:11:07.89 ID:???.net
>>722
海底にぴったり着床した潜水艦が浮き上がれなくなったことがあるらしい

724 :ご冗談でしょう?名無しさん:2014/07/14(月) 19:33:45.91 ID:???.net
>>722
そりゃあ吸盤で海底にはりつくような状況ならね。

725 :ご冗談でしょう?名無しさん:2014/07/14(月) 20:05:57.35 ID:nQeyUvTb.net
ぴったりはりついたところで圧力は全方向一定だと思うが
水圧がただの垂直抗力になっただけで

726 :ご冗談でしょう?名無しさん:2014/07/14(月) 20:23:19.81 ID:???.net
空っぽの風呂底に洗面器を押し付けたままお湯を張っていってみ、
手を放しても洗面器は浮き上がってこない

727 :ご冗談でしょう?名無しさん:2014/07/14(月) 20:52:49.30 ID:y3Sxwd7C.net
小麦粉、片栗粉、白砂糖と寒天粉、その上に白砂糖を入れて、かき混ぜる。聖水を入れてかき混ぜる 中級聖精水 登記

小麦粉と聖水、聖水と片栗粉、白砂糖と聖水寒天粉、その上に白砂糖を入れて、かき混ぜる。上級聖精水 登記

728 :ご冗談でしょう?名無しさん:2014/07/14(月) 20:53:42.65 ID:???.net
海底は滑らかな面じゃないから水が下に回りこんで浮力が生じるっていう理解でいいの?
>>723によると、潜水艦スケールでは浮力の低下が生じることもあるみたいだけど。

729 :ご冗談でしょう?名無しさん:2014/07/14(月) 20:54:31.72 ID:y3Sxwd7C.net
小麦粉、片栗粉、白砂糖と寒天粉、その上に白砂糖を入れて、かき混ぜる。聖水を入れてかき混ぜる 中級聖精水 登記

小麦粉と聖水、聖水と片栗粉、白砂糖と聖水寒天粉、その上に白砂糖を入れて、かき混ぜる。上級聖精水 登記

聖精水と、プロトタイプワクチン(ポッカのグリーンのキレイトでも良い)で、初期解毒水

730 :ご冗談でしょう?名無しさん:2014/07/14(月) 20:54:56.18 ID:???.net
潜水艦くらい巨大で重いと、ある意味で吸盤のような働きが生じるっていうのは納得できる。

731 :ご冗談でしょう?名無しさん:2014/07/14(月) 20:55:54.06 ID:y3Sxwd7C.net
小麦粉、片栗粉、白砂糖と寒天粉、その上に白砂糖を入れて、かき混ぜる。聖水を入れてかき混ぜる 中級聖精水 登記

小麦粉と聖水、聖水と片栗粉、白砂糖と聖水寒天粉、その上に白砂糖を入れて、かき混ぜる。上級聖精水 登記

聖精水と、プロトタイプワクチン(ポッカのグリーンのキレイトでも良い)で、初期解毒水 登記

732 :ご冗談でしょう?名無しさん:2014/07/14(月) 21:18:57.77 ID:nQeyUvTb.net
脳ってアナログ回路なのになんでコンピュータより複雑な計算ができるの

733 :ご冗談でしょう?名無しさん:2014/07/14(月) 21:20:04.19 ID:???.net
コンピューターの方が寄せ集めの力押しなんですよ

734 :ご冗談でしょう?名無しさん:2014/07/14(月) 21:25:06.59 ID:nQeyUvTb.net
しょせん脳も物理現象なんだから同じように「知覚」を再現することはできないのかな
フィードバック制御みたいなものでしょ

735 :ご冗談でしょう?名無しさん:2014/07/14(月) 21:27:50.02 ID:nQeyUvTb.net
やっぱこれからの時代NANDゲートつなげるだけじゃなにも進歩しない気がする
ここ100年1000年でなにが発展するかと言えばアナログ回路が発達すると思う

736 :ご冗談でしょう?名無しさん:2014/07/14(月) 21:33:55.63 ID:???.net
>>732
脳ってのはうまく行くような回路が出来るように組み変わってくからね。
一般的なコンピュータプログラムというのは作る人が把握出来る以上に複雑にならない。
脳の働きを模倣するコンピュータプログラムもあるんだけどね。

737 :ご冗談でしょう?名無しさん:2014/07/14(月) 21:40:03.09 ID:???.net
>>735
キミの脳は算数をアナログ(実数)で計算するのか、NANDもアナログ素子で出来てるぞ。

738 :ご冗談でしょう?名無しさん:2014/07/14(月) 23:16:01.03 ID:9UjLRm+p.net
【開いた口が塞がらない疑問その1】:>>514さんの、「ハイ、今ここの交差点の信号器の色は赤?白?黒?」
みたいなレベルの問題…つか疑問に、ここの誰一人としてちゃんと答えてやらないのはいったい何故?

【開いた口が塞がらない疑問その2】:>>513さんの、
>たぶん 12Ω抵抗の誤植でしょう
…という、あんたよっぽど目が悪いのかそれともバカァ?なボケ回答に誰一人として
ツッコまないのはいったい何故?

>大学の教科書だってそれくらいの間違いはよくある事です
いんや教科書じゃなくって間違ってるのはアンタの頭か脳ミソおよび眼ン球でしょう。

【開いた口が塞がらない疑問その3】:>>514さんの(以下同文

#三人が三人とも、物理学的には物凄いこと言っちゃってるんだけど?w

739 :ご冗談でしょう?名無しさん:2014/07/14(月) 23:22:13.60 ID:???.net
>>722
底に水が入らない状況ならそう。
空気の入らないようぴったり床につけた紙がなかなか剥がせないのも同じ事。

740 :ご冗談でしょう?名無しさん:2014/07/14(月) 23:40:39.51 ID:???.net
>>738
おまえが塞げよ

741 :ご冗談でしょう?名無しさん:2014/07/14(月) 23:50:34.84 ID:???.net
>>738
お前がフォローしろ

742 :ご冗談でしょう?名無しさん:2014/07/14(月) 23:58:06.06 ID:???.net
いや垂直抗力で極限に収束するよ

743 :ご冗談でしょう?名無しさん:2014/07/15(火) 00:13:19.25 ID:zDLqkQOq.net
>>740,>>741
いやコレぶっちゃけ、コレがわからんのならもお今すぐ退学か辞職勧告
ってレベルちがう?

744 :ご冗談でしょう?名無しさん:2014/07/15(火) 00:14:46.60 ID:???.net
>>743
だから、おまえが解答すればいいんだよ

745 :ご冗談でしょう?名無しさん:2014/07/15(火) 00:40:55.03 ID:zDLqkQOq.net
よし来た、あのネ、ボクが自転車で走ってたら工事中で砂利だらけの道と、
キレイに舗装された道と、中くらいだけどキレイな女子高校生が足を開いて
ツンパ丸見え〜な…

746 :ご冗談でしょう?名無しさん:2014/07/15(火) 00:47:29.31 ID:???.net
OK充分だ
それは200も前のレスをぶり返して今頃アピールするようなネタじゃない

747 :ご冗談でしょう?名無しさん:2014/07/15(火) 01:29:22.70 ID:???.net
http://i.imgur.com/sKR3EoH.jpg

こちらの問題 課題1に取り組んでいるのですが、解答(といっても与えられていないのですが)にたどり着くことができません。

現在はcos部分をRe[exp(i ω_0 t)exp(i nΔωt)exp(i φ_0)]としてnに関係しない部分を総和記号の前に出して...という方針で計算してみてはいるのですが、いまひとつすっきりと解けていないような感じがします。
どのような方針で解いていけばよいのでしょうか。ヒント程度でも教えていただけると大変助かります。よろしくお願いします。

748 :ご冗談でしょう?名無しさん:2014/07/15(火) 01:33:40.10 ID:giT8tajs.net
人間も宇宙も同じことをやっています
あなたは信じますか?
証明は数学でできますが、非常に長いものになります。
いくつかの領域については新規の学問領域の開拓となるので
私には無理かもしれません
だれか やりませんか?

749 :ご冗談でしょう?名無しさん:2014/07/15(火) 02:38:34.68 ID:???.net
>>747
等比数列の和

750 :ご冗談でしょう?名無しさん:2014/07/15(火) 03:27:15.15 ID:???.net
>>749
和の部分が分数に指数に変数にと入り乱れたような結果が導かれているのですが、そんなものでしょうか。

751 :ご冗談でしょう?名無しさん:2014/07/15(火) 03:34:59.55 ID:???.net
>>750
>cos部分をRe[exp(i ω_0 t)exp(i nΔωt)exp(i φ_0)]としてnに関係しない部分を総和記号の前に出し
たら総和の中身は q^n exp(i nΔωt) だけじゃないの?

752 :ご冗談でしょう?名無しさん:2014/07/15(火) 07:23:41.91 ID:???.net
>>748
いたち
http://anago.2ch.net/test/read.cgi/river/1393854627/

753 :ご冗談でしょう?名無しさん:2014/07/15(火) 08:56:06.57 ID:???.net
巨大タンカーの船長の年収はどのくらいなんでしょうか?

754 :ご冗談でしょう?名無しさん:2014/07/15(火) 09:25:21.29 ID:???.net
>>753
http://anago.2ch.net/test/read.cgi/tubo/1343379513/162,163

755 :ご冗談でしょう?名無しさん:2014/07/15(火) 13:58:59.99 ID:???.net
>>751
総和記号を展開することを考えたら、結果が〜ということです。
そのような結果になるのであればそもそも展開の必要がないのでしょうか

756 :ご冗談でしょう?名無しさん:2014/07/15(火) 14:34:24.75 ID:???.net
磁石をスチール棚に長期間ひっつけたままだと、
棚が磁化してだんだんひっつき力が減ってきますか。

地震対策のため、強力磁石で止めてあるが、大丈夫か心配になっって来ました。

757 :ご冗談でしょう?名無しさん:2014/07/15(火) 14:54:18.80 ID:???.net
良い質問です

758 :ご冗談でしょう?名無しさん:2014/07/15(火) 18:42:23.81 ID:???.net
>>756
磁化したらむしろ強くなるんじゃね??

759 :ご冗談でしょう?名無しさん:2014/07/15(火) 20:10:27.46 ID:0k4IBs2U.net
海抜0メートル付近の低地で
魔法瓶に沸騰したお湯を入れて
しっかりフタをして、ヘリコプターで
すみやかに富士山の頂上に行ったら
どうなりますか?

760 :ご冗談でしょう?名無しさん:2014/07/15(火) 20:33:47.89 ID:???.net
そりゃ素晴らしい景色が見れるでしょうね

761 :ご冗談でしょう?名無しさん:2014/07/15(火) 20:37:48.94 ID:???.net
お金がかかります

762 :ご冗談でしょう?名無しさん:2014/07/15(火) 20:40:02.23 ID:???.net
魔法瓶の耐圧性能は信じるとして
フタを開けたとたんに水蒸気爆発するんじゃないかな・・・

763 :ご冗談でしょう?名無しさん:2014/07/15(火) 20:47:24.05 ID:???.net
魔法瓶のフタを閉めた時点で中は1気圧ですよ。
爆発するわけないでしょう。1気圧の水蒸気が出てくるだけです。

764 :ご冗談でしょう?名無しさん:2014/07/15(火) 21:17:29.16 ID:???.net
民間人がヘリコプターで富士山頂上へ行くのは禁止されているはず。

765 :ご冗談でしょう?名無しさん:2014/07/15(火) 21:22:09.49 ID:???.net
速やかに深海へ持って行ったのと逆の現象が起きます

766 :ご冗談でしょう?名無しさん:2014/07/15(火) 21:55:16.55 ID:???.net
>>755
どう計算してどうなったのか詳しく書いてみ

767 :ご冗談でしょう?名無しさん:2014/07/15(火) 22:01:16.11 ID:???.net
>>763
詰め切れてなかった気体部分はそうだけど
高地(低圧下)での沸点を10度ほど越えてる水を開放するわけですよ

768 :ご冗談でしょう?名無しさん:2014/07/15(火) 22:12:06.28 ID:St8oidex.net
そりゃ飽和蒸気圧≫大気圧になるんだから
ちょっとした振動で突沸を起こして水蒸気爆発するだろ
ボイラー事故みたいに

769 :ご冗談でしょう?名無しさん:2014/07/15(火) 22:46:11.01 ID:???.net
>>767
フタを閉めた時点で1気圧は1気圧です。
フタを開けて中の気圧が下がれば徐々に沸騰しますが、そのとき1気圧以上の蒸気圧で出てくることはありません。

>>768
飽和蒸気圧(1気圧) > 大気圧
ですから魔法瓶から空気抜ける前に持っていけばいちおう開けた瞬間は出てきますよ、普通に1気圧以下の圧力で。
そんなもの爆発とは程遠いですが。

770 :ご冗談でしょう?名無しさん:2014/07/15(火) 22:54:14.41 ID:???.net
手法を変えてきたか

771 :ご冗談でしょう?名無しさん:2014/07/15(火) 22:58:20.27 ID:St8oidex.net
爆発ってほどにはならないかもしれないけど
突沸はするだろ

772 :ご冗談でしょう?名無しさん:2014/07/15(火) 22:59:19.96 ID:???.net
>>762
突沸というのは体積は爆発的に増えるけど圧力が爆発的に高くなる訳じゃあないよ

773 :ご冗談でしょう?名無しさん:2014/07/15(火) 23:03:13.25 ID:St8oidex.net
結局膨張速度がどれくらいかって話だろ

774 :ご冗談でしょう?名無しさん:2014/07/15(火) 23:03:21.67 ID:???.net
要するに笛吹ケトル以上のことにはならねえな

775 :ご冗談でしょう?名無しさん:2014/07/15(火) 23:05:59.30 ID:St8oidex.net
ガス爆発みたいな連鎖反応とはわけが違うから
たいしたことないだろうな
開けた瞬間蓋はふっとぶかもしれないけど

776 :ご冗談でしょう?名無しさん:2014/07/15(火) 23:08:15.87 ID:???.net
ピシャっと一瞬で霧状に振りまけるならそうかもしれんけど
容器にある程度囲まれた状態だと圧は高まるよ。
不良品の圧力鍋で事故ったのとほぼ同じ状況だ

777 :ご冗談でしょう?名無しさん:2014/07/15(火) 23:09:31.20 ID:???.net
>>776
嘘をこけ嘘を。
圧力鍋の事故は圧力が開放されてるだけで圧力が上がってる訳じゃない

778 :ご冗談でしょう?名無しさん:2014/07/15(火) 23:12:30.11 ID:???.net
突沸の問題はむしろ圧力が「あまり下がらない」ことにあるのであって、
最初の与圧以上の圧力に達するわけがないだろうが。

ただまあ瓶の底の方で突沸すると湯がこぼれて事故ることはあるだろうね

779 :ご冗談でしょう?名無しさん:2014/07/15(火) 23:24:51.42 ID:???.net
>>766
http://i.imgur.com/CfsxJiW.jpg

こんな感じで進めました。
ただ計算しただけというような状況です。

780 :ご冗談でしょう?名無しさん:2014/07/16(水) 00:04:18.22 ID:???.net
>>779
qって各モードの重みでしょ?
q^|n|では
計算出来るけど

781 :ご冗談でしょう?名無しさん:2014/07/16(水) 00:21:24.43 ID:???.net
>>780
ただこの問題を投げられており、知識の土台がほぼない状況です。
q^|n|の意味や、適用のしかたを教えていただけると助かります。

782 :ご冗談でしょう?名無しさん:2014/07/16(水) 00:28:51.43 ID:???.net
>>781
とりあえず複素数の扱いを復習

783 :ご冗談でしょう?名無しさん:2014/07/16(水) 00:34:54.76 ID:???.net
学部4年です。超伝導についてわからないことがあるので質問させてください。
超伝導体に電流が流れているとき、伝導バンドはEfより下のバンドでクーパー対が電荷を運んでいるという認識で合ってますか?
外部電場が0のときは格電子が反対方向に動いて対を形成しているようですが、電場を加えるとその運動量に差が生まれてトータルで電荷が流れるということでしょうか?
まったく的外れなことを言っていたらすみません。

784 :ご冗談でしょう?名無しさん:2014/07/16(水) 00:40:33.90 ID:???.net
テスト。

785 :ご冗談でしょう?名無しさん:2014/07/16(水) 00:41:49.77 ID:???.net
>>782
実部は偏角の正負に関係なく同値、ということでしょうか。

786 :ご冗談でしょう?名無しさん:2014/07/16(水) 00:57:11.69 ID:???.net
公比の2N+1乗に打ち込みミス発見しました
そこは訂正するとして、相変わらず計算が進められません..

787 :ご冗談でしょう?名無しさん:2014/07/16(水) 01:02:44.12 ID:???.net
宇宙船のパイロットになりたいのですが、どうすれば良いでしょうか?

788 :ご冗談でしょう?名無しさん:2014/07/16(水) 01:05:12.09 ID:???.net
とりあえずその質問を英語で書けるようになって下さい

789 :ご冗談でしょう?名無しさん:2014/07/16(水) 01:07:09.92 ID:???.net
>>787
急ぎならロシアか中国の空軍にどうぞ。

790 :ご冗談でしょう?名無しさん:2014/07/16(水) 01:13:07.99 ID:???.net
ロシアか中国の空軍に入ったら宇宙船のパイロットになれるのですか?

791 :ご冗談でしょう?名無しさん:2014/07/16(水) 01:39:08.84 ID:oxNf+N1i.net
例えばだけど若田さんの経歴をさらってみるのはどう?

792 :ご冗談でしょう?名無しさん:2014/07/16(水) 01:40:23.24 ID:oxNf+N1i.net
どんな大学のどんな学科出てるんだろう
どんな資格を持っているんだろう
先人に学ぶといいと思います

793 :ご冗談でしょう?名無しさん:2014/07/16(水) 01:44:29.97 ID:???.net
ホリエモンの人体実験に死癌しればそれで宇宙飛行死

794 :ご冗談でしょう?名無しさん:2014/07/16(水) 02:05:30.10 ID:???.net
100億くらいあればなれるだろう

795 :ご冗談でしょう?名無しさん:2014/07/16(水) 11:41:48.72 ID:2kt3OsPl.net
>>788
I'm thinking to become an astronaut, paticulary the pilot of a spaceship. How can I be ?

796 :ご冗談でしょう?名無しさん:2014/07/16(水) 12:48:08.61 ID:???.net
>>793
堀江の?知らんな「なつのロケット団」のなら知ってる。こっちはまだ先は長いがいざ有人となれば志願者は多そうだ。例え事故が起きてもな

797 :ご冗談でしょう?名無しさん:2014/07/16(水) 12:50:21.29 ID:???.net
ヒマラヤに釣られるアホがいっぱい

798 :ご冗談でしょう?名無しさん:2014/07/16(水) 15:02:54.26 ID:???.net
84×49

倍  半分(端数略)
◎84  49
168 24
336 12
672  6
◎1344 3
◎2688 1

◎を足す
=4116

これ何やってるか分かる?

799 :ご冗談でしょう?名無しさん:2014/07/16(水) 15:13:18.19 ID:???.net
左側を倍倍、右側を1/2倍ずつしていって右側が奇数の時の左側の数字を足す

800 :ご冗談でしょう?名無しさん:2014/07/16(水) 15:24:06.31 ID:???.net
ああそっか
84×49=84×1+84×48=84×1+168×24
だから
84×49=84×1+168×24=84×1+336×12=84×1+672×6=84×1+1344×3=84×1+1344×1+1344×2
=84×1+1344×1+2688×1
ってことかなるほどね

801 :ご冗談でしょう?名無しさん:2014/07/16(水) 15:38:25.25 ID:???.net
>>798
CPUの2進の掛算と同じことを10進でやってるだけだろ
違いは計算時間が千億倍もかかってることくらい

802 :ご冗談でしょう?名無しさん:2014/07/16(水) 15:53:46.60 ID:???.net
>>801
正解
例えば情報工だと知識として知ってる人も多いようだけど、知らないと少し考えちゃうよね

803 :ご冗談でしょう?名無しさん:2014/07/16(水) 17:13:20.35 ID:???.net
珍しく面白かった
物理ではないが

804 :ご冗談でしょう?名無しさん:2014/07/16(水) 20:44:21.49 ID:???.net
この英文訳してくれ

Use Nyquist diagram to find conditions on K() such that the following feedback-loop system with the loop transfer function.
※loop transfer function=一巡伝達関数

such thatとwithがわからん

805 :ご冗談でしょう?名無しさん:2014/07/16(水) 20:45:31.17 ID:???.net
日本語の本にしたら

806 :ご冗談でしょう?名無しさん:2014/07/16(水) 20:49:44.62 ID:???.net
>>805
問題の意味はなんとなくわかるんだけど正確に訳すとどうなるのか教えてほしいです

807 :ご冗談でしょう?名無しさん:2014/07/16(水) 20:54:47.91 ID:???.net
such that 〜
〜であるような

with 〜
〜を持つ

808 :ご冗談でしょう?名無しさん:2014/07/16(水) 20:55:12.63 ID:???.net
高校生レベルだろ

809 :ご冗談でしょう?名無しさん:2014/07/16(水) 21:03:31.93 ID:???.net
翻訳ページでもそれなりに訳してくれる

810 :ご冗談でしょう?名無しさん:2014/07/16(水) 21:11:41.01 ID:???.net
http://i.imgur.com/rM0Z1Gt.jpg
この回路のImで表した消費電力を求めるのに、インピーダンスをZとして|i|/|v|=|Im|/|Vm|=1/|Z|から|Vm|を|Im|を用いて表すみたいですが、なぜ|i|/|v|=|Im|/|Vm|が成り立つのですか?
|cosωt|/|cos(ωt+φ)|は常に1なのですか?

811 :ご冗談でしょう?名無しさん:2014/07/16(水) 21:25:50.53 ID:???.net
インピーダンスは複素数

812 :ご冗談でしょう?名無しさん:2014/07/16(水) 21:58:42.08 ID:???.net
>>810
複素インピーダンスを使うときは、cosωtの代わりにexpiωtをつかう。
するってーと|expiωt|=1だからそれらの式が成り立つことがわかる。

813 :ご冗談でしょう?名無しさん:2014/07/16(水) 22:06:26.17 ID:???.net
虚数単位jの方がよかったな…

因みにexpjωtをつかうと、I(t)=Imcos(ωt+φ)の代わりに
I(t)=Imexpj(ωt+φ)=Imexpjφexpjωt=iexpjωt
のように、係数i=Imexpjφに位相ズレの情報まで押し込めるのでべんりなのよ。

814 :ご冗談でしょう?名無しさん:2014/07/16(水) 22:19:09.84 ID:???.net
>>812-808
納得です
ありがとうございます

815 :ご冗談でしょう?名無しさん:2014/07/16(水) 22:26:16.36 ID:???.net
熱力学の第二法則って人間の活動にも及ぶんですか?

816 :ご冗談でしょう?名無しさん:2014/07/16(水) 22:34:52.57 ID:???.net
>>810
追加の質問で
sinωtもexpjωtに置き換えていいのですか?

817 :ご冗談でしょう?名無しさん:2014/07/16(水) 22:35:13.27 ID:8pHinMj4.net
電気回路のベクトル図ってもし位相差が直角(π/2)じゃないときは使えないの?

818 :ご冗談でしょう?名無しさん:2014/07/16(水) 22:44:03.57 ID:???.net
大型石油タンカーの船長とLNGタンカーの船長の年収はどのくらいなんでしょうか?

819 :ご冗談でしょう?名無しさん:2014/07/16(水) 22:53:18.13 ID:8pHinMj4.net
ってかそもそもなぜ交流回路の電圧や電流やリアクタンスがベクトルのように考えることができるのか理解できない

820 :ご冗談でしょう?名無しさん:2014/07/16(水) 23:01:35.98 ID:8pHinMj4.net
sinに乗ってる項Aとcosに乗ってる項Bとでわけられて
そのまま計算するとたとえばインダクタンスに√A^2+B^2という形が出てくるから
ベクトルみたいに考えるのかなと思ったんですが

821 :ご冗談でしょう?名無しさん:2014/07/16(水) 23:10:28.37 ID:???.net
教科書だか講義ノートは手元にないのか

822 :ご冗談でしょう?名無しさん:2014/07/16(水) 23:20:48.80 ID:???.net
>>816
cosを置き換えるのかsinを置き換えるのか決めておけばどちらを置き換えてもよい。
まあでもexpjωt=cosωt+jsinωtだから、普通はcosを置き換えて、実部を取れば置き換え前に戻るようにする。
従ってcosωtをexpjωtに置き換えるのであればsinωtは-jexpjωt=-jcosωt+sinωtに置き換わる。

823 :ご冗談でしょう?名無しさん:2014/07/16(水) 23:21:19.41 ID:8pHinMj4.net
持ってない うち機械系やし

824 :ご冗談でしょう?名無しさん:2014/07/16(水) 23:22:47.68 ID:???.net
>>820
そのように考えてもいいが、ベクトル平面で考えるより複素数平面で考える方が利点が多い。

825 :ご冗談でしょう?名無しさん:2014/07/16(水) 23:27:09.76 ID:???.net
>>820
例えばベクトルの除算を定義するのには問題が多いが、複素数の除算は自然に定義される。

826 :ご冗談でしょう?名無しさん:2014/07/16(水) 23:41:24.10 ID:8pHinMj4.net
>>824
ベクトル平面からガウス平面への拡張はわかるんだけど

827 :ご冗談でしょう?名無しさん:2014/07/16(水) 23:48:19.18 ID:wPyZIFlx.net
高校物理の質問です 
高校三年 物理は学習済みです

問題文
http://www.dotup.org/uploda/www.dotup.org5188285.jpg
疑問に思った部分の解説
http://www.dotup.org/uploda/www.dotup.org5188300.jpg
この解説で
Mα=Mg−(物体Aの遠心力)
という式がありますが、なぜ
(M+m)α=・・・
ではないのでしょうか?
Mα=Mg−(糸の張力)

mα=(糸の張力)ー(物体Aの遠心力)
の連立で解くとそうなるはずなんですが・・・

振動開始後、Aの回転運動の角速度ω_A Bの振動の角速度ω_B(求める答え)として、

ω_B^2=3Mg/{(m+M)r} ※(M+m)α=・・・の式から導き出される答え
ω_A^2〜Mg/mr ※最初の釣り合いと、最大変位a<<rより最初のAの角速度が振動開始後のω_A程度であることから立式
と(ω_B/ω_A)^2≒0からm/M≒0を導き、
ω_B^2=3Mg/{(m+M)Rr}≒3g/r

このように解説を解釈するのが妥当でしょうか?
よろしくお願いします

828 :ご冗談でしょう?名無しさん:2014/07/16(水) 23:49:19.85 ID:???.net
交流信号の周波数は線形素子によって変化しないので、変化するのは振幅と位相。
時間依存成分をexpjωtで表してしまえばその二つはこの関数の係数で表せる。
そこで係数のみを考えることにすると、これは複素数平面上の点と考えることが出来る。
よって交流信号の線形素子による変化は複素数平面上の点により完全に考察することが出来る。

829 :ご冗談でしょう?名無しさん:2014/07/16(水) 23:52:27.75 ID:???.net
>>827
あっちのスレは荒れちゃったねえ。おれも原因の一端を担っちゃったかもしれないが。
その問題は色々と高校の範囲で考えるには無理があるから適当な理解でとどめた方がいいと思うよ。

830 :ご冗談でしょう?名無しさん:2014/07/16(水) 23:55:00.10 ID:???.net
>>822
度々ありがとうございました

831 :ご冗談でしょう?名無しさん:2014/07/16(水) 23:55:07.64 ID:wPyZIFlx.net
>>829
解説は結構ですので、僕の考え方が概ね合ってるかどうか、
Mαとすべきなのか(M+m)αとすべきなのかだけでも教えていただけませんか

832 :ご冗談でしょう?名無しさん:2014/07/17(木) 00:19:44.44 ID:???.net
(M+m)α=が厳密だが近似すれば結果としてMα=の式が出る
あとマルチは死ね

833 :ご冗談でしょう?名無しさん:2014/07/17(木) 00:28:20.09 ID:???.net
>>832
ありがとうございます
マルチ失礼しました

834 :ご冗談でしょう?名無しさん:2014/07/17(木) 00:46:55.33 ID:???.net
>>833
しっかりやろうとするとAを極座標表示して運動方程式考えたほうがいい
Bが動く時Aのαはベクトル量でしょ

835 :ご冗談でしょう?名無しさん:2014/07/17(木) 01:59:55.45 ID:???.net
宇宙を冒険したいのですが、今現在の技術では、宇宙を冒険することはできないのでしょうか?

836 :ご冗談でしょう?名無しさん:2014/07/17(木) 07:29:52.59 ID:???.net
東京大学理学部情報科学科を目指しているのですが、この学科は講義でプログラミングをやったりするのでしょうか?
プログラミングがまったくできないのですが、先にどこかで学んだほうが良いのでしょうか?
東京大学理学部情報科学科は、プログラミングができないとやっていけいない場所でしょうか?
教えてください。

837 :ご冗談でしょう?名無しさん:2014/07/17(木) 08:54:31.22 ID:???.net
>>836
巣から出てくるな
http://peace.2ch.net/test/read.cgi/gender/1393296640/

838 :ご冗談でしょう?名無しさん:2014/07/17(木) 09:00:09.58 ID:???.net
>>835
今の技術だと月の有人探査しか実現できていないよ
将来的には火星や他の惑星に行けるようになると思う
「冒険」というくらい自由度高いことが出来るようになるのは何世紀も先じゃないかな
そして人類が太陽系外への恒星へ到達可能になるのはさらに遠い未来になると思うよ

839 :ご冗談でしょう?名無しさん:2014/07/17(木) 09:18:05.78 ID:???.net
正直先の2進数のかけ算の意味が分からなかった香具師いる?
仕組みがわからん(汗)ヤバいのかな

840 :ご冗談でしょう?名無しさん:2014/07/17(木) 10:22:52.20 ID:???.net
>>839
2進数との関連は俺もわからん。

841 :ご冗談でしょう?名無しさん:2014/07/17(木) 10:39:26.52 ID:???.net
2進法だと2倍は0を書き足して桁を上げるだけ、2で割るのは桁を下げるだけだから簡単ってことなのかな?
奇数が出るたびに除けておいてあとで足し合わせる。

842 :ご冗談でしょう?名無しさん:2014/07/17(木) 12:42:12.89 ID:???.net
>>827

> Mα=Mg−(糸の張力)
> と
> mα=(糸の張力)ー(物体Aの遠心力)
> の連立で解くと

そんな式ではない

Mα=Mg−(糸の張力)
mα’=(糸の張力)

が正しい

Bが静止していてもAは加速度運動(円運動)しているのだから
明らかにAとBの加速度は同じではない

遠心力を導入するのもめんどうにするだけ

843 :ご冗談でしょう?名無しさん:2014/07/17(木) 13:10:11.61 ID:???.net
>>842
ちゃんと運動方程式たててもちゃんと遠心力の項と動径の二次微分の項に別れるけど
何が言いたいの?

844 :ご冗談でしょう?名無しさん:2014/07/17(木) 15:18:14.06 ID:???.net
分ける意味がないと言っている

845 :ご冗談でしょう?名無しさん:2014/07/17(木) 16:04:36.67 ID:???.net
直定規っていいのだと百万は下らないやん?あれ見てて思ったんだけんど母性の原理ってあるけんどあれどこまで遡れるのかな

846 :ご冗談でしょう?名無しさん:2014/07/17(木) 16:06:03.72 ID:???.net
つまり最高の直定規の精度はどうやって計るのかってことか
精密工学の難問だぬ

847 :ご冗談でしょう?名無しさん:2014/07/17(木) 17:56:00.60 ID:???.net
位相は量子論以前から整数性が高い。この場合の位相の意味はどっちも含む。

848 :ご冗談でしょう?名無しさん:2014/07/17(木) 18:09:01.43 ID:???.net
そうですね

849 :ご冗談でしょう?名無しさん:2014/07/17(木) 18:53:10.70 ID:???.net
>>845-841
位相は量子論以前から整数性が高い。この場合の位相の意味はどっちも含む。

850 :ご冗談でしょう?名無しさん:2014/07/17(木) 18:56:28.48 ID:???.net
干渉縞で姿勢のゆがみを測定する方法のこと何て呼ぶんですか?

851 :ご冗談でしょう?名無しさん:2014/07/17(木) 19:34:37.43 ID:???.net
それは残念ながら物理用語ではないんだ

852 :ご冗談でしょう?名無しさん:2014/07/17(木) 19:38:12.80 ID:???.net
一番幼稚な放射線医療をするぐらいの気持ちでひとつ物理分野扱いしてくださいよ?

853 :ご冗談でしょう?名無しさん:2014/07/17(木) 20:08:53.34 ID:gbFxF8fk.net
>>832の質問は>>832で解決しました
ありがとうございました

854 :ご冗談でしょう?名無しさん:2014/07/17(木) 21:47:42.28 ID:???.net
誰か>>836の質問に答えてください。お願いします。

855 :ご冗談でしょう?名無しさん:2014/07/17(木) 21:49:55.63 ID:???.net
>>854
ここへどうぞ
http://maguro.2ch.net/test/read.cgi/budou/1329235888/

856 :ご冗談でしょう?名無しさん:2014/07/17(木) 21:55:25.81 ID:???.net
>>855
教えてください。

857 :ご冗談でしょう?名無しさん:2014/07/17(木) 21:59:27.26 ID:???.net
>>856
ここで答えます
http://katahiromz.bbs.fc2.com/

858 :ご冗談でしょう?名無しさん:2014/07/17(木) 21:59:53.75 ID:???.net
>>855
な、荒らしに反応してもこういうことになるのさ。もうさわるなよ

859 :ご冗談でしょう?名無しさん:2014/07/17(木) 22:01:57.85 ID:???.net
>>858
物理板らしくスルーして

860 :ご冗談でしょう?名無しさん:2014/07/17(木) 22:05:22.91 ID:IMbEq2Qj.net
ローレンツ力はファラデーの電磁誘導の法則の特殊な場合ですか?
それともローレンツ力と電磁誘導の法則は数学的に同値ですか?

861 :ご冗談でしょう?名無しさん:2014/07/17(木) 22:11:46.60 ID:???.net
>>860
ローレンツ力は電場による力をローレンツ変換したもので、
電磁誘導の法則とは関係ない。

862 :ご冗談でしょう?名無しさん:2014/07/17(木) 22:13:55.07 ID:???.net
>>860
と思ったが電磁誘導の法則とは電場と磁場のローレンツ変換だからまあ関係はあるか。

863 :ご冗談でしょう?名無しさん:2014/07/17(木) 22:15:21.92 ID:???.net
>>860
と思ったが電磁誘導の法則とは電場と磁場のローレンツ変換だからまあ関係はあるか。

864 :ご冗談でしょう?名無しさん:2014/07/17(木) 22:28:00.39 ID:???.net
>>860
と思ったが電磁誘導の法則とは電場と磁場のローレンツ変換だからまあ関係はあるか。

865 :ご冗談でしょう?名無しさん:2014/07/17(木) 22:40:08.40 ID:???.net
関係ないよ

866 :ご冗談でしょう?名無しさん:2014/07/17(木) 23:24:48.20 ID:???.net
宇宙には全部で何個星が存在するのでしょうか?

867 :ご冗談でしょう?名無しさん:2014/07/17(木) 23:38:23.73 ID:???.net
一つあることは確実です

868 :ご冗談でしょう?名無しさん:2014/07/18(金) 08:07:56.78 ID:???.net
フェリーの船長と一流大学の理学部物理学科の教授はどっちの方が凄いのでしょうか?

869 :ご冗談でしょう?名無しさん:2014/07/18(金) 08:11:49.50 ID:vcsuHoKB.net
【STAP問題】小保方リーダー、早稲田大学の博士論文 6か所で不正も学位維持★5
http://peace.2ch.net/test/read.cgi/newsplus/1405619980/

870 :ご冗談でしょう?名無しさん:2014/07/18(金) 08:50:10.55 ID:???.net
ロシアの皇帝も英国王室も米国最大の財閥もドイツ系って本当ですか?

871 :ご冗談でしょう?名無しさん:2014/07/18(金) 08:51:07.40 ID:???.net
「AとBはどっちが凄い」と「メコ・・」のどっちが凄いのでしょうか?

872 :ご冗談でしょう?名無しさん:2014/07/18(金) 08:58:46.08 ID:???.net
ドイツ最大の天才は誰ですか?

873 :ご冗談でしょう?名無しさん:2014/07/18(金) 10:04:46.07 ID:???.net
カルロス・スリム・ヘルとNASAの主任科学者はどっちの方がIQが高いのでしょうか?

874 :佐藤光:2014/07/18(金) 12:25:27.97 ID:/Mpoj1wK.net
球形のダイヤモンドの中に空洞を作り、中に爆薬を入れ完全に密閉したあと爆発させたらどうなりますか?

875 :ご冗談でしょう?名無しさん:2014/07/18(金) 12:34:16.43 ID:???.net
小学生でも分かる

876 :ご冗談でしょう?名無しさん:2014/07/18(金) 13:46:34.02 ID:???.net


877 :ご冗談でしょう?名無しさん:2014/07/18(金) 21:22:28.34 ID:???.net
ダイアモンドをハンマーで叩いたらどうなりますか?

砕け散ります。

878 :ご冗談でしょう?名無しさん:2014/07/18(金) 21:28:07.72 ID:???.net
東大法学部と東大理学部はどっちの方がエリートですか?

879 :ご冗談でしょう?名無しさん:2014/07/18(金) 21:29:33.72 ID:???.net
人間は誰しも仕事をしてから評価されます

880 :ご冗談でしょう?名無しさん:2014/07/18(金) 21:32:13.00 ID:???.net
宇宙飛行士と最高裁長官はどっちの方がエリートですか?

881 :ご冗談でしょう?名無しさん:2014/07/18(金) 22:56:06.28 ID:???.net
>>880
エリートを物理的に定義した上でもう二度と来るな

882 :ご冗談でしょう?名無しさん:2014/07/19(土) 01:04:14.89 ID:???.net
スケート板からきたけどあのさフィギュアの人がさ回転早めるとき手を内側に狭めるんだけどさあれはなに、角運動量保存がされてるってこと?

883 :ご冗談でしょう?名無しさん:2014/07/19(土) 01:22:16.28 ID:???.net
>>882
そうだよ

884 :ご冗談でしょう?名無しさん:2014/07/19(土) 01:24:08.62 ID:???.net
>>883
サンクスω

885 :ご冗談でしょう?名無しさん:2014/07/19(土) 06:38:43.08 ID:2qVC2aTP.net
弦理論で世界面の計量をミンコフスキー計量にしたかったら、
一般座標変換で共形平坦にしてからワイル変換を行いますが、
一般座標変換だけでできないのでしょうか?
一般相対性理論でいつでも局所平坦に取れることと何が違うのですか?

886 :ご冗談でしょう?名無しさん:2014/07/19(土) 07:09:16.02 ID:???.net
それ明らかに程度の高すぎる問題だよね
こんな質問出来る俺かっこいいみたいな自己満足と違う?

887 :ご冗談でしょう?名無しさん:2014/07/19(土) 07:23:14.88 ID:2qVC2aTP.net
弦理論スレは人少ないしここの方が専門は違っても分かる人いる可能性が高いと思って質問しました

888 :ご冗談でしょう?名無しさん:2014/07/19(土) 07:24:51.73 ID:???.net
旅客機の操縦と旅客船の操縦はどっちの方が難しいのでしょうか?

889 :ご冗談でしょう?名無しさん:2014/07/19(土) 07:29:44.37 ID:???.net
まず弦理論を学ぶには「超弦理論←場の理論←量子力学・電磁気学・解析力学・統計力学・物理数学」の順で物理を理解する必要がある
学部レベルは第一段階の習得が精いっぱいでして、せいぜい理論系に配属された4年生が場の理論の初歩レベルの本を輪読する程度に過ぎない
一応学部で授業を設けている学校もあるが、せいぜい初歩のお話を聞いてレポートを数枚出すの関の山

専門外(しかも弦理論)の事を
本当に分かる人がここにいるのかねぇ

890 :ご冗談でしょう?名無しさん:2014/07/19(土) 07:30:28.40 ID:???.net
ここまでの内容を見る限りいなさそうだ

891 :ご冗談でしょう?名無しさん:2014/07/19(土) 07:31:55.24 ID:???.net
超弦理論->失われた20年

892 :ご冗談でしょう?名無しさん:2014/07/19(土) 08:15:48.19 ID:???.net
>>885
超弦理論/superstring theory【Part 2】
http://wc2014.2ch.net/test/read.cgi/sci/1379939285/

893 :ご冗談でしょう?名無しさん:2014/07/19(土) 11:06:46.46 ID:???.net
東京大学理科一類の受験科目を教えてください。

894 :ご冗談でしょう?名無しさん:2014/07/19(土) 12:13:12.44 ID:???.net
マーク試験と筆記試験です

895 :ご冗談でしょう?名無しさん:2014/07/19(土) 12:21:32.25 ID:???.net
真面目に教えてください。

896 :ご冗談でしょう?名無しさん:2014/07/19(土) 13:08:25.62 ID:???.net
分かる人が相手にするはずはない

897 :ご冗談でしょう?名無しさん:2014/07/19(土) 13:11:26.84 ID:J8pYRCib.net
すごい発見をしたかもしれない

F=-Gmm'/r^2
=α+Xとおく
kα=Gmm'とおく


X=
(-kα-αr^2)/r^2
=-α(k+r^2)/r^2

F=α-α(k+r^2)/r^2
=α-α(k/r^2+1)

F=α-(-F/α+1)α

F-F=0
0=0
つまり重力は存在しないのではないだろうか

898 :ご冗談でしょう?名無しさん:2014/07/19(土) 13:35:03.54 ID:???.net
じゃあ、そういうことで。

899 :ご冗談でしょう?名無しさん:2014/07/19(土) 13:36:33.28 ID:???.net
http://astro.sci.kagoshima-u.ac.jp/omodaka-nishio/member/kameno/Physics2A2007Thu/Answer081009.pdf

極座標の球殻の微小面積を視覚的に出す方法を探してこのPDFを見つけました。
1.の長さはrdθ(dθは微小量なので成立)
2.の長さはrdφ
というのは分かるのですが

3.の長さはrsinθdφってのがよく分かりません。
図だけを見るとrdφに見えるのですが…

900 :ご冗談でしょう?名無しさん:2014/07/19(土) 13:58:50.29 ID:???.net
xy平面に投射すると動径の長さを短く考える必要があるんだよ
この記述はそういうことを言いたいんじゃないかな

901 :ご冗談でしょう?名無しさん:2014/07/19(土) 14:02:16.87 ID:J8pYRCib.net
平面のベクトルをS(x(r,θ),y(r,θ)z(r,θ))として
面素ベクトルはdrS_r×dθS_θ
体積素は平行な成分を取り出せばいいから(drS_r×dθS_θ)・dφS_φ

902 :ご冗談でしょう?名無しさん:2014/07/19(土) 14:20:20.53 ID:J8pYRCib.net
間違った
r上の平面のベクトルはS(x(θ,φ),y(θ,φ)z(θ,φ))
面素ベクトルはdθS_θ×dφS_φ
dV=(dθS_θ×dφS_φ)・drS_r
=(S_θ×S_φ)・S_r dθdφdr

903 :ご冗談でしょう?名無しさん:2014/07/19(土) 14:30:37.29 ID:???.net
>>899
θ=0の時考えてみれば明らかにrdφじゃあないだろ

904 :ご冗談でしょう?名無しさん:2014/07/19(土) 14:36:11.48 ID:J8pYRCib.net
Bの長さは|S(θ,φ+Δφ)-S(θ,φ)|〜|S_θdφ|で計算すれば出てくるやで

zはrが決まってればφによらないから
z(θ)と書くべきやな

905 :ご冗談でしょう?名無しさん:2014/07/19(土) 18:04:39.04 ID:???.net
明けの明星を利用した物理学的に面白い実験ないですかね?

906 :ご冗談でしょう?名無しさん:2014/07/19(土) 23:15:27.81 ID:???.net
正接波という単語は使われていますか

907 :ご冗談でしょう?名無しさん:2014/07/19(土) 23:26:47.03 ID:???.net
正接波を仮定して波動方程式満たすか考えてみて

908 :ご冗談でしょう?名無しさん:2014/07/19(土) 23:29:25.26 ID:???.net
普通に満たすよ

909 : ◆PHySica/AE :2014/07/20(日) 01:15:44.33 ID:???.net
tanはもはや波じゃないっていう…

910 :俺、オレオレ:2014/07/20(日) 01:18:53.63 ID:???.net
西の空に明けの明星を見る方法知ってる?

911 :ご冗談でしょう?名無しさん:2014/07/20(日) 01:20:29.66 ID:???.net
いや知らない(´・ω・`)

912 :俺、オレオレ:2014/07/20(日) 01:43:55.92 ID:???.net
地球の自転(≒音速)より早い飛行機で西からの日の出、東への日没を実現できる。
これの再現が主目的でフライトシミュ買ったが実装は中途半端でバグった。

913 :ご冗談でしょう?名無しさん:2014/07/20(日) 01:57:40.92 ID:???.net
>>889
別に物理学系の実験系以上に
数学系の特に幾何分野だと研究テーマ求めて
物理学じゃない(笑)超弦理論の
お勉強リサーチするんじゃないですか?

914 :ご冗談でしょう?名無しさん:2014/07/20(日) 14:33:54.29 ID:???.net
理論物理学は天才的な素質がある奴以外専攻しても無駄だろ
超弦理論とか物理検証できないなら数学か宗教の分野だろ

915 : ◆PHySica/AE :2014/07/20(日) 14:41:16.48 ID:???.net
>>914
その通りだね
実証できてこそ役に立つ理論となり得る物理学

916 :ご冗談でしょう?名無しさん:2014/07/20(日) 15:10:48.09 ID:???.net
物性理論って知ってるか

917 :ご冗談でしょう?名無しさん:2014/07/20(日) 21:41:05.62 ID:???.net
-2πGmρz∫[0,a]ξ/(z^2+ξ^2)^3/2dξ=πGmρz/(z^2+ξ^2)^1/2[0,a]これの過程を誰か教えて下さい

918 :912:2014/07/20(日) 21:41:38.09 ID:???.net
途中で送ってしまった

919 :ご冗談でしょう?名無しさん:2014/07/20(日) 22:18:47.95 ID:???.net
>>917
普通に置換積分

920 :ご冗談でしょう?名無しさん:2014/07/21(月) 00:46:07.17 ID:???.net
何でもいいけど、座屈するより先に降伏するような設計をしたことある奴いる?

921 :ご冗談でしょう?名無しさん:2014/07/21(月) 01:20:19.73 ID:???.net
新入社員研修で覚えた用語を早速使いたかったんだね。

922 :ご冗談でしょう?名無しさん:2014/07/21(月) 01:32:49.61 ID:???.net
物理やってる奴らでも材力ある程度出来るの?

923 :ご冗談でしょう?名無しさん:2014/07/21(月) 01:35:58.71 ID:???.net
もの作りに対してはてんで素人
理学と工学は違う(´・ω・`)

924 :ご冗談でしょう?名無しさん:2014/07/21(月) 01:43:40.28 ID:???.net
東京大学理学部情報科学科に入ってやっていけるには、最低限どれくらいの知識がないと駄目ですか?

925 :ご冗談でしょう?名無しさん:2014/07/21(月) 01:50:57.26 ID:GddERxCd.net
1+1=の計算ができるくらい
もちろん「2」なんて間違った答えを言っちゃう人には向いてない

926 :ご冗談でしょう?名無しさん:2014/07/21(月) 01:51:35.58 ID:???.net
四則演算が出来ればいいって良く言われてる

927 :ご冗談でしょう?名無しさん:2014/07/21(月) 01:56:53.44 ID:???.net
東大理学部情報科学科に入るまでに身に付けておかないといけない知識は何ですか?

928 :ご冗談でしょう?名無しさん:2014/07/21(月) 02:17:14.83 ID:???.net
誰か>>927の質問に答えてください。

929 :ご冗談でしょう?名無しさん:2014/07/21(月) 03:24:48.94 ID:???.net
全ての言語に対応した超巨大掲示板や超巨大動画共有サイトを作って大金持ちになって、
ラーメンとか寿司とか蟹とか帆立の貝柱とかステーキとかを超いっぱい食いたい。

930 :ご冗談でしょう?名無しさん:2014/07/21(月) 07:26:33.34 ID:???.net
数学や理学は工学の基本的なツール(ラプラス変換など)となって素晴らしいけれど
一番使えないのが中途に理学をかじってこういうところでしたり顔してる奴ら

931 :ご冗談でしょう?名無しさん:2014/07/21(月) 10:26:56.20 ID:???.net
大型フェリーの船長とジャンボジェット機の機長はどっちの方が給料が高いのでしょうか?

932 :ご冗談でしょう?名無しさん:2014/07/21(月) 10:45:17.70 ID:5WLQSSK3.net
≫897
dr だよ

933 :ご冗談でしょう?名無しさん:2014/07/21(月) 11:38:07.01 ID:???.net
まともな質問がないとゴミが湧くな

934 :ご冗談でしょう?名無しさん:2014/07/21(月) 11:47:19.90 ID:???.net
>>930
また、おまえか

935 :ご冗談でしょう?名無しさん:2014/07/21(月) 11:49:15.92 ID:???.net
誰か>>927>>931の質問に答えてください。

936 :ご冗談でしょう?名無しさん:2014/07/21(月) 11:52:56.49 ID:???.net
>>935
こちらへどうぞ
http://peace.2ch.net/test/read.cgi/gender/1356576924/

937 :ご冗談でしょう?名無しさん:2014/07/21(月) 13:02:07.13 ID:SsdMhZCY.net
教科書の問題には、いつも
空気抵抗がない時や
床との摩擦力が無い時
と、但し書きがあるけど
これじゃあ、実用にならないんじゃないか

938 :ご冗談でしょう?名無しさん:2014/07/21(月) 13:22:21.27 ID:???.net
>>937
それすらわからんやつが使い物にならんのよ。

939 :ご冗談でしょう?名無しさん:2014/07/21(月) 13:24:06.47 ID:GddERxCd.net
別に実用を目的にしてないから

940 :ご冗談でしょう?名無しさん:2014/07/21(月) 14:27:31.43 ID:???.net
難易度から考えて、工学は理学より格下ということで間違ってませんか?

941 :ご冗談でしょう?名無しさん:2014/07/21(月) 14:29:24.78 ID:???.net
馬鹿らしい

942 :ご冗談でしょう?名無しさん:2014/07/21(月) 14:42:03.39 ID:GddERxCd.net
目の前の箱すら一人で作れないチンカスがなんか言ってるけどよく聞こえない

943 :ご冗談でしょう?名無しさん:2014/07/21(月) 15:13:15.97 ID:???.net
と、チンカス以下のゴミがほざいております

944 :ご冗談でしょう?名無しさん:2014/07/21(月) 15:26:05.59 ID:???.net
そもそも分野が違うんだから比べられない。それを比べようとしているやつは滑稽だな。

945 :ご冗談でしょう?名無しさん:2014/07/21(月) 19:50:55.57 ID:???.net
>>899
図無しでは説明しにくいのですが理解出来ました。
半球上の微笑面積なのが理由だったのですね。
つまり、真ん中から上に行くにつれて球全体が狭くなってその割合がsinθに比例していると

見落としも少ないし解析的に出す方が楽ですねこれ…

946 :ご冗談でしょう?名無しさん:2014/07/21(月) 22:36:39.76 ID:???.net
物体に働く力を分解するときの角度θをおく場所はどのように決めたら良いのでしょうか?

947 :ご冗談でしょう?名無しさん:2014/07/21(月) 22:42:52.70 ID:???.net
>>946
計算するのに都合のいいところ。

948 :ご冗談でしょう?名無しさん:2014/07/21(月) 22:51:06.58 ID:???.net
>>947
答えと違うほうに置いて計算したら違う答えが出てきました…

949 :ご冗談でしょう?名無しさん:2014/07/21(月) 22:58:51.67 ID:GddERxCd.net
独立の基底を二つ選んで正射影

950 :ご冗談でしょう?名無しさん:2014/07/22(火) 01:19:47.86 ID:???.net
そりゃあ違うように置いたら違う値になるだろうよ

951 :ご冗談でしょう?名無しさん:2014/07/22(火) 04:40:39.02 ID:???.net
金儲けが第一の人は、やはり、実業家になるのが良いのでしょうか?

952 :ご冗談でしょう?名無しさん:2014/07/22(火) 06:23:16.23 ID:???.net
旅客機のパイロットとかタンカーの船長とかはエリートの部類に入りますか?

953 :ご冗談でしょう?名無しさん:2014/07/22(火) 07:27:17.77 ID:???.net
>>952
エリートde勝ち組です

954 :ご冗談でしょう?名無しさん:2014/07/22(火) 12:39:24.01 ID:???.net
医師と宇宙飛行士はどっちの方が多くの知識を必要としますか?

955 :ご冗談でしょう?名無しさん:2014/07/22(火) 13:45:50.92 ID:???.net
俺はどちらかというとカレーうどんが好きだな

956 :ご冗談でしょう?名無しさん:2014/07/22(火) 14:17:28.08 ID:???.net
東京大学理学部情報科学科に入るまでに勉強しとかないと駄目なことを全て教えてください。

957 :ご冗談でしょう?名無しさん:2014/07/22(火) 14:26:15.19 ID:???.net
>>12
チャプマンいたところでキンブレルは格が違いすぎる

958 :ご冗談でしょう?名無しさん:2014/07/22(火) 14:30:40.68 ID:???.net
なぜ物理板できく

959 :ご冗談でしょう?名無しさん:2014/07/22(火) 17:02:08.11 ID:???.net
>>956
40過ぎた偏差値40のおっさんには無理です
http://maguro.2ch.net/test/read.cgi/budou/1329235888/

960 :ご冗談でしょう?名無しさん:2014/07/22(火) 18:02:59.37 ID:???.net
ゴミはゴミ箱へ

961 :ご冗談でしょう?名無しさん:2014/07/22(火) 23:13:04.13 ID:???.net
国際物理五輪は銀4・銅1…参加全員がメダル
http://www.yomiuri.co.jp/science/20140720-OYT1T50116.html?from=ytop_ylist

このニュース、肝心の金がどの国なのか書いてないけど
まさか中漢だらけじゃないだろうな

962 :ご冗談でしょう?名無しさん:2014/07/22(火) 23:15:38.13 ID:???.net
どの国っていろんな国だよ

963 :ご冗談でしょう?名無しさん:2014/07/22(火) 23:19:46.51 ID:???.net
韓国は全員金らしいけどな

964 :ご冗談でしょう?名無しさん:2014/07/22(火) 23:21:47.07 ID:???.net
韓国の留学生いて向こうの高校物理の参考書見たがレベル高かったな

965 :ご冗談でしょう?名無しさん:2014/07/22(火) 23:24:13.03 ID:???.net
運動方程式の補足としてラグランジュ方程式が出てたりな
偏微分もやってるってことだ

966 :ご冗談でしょう?名無しさん:2014/07/22(火) 23:27:16.13 ID:???.net
シンガポールだと中3で微積分が出て来たり国によって様々ですな

967 :ご冗談でしょう?名無しさん:2014/07/23(水) 00:35:16.43 ID:???.net
光の円偏光がどうしても理解できません。
直線偏光はわかるのですが、なぜそれが回転するのか、回転するというのは直感的にどういうことなのか、なぜ直線偏光と円偏光が相互に変換できるのか、わかりません。
どなたかわかりやすく教えてくださるか、わかりやすいサイトまたは教科書を教えていただけませんか。

968 :ご冗談でしょう?名無しさん:2014/07/23(水) 00:37:17.54 ID:???.net
直線偏光の重ね合わせから作られる楕円偏光もまたマクスウェル方程式の解になる横波の電磁波だから

969 :ご冗談でしょう?名無しさん:2014/07/23(水) 00:40:22.35 ID:???.net
前野あたりがアプレット組めば直感的な理解は進みそうだけど

あれほど計算はできるのにイメージが分かんもんもそうそうない<偏光

970 :ご冗談でしょう?名無しさん:2014/07/23(水) 01:45:19.30 ID:???.net
右手の親指,人差し指,中指の順に波数ベクトル,電場振幅,磁場振幅だと思って
親指の方向に進みながら親指周りに回転するのが円偏光

971 :ご冗談でしょう?名無しさん:2014/07/23(水) 07:16:33.73 ID:???.net
飽和潜水士の給料ってどのくらいなんでしょうか?

972 :ご冗談でしょう?名無しさん:2014/07/23(水) 07:31:25.56 ID:???.net
>>967
光子はネジと同じで回転(スピン)しながら空間を光速で進行する。
ローレンツ変換で不変の左回転の光子と右回転の光子の2状態があり
2つの位相差を固有にするフィルタ(orアンテナ)で円偏光と直線偏光ができる。

973 :ご冗談でしょう?名無しさん:2014/07/23(水) 08:10:46.04 ID:???.net
偏光の説明で出てくる線とか円とかって、磁場を表してるの?電場を表してるの?

974 :ご冗談でしょう?名無しさん:2014/07/23(水) 08:16:05.37 ID:???.net
両方だよ
つかそれどの教科書にも書いてあるやん

975 :ご冗談でしょう?名無しさん:2014/07/23(水) 08:17:05.19 ID:???.net
両方。電磁波だから電場も磁場もある

976 :ご冗談でしょう?名無しさん:2014/07/23(水) 08:20:10.80 ID:???.net
物理に詳しいのとコンピュータに詳しいのだと、どっちの方が凄いですか?

977 :ご冗談でしょう?名無しさん:2014/07/23(水) 09:13:00.25 ID:???.net
従来の電磁波説では光子一個でも起きる偏光が説明できないだろ

978 :ご冗談でしょう?名無しさん:2014/07/23(水) 09:23:35.33 ID:???.net
>>977
何が説明でけへんの

979 :ご冗談でしょう?名無しさん:2014/07/23(水) 11:35:14.72 ID:???.net
コンピュータを極めれば年収億単位になりますか?

980 :ご冗談でしょう?名無しさん:2014/07/23(水) 11:49:12.56 ID:???.net
万単位にはなります

981 :ご冗談でしょう?名無しさん:2014/07/23(水) 12:08:24.91 ID:???.net
億単位にはなりませんか?

982 :ご冗談でしょう?名無しさん:2014/07/23(水) 12:58:51.31 ID:???.net
ゴミは去れ

983 :ご冗談でしょう?名無しさん:2014/07/23(水) 13:08:53.04 ID:???.net
と、ゴミがほざいております。

984 :ご冗談でしょう?名無しさん:2014/07/23(水) 19:25:47.05 ID:???.net
ギブス和のZの花文字のいい書き方ないですか?分配関数と区別したいのです

985 :ご冗談でしょう?名無しさん:2014/07/23(水) 19:30:38.82 ID:???.net
グランドカノニカル分布の分配関数なら普通はギリシャ文字のΞ(ξの大文字)で表す。

986 :ご冗談でしょう?名無しさん:2014/07/23(水) 19:38:23.71 ID:???.net
>>985さん
返信有難うございます
Ξで表す流儀もあるようですが、キッテル『熱物理学』(丸善)ですと、大分配関数の表記がZの飾り文字で統一されているようですので、そちらの表記に準拠したかったのです。

987 :ご冗談でしょう?名無しさん:2014/07/23(水) 19:41:38.84 ID:???.net
>>986
そうしたいのならしろ。以上。

988 :ご冗談でしょう?名無しさん:2014/07/23(水) 19:45:24.52 ID:???.net
>>987さん
書き方を聞いてるのですが

989 :ご冗談でしょう?名無しさん:2014/07/23(水) 19:55:22.85 ID:???.net
好きにすれば

990 :ご冗談でしょう?名無しさん:2014/07/23(水) 20:37:23.18 ID:???.net
ぐとかにしたらオリジナリティある

991 :ご冗談でしょう?名無しさん:2014/07/23(水) 20:48:09.37 ID:???.net
Zに下付き文字のぐにします!

992 :ご冗談でしょう?名無しさん:2014/07/23(水) 21:13:51.79 ID:???.net
「大」でええやん

993 :ご冗談でしょう?名無しさん:2014/07/23(水) 21:41:46.14 ID:???.net
光度の単位ってもうちょっとどうにかならんの?

994 :ご冗談でしょう?名無しさん:2014/07/23(水) 22:18:14.55 ID:???.net
>>993
というと?

995 :ご冗談でしょう?名無しさん:2014/07/23(水) 22:21:07.59 ID:???.net
カンデーラ

996 :ご冗談でしょう?名無しさん:2014/07/23(水) 22:21:43.27 ID:???.net
>>994
ルーメンとかルクスとかカンデラとか

634nmを基準に一定の分布を作るようにするとかなんつーか

997 :ご冗談でしょう?名無しさん:2014/07/23(水) 22:24:15.37 ID:???.net
次スレ立てます

998 :ご冗談でしょう?名無しさん:2014/07/23(水) 22:24:28.44 ID:???.net
なるほど
光度の定義自体、人間に合わせて規定されてるからのう

999 :ご冗談でしょう?名無しさん:2014/07/23(水) 22:27:00.79 ID:???.net
次スレ立てます
■ちょっとした物理の質問はここに書いてね181■
http://wc2014.2ch.net/test/read.cgi/sci/1406121891/

1000 :ご冗談でしょう?名無しさん:2014/07/24(木) 01:23:14.15 ID:???.net
>>996
1979年から、だいたいそんな感じの定義になってるみたいだよ。
カンデラの定義: カンデラは、周波数 540×1012 ヘルツの単色放射を放出し、所定の方向における
その放射強度が 1/683 ワット毎ステラジアンである光源の、その 方向における光度である。(第16回CGPM, 1979)

https://www.nmij.jp/library/units/photo/

実際にはカロリメトリーで放射強度はかるみたいね。

1001 :ご冗談でしょう?名無しさん:2014/07/24(木) 01:46:17.36 ID:???.net
>>1000
やるな。大塚食品。

1002 :ご冗談でしょう?名無しさん:2014/07/24(木) 18:12:30.53 ID:???.net
>>1001
それを言うなら大塚製薬

1003 :ご冗談でしょう?名無しさん:2014/07/24(木) 18:25:32.87 ID:???.net
>>1002
ほんとだ、気付いてなかったわ。

1004 :ご冗談でしょう?名無しさん:2014/07/24(木) 18:39:27.16 ID:AEfYZCGn.net
ああカロリーメイトということか

1005 :ご冗談でしょう?名無しさん:2014/07/25(金) 00:02:06.59 ID:???.net
とっとと埋めようよ

1006 :2ch.net投稿限界:Over 1000 Thread
2ch.netからのレス数が1000に到達しました。

総レス数 1006
219 KB
掲示板に戻る 全部 前100 次100 最新50
read.cgi ver.24052200